IASBaba 29

You might also like

Download as pdf or txt
Download as pdf or txt
You are on page 1of 80

IAS BABA

TEST SERIES
TEST-NO-29
�हंद� माध्यम
2020
AIPTS/ILP VETERANS-
Exam Title :
2020 TE...
Email : yadavanurag075@gmail.com
Contact : 8882839768

Review in Hindi
QUESTION 1.
भरत�इ�तहास�समशोधक�मंडल�( BISM) क���थापना�1910 म���कसके��ारा��ई�थी:

a) �व�नाथ�काशीनाथ�राजवाड़े�
b) रामकृ�ण�गोपाल�भंडारकर�
c) राज���लाल��म�ा�
d) के.ए. नीलकंठ�शा��ी�
QUESTION 2.
भीमा-कोरेगांव�क��लड़ाई��न�न�ल�खत�म��से��कस�यु��का��ह�सा�थी�?

a) चौथा�आं�ल-मैसूर�यु��
b) �सरा�आं�ल-मराठा�यु��
c) तीसरा�आं�ल-मराठा�यु��
d) इनम��से�कोई�भी�नह��
QUESTION 3.
�न�न�ल�खत�कथन��पर��वचार�कर�:

1. वह�एक��श�क�बनने�वाली�पहली�भारतीय�म�हला�थ�।�
2. उ�ह�ने�क�या��ूण�ह�या�को�रोकने�तथा�गृह�, बालह�या���तबंधक�गृह��था�पत�करने�क���दशा�म��भी�काम��कया।�
3. उसने�बावन�काशी�सुबोध�र�नाकर��लखी।�

उपयु���कथन��न�न�ल�खत�म��से��कस�समाज�सुधारक�से�जुड़े�ह�:

a) फा�तमा�शेख�
b) पं�डत�रमाबाई�
c) रमाबाई�रानाडे�
d) सा�व�ीबाई�फुले�
QUESTION 4.
�न�न�ल�खत�कथन��पर��वचार�कर�:

1. उ�ह�ने�1899 के�कलक�ा�नगर�संशोधन�अ�ध�नयम�के�मा�यम�से�कलक�ा��नगम�के��नवा��चत���त�न�धय��क��सं�या�म��कमी�क�।�
2. 1904 के�भारतीय��व��व�ालय��अ�ध�नयम�के�मा�यम�से�उ�ह�ने�भारत�म���व��व�ालय��पर�सरकारी��नयं�ण�बढ़ा��दया।�
3. 1904 के�भारतीय�आ�धका�रक�गोपनीयता�संशोधन�अ�ध�नयम�के�तहत�उ�ह�ने��ेस�क���वतं�ता�को���तबं�धत�कर��दया।�

�न�न�ल�खत�म��से�कौन�सा�गवन�र�जनरल�उपरो��कथन��से�संब��है�?

a) लॉड��डलहौजी�
b) लॉड��कज�न�
c) लॉड���लटन�
d) लॉड���म�टो�
QUESTION 5.
मदन�मोहन�मालवीय�के�संदभ��म���न�न�ल�खत�कथन��पर��वचार�कर�:

1. उ�ह�ने�बनारस��ह����व��व�ालय�क���थापना�म��योगदान��दया।�
2. उ�ह�ने�स�वनय�अव�ा�आंदोलन�म��भाग�नह���लया�

IASbaba
Web: http://ilp.iasbaba.com/
Email: ilp@iasbaba.com
Page 34
AIPTS/ILP VETERANS-
Exam Title :
2020 TE...
Email : yadavanurag075@gmail.com
Contact : 8882839768

3. उ�ह��मरणोपरांत�दे श�के�सव��च�नाग�रक�स�मान�भारत�र�न�से�स�मा�नत��कया�गया�था।�

ऊपर��दए�गए�कथन��म��से�कौन�सा�सही�है�/ ह��?

a) केवल�1 और�2
b) केवल�2 और�3
c) केवल�1 और�3
d) 1, 2 और�3
QUESTION 6.
�न�न�ल�खत�कथन��पर��वचार�कर�:

1. भारतीय�इ�तहास�कां�ेस�( IHC) का�ज�म�पुणे�, महारा��म��लगभग�50 ��त�न�धय��के�साथ��आ�था।�


2. नवंबर�1919 म��पहला�अ�खल�भारतीय��ा�य�स�मेलन�( All India Oriental Conference) बंगाल�म��आयो�जत��कया�गया
था।�

ऊपर��दए�गए�कथन��म��से�कौन�सा�गलत�है�/ ह��?

a) केवल�1
b) केवल�2
c) 1 और�2 दोन��
d) न�तो�1 और�न�ही�2
QUESTION 7.
आय��समाज�के�संदभ��म���न�न�ल�खत�कथन��पर��वचार�कर�:

1. आय��समाज�ने�शु���क��अवधारणा�को��वक�सत��कया�, �जसका�उ�े �य�ईसाई�धम��, इ�लाम�और��सख�धम��से�पुनःप�रवत�न�का�था।�


2. इसने�मू�त�पूजा�, ब�दे ववाद�, बाल��ववाह�, �वधवा���चय��, �ाहमण��के��भु�व�और�जा�त��व�था�जैसी�मौजूदा��ह����था��क�
तीखी�आलोचना�क�।�
3. लाला�लाजपत�राय�आय��समाज�के�सं�थापक��ववेकानंद�के�अनुयायी�बन�गए�

ऊपर��दए�गए�कथन��म��से�कौन�सा�सही�है�/ ह��?

a) केवल�1 और�2
b) केवल�2 और�3
c) केवल�1 और�3
d) 1, 2 और�3
QUESTION 8.
“ पावट��एंड�अन���टश��ल�इन�इं�डया�” �कसके��ारा��लखा�गया�था�

a) दादाभाई�नौरोजी�
b) एम�जी�रानाडे�
c) आर�सी�द��
d) महा�मा�गांधी�
QUESTION 9.
पानीपत�क��तीसरी�लड़ाई�( 1761) के�संदभ��म���न�न�ल�खत�कथन��पर��वचार�कर�:

1. यह�मराठा�सा�ा�य�और��रा�नी�सा�ा�य�(अफगा�न�तान) क��सेना��के�बीच�लड़ा�गया�था।�
2. बाजीराव�पेशवा�मराठा�सेना�के��मुख�कमांडर�थे।�

IASbaba
Web: http://ilp.iasbaba.com/
Email: ilp@iasbaba.com
Page 35
AIPTS/ILP VETERANS-
Exam Title :
2020 TE...
Email : yadavanurag075@gmail.com
Contact : 8882839768

3. शाह�आलम���तीय�यु��के�दौरान�मुगल�स�ाट�था।�

ऊपर��दए�गए�कथन��म��से�कौन�सा�सही�है�/ ह��?

a) केवल�1
b) केवल�2 और�3
c) केवल�1 और�3
d) 1, 2 और�3
QUESTION 10.
�न�न�ल�खत�कथन��पर��वचार�कर�:

1. झांसी�क��रानी�क��मृ�यु�कै�टन�हे�गे�( Captain Heneage) के�नेतृ�व�म��8 व���सर��( 8th Hussars) के�एक�द�ते�से�लड़ाई


के�दौरान��ई�
2. झाँसी�पहली�ऐसी��रयासत�थी��जसे�अं�ेज़��ने��पगत�के��स�ांत�के�तहत�अ�ध��हत��कया�था।�

ऊपर��दए�गए�कथन��म��से�कौन�सा�सही�है�/ ह��?

a) केवल�1
b) केवल�2
c) 1 और�2 दोन��
d) न�तो�1 और�न�ही�2
QUESTION 11.
�न�न�ल�खत�म��से��कस�भारतीय��ां�तकारी�ने�कन�ल�रे�जना�ड�एडवड��हैरी�डायर�क��ह�या�क��थी�?

a) मदन�लाल�ढ�गरा�
b) उधम��स�ह�
c) खुद�राम�बोस�
d) इनम��से�कोई�भी�नह��
QUESTION 12.
�न�न�ल�खत�कथन��पर��वचार�कर�:

1. आज़ाद��ह�द�फ़ौज�का�गठन�दो�बार�- कै�टन�मोहन��स�ह�और�सुभाष�चं��बोस�के�अंतग�त��आ�था।�
2. कै�टन�मोहन��स�ह�भारतीय�रा�ीय�सेना�के�पहले�कमांडर�बने।�
3. बोस�ने�जम�नी�क��रे�डयो�सेवा�के��ह�से�के��प�म��आज़ाद��ह�द�रे�डयो�शु���कया�, जो�पहली�बार�7 जनवरी�1942 को��स�गापुर�म�
�सा�रत��आ।�

ऊपर��दए�गए�कथन��म��से�कौन�सा�सही�है�/ ह��?

a) केवल�1 और�2
b) केवल�2
c) केवल�1 और�3
d) केवल�2 और�3
QUESTION 13.
" सव�ट्स�ऑफ�इं�डया�सोसाइट��" �कसके��ारा��था�पत��कया�गया�था�

a) बाल�गंगाधर��तलक�

IASbaba
Web: http://ilp.iasbaba.com/
Email: ilp@iasbaba.com
Page 36
AIPTS/ILP VETERANS-
Exam Title :
2020 TE...
Email : yadavanurag075@gmail.com
Contact : 8882839768

b) एम�के�गांधी�
c) गोपाल�कृ�ण�गोखले�
d) दादाभाई�नौरोजी�
QUESTION 14.
1946 म��अंत�रम�सरकार�के�गठन�के�संदभ��म���न�न�ल�खत�कथन��पर��वचार�कर�:

1. यह�भारत�के�इ�तहास�का�एकमा��ऐसा�मं��मंडल�था��जसम��क�र���त�ं ���कां�ेस�और�मु��लम�लीग�ने�क���म��स�ा�साझा�क�।�
2. आ�खरकार�मु��लम�लीग�के���त�न�धय���ारा�मुसलमान��के��लए�आर��त�सभी�पाँच�मं�ालय��पर�अ�धकार�कर��लया�गया।�
3. �ारंभ�म��, मौलाना�अबुल�कलाम�आज़ाद�को�मं��मंडल�म��शा�मल�नह���कया�गया�था।�

ऊपर��दए�गए�कथन��म��से�कौन�सा�सही�है�/ ह��?

a) केवल�1 और�2
b) केवल�2 और�3
c) केवल�1 और�3
d) 1, 2 और�3
QUESTION 15.
�न�न�म��से�कौन�सा�����व�मुंबई�क��ए�शया�टक�सोसाइट��का�सं�थापक�है�?

a) सर��व�लयम�जो�स�
b) जे�स����सेप�
c) सर�चा�स���व��कंस�
d) सर�जे�स�मै�कनटोस�
QUESTION 16.
�न�न�ल�खत�कथन��पर��वचार�कर�:

1. वह�भारत�के�पहले�उपरा�प�त�और��सरे�रा�प�त�थे।�
2. उ�ह�ने�अ�ै त�वेदांत�क��सही��श�ा��का��सार�करने�क��मांग�क�।�
3. उनके��मुख�काय��म��भारतीय�दश�न�, लेख�‘ द�ए�थ�स�ऑफ�द�भगवद�गीता�और�कांट�’ शा�मल�ह�।�

उपयु���कथन��से�कौन-सा�����व�संब��है�?

a) लाल�बहा�र�शा��ी�
b) डॉ. सव�प�ली�राधाकृ�णन�
c) वराह�ग�र�व�कट��ग�र�
d) डॉ. जा�कर��सैन�
QUESTION 17.
�न�न�ल�खत�म��से�कौन�सी�घटना�स�वनय�अव�ा�आंदोलन�क��समा��त�के�बाद�नह���ई�?

a) जय�काश�नारायण�, अचुत�पटवध�न�, अशोक�मेहता�, यूसुफ�मेहरअली�, नर���दे व�और�मीनू�मसानी�ने��वतं��पाट��(सीएसपी) का�गठन


�कया।�
b) रा�ीय�आंदोलन�क��भावी�रणनी�त�को�लेकर�भारतीय�रा�ीय�कां�ेस�म��आंत�रक�मतभेद�उभर�कर�सामने�आए।�
c) गांधी�जी�ने�कां�ेस�से�अपने�इ�तीफे�क��घोषणा��वचार�, श�द�और�कम��म��बेहतर�तरीके�से�करने�के��लए�क�।�
d) दोन���वक�प�( और�(
QUESTION 18.

IASbaba
Web: http://ilp.iasbaba.com/
Email: ilp@iasbaba.com
Page 37
AIPTS/ILP VETERANS-
Exam Title :
2020 TE...
Email : yadavanurag075@gmail.com
Contact : 8882839768

�न�न�ल�खत�म��से��कस�जन�आंदोलन�क��शु�आत�करते��ए�, गांधी�ने�इसे�" मेरे�जीवन�क��सबसे�बड़ी�लड़ाई�" कहा�?

a) स�वनय�अव�ा�आंदोलन�
b) सां�दा�यक�अवाड��के��खलाफ�उपवास�
c) असहयोग�आंदोलन�
d) भारत�छोड़ो�आंदोलन�
QUESTION 19.
लाल�बहा�र�शा��ी�के�संदभ��म���न�न�ल�खत�कथन��पर��वचार�कर�:

1. वे�लाला�लाजपत�राय��ारा��था�पत�सव�ट्स�ऑफ�द�पीपुल�सोसाइट��(लोक�सेवक�मंडल) के�आजीवन�सद�य�बने।�
2. उ�ह�ने�असहयोग�आंदोलन�और�नमक�स�या�ह�म��भाग��लया।�
3. नेह��क��बीमारी�के�दौरान�, वह�भारत�के�र�ा�मं�ी�थे।�

ऊपर��दए�गए�कथन��म��से�कौन�सा�सही�है�/ ह��?

a) केवल�1 और�2
b) केवल�1 और�3
c) केवल�3
d) केवल�2 और�3
QUESTION 20.
होम��ल�आंदोलन�का�उ�े �य��या�था�?

a) ���टश�कानून��क��अव�ा�करना�तथा�शासन�करना�क�ठन�बना�दे ना।�
b) भारतीय��को�स�ा�का�पूण��ह�तांतरण।�
c) ���टश�सा�ा�य�के�भीतर�डो�म�नयन��टे टस��ा�त�करना।�
d) �वदे शी�व�तु��का�ब�ह�कार�
QUESTION 21.
�न�न�ल�खत�म��से�कौन�सा�1939 म��सात��ांत��म��कां�ेस�के�मं��य��के�इ�तीफे�का�सही�कारण�है�?

a) दे श�म��बढ़ते�समाजवाद��और�सा�यवाद���भाव�ने�कां�ेस�के�मं��य��के��लए�काम�करना�मु��कल�कर��दया।�
b) दे श�म���ापक�सां�दा�यक�तनाव।�
c) भारतीय��के�परामश��के��बना�वायसराय�के�भारत�को���तीय��व��यु��के��लए�पाट��घो�षत�करने�के��वरोध�म��
d) सं�वधान�सभा�का�गठन�करने�के�कां�ेस�के���ताव�को�अ�वीकार�करने�के�कारण।�
QUESTION 22.
ई�र�चं���व�ासागर�के�संदभ��म���न�न�ल�खत�कथन��पर��वचार�कर�:

1. उ�ह�ने�सं�कृत��वभाग�म���मुख�पं�डत�के��प�म��फोट� ��व�लयम�कॉलेज�म���वेश��लया।�
2. उ�ह��1851 म��सं�कृत�कॉलेज�का�����सपल��नयु���कया�गया�था।�
3. उनका��मुख�योगदान�1829 म��सती��था�को�समा�त�करने�वाले�कानून�को�लागू�करने�के��लए����टश�सरकार�को�राजी�करना�था।�

ऊपर��दए�गए�कथन��म��से�कौन�सा�सही�है�/ ह��?

a) केवल�1 और�2
b) केवल�2 और�3
c) केवल�1 और�3
d) 1, 2 और�3

IASbaba
Web: http://ilp.iasbaba.com/
Email: ilp@iasbaba.com
Page 38
AIPTS/ILP VETERANS-
Exam Title :
2020 TE...
Email : yadavanurag075@gmail.com
Contact : 8882839768

QUESTION 23.
भारतीय��वतं�ता�सं�ाम�क��अव�ध�के�संदभ��म��, नेह���रपोट� �म���न�न�ल�खत�म��से��कसक���सफा�रश�क��गई�थी�?

1. भारत�के��लए�पूण���वतं�ता�
2. अ�पसं�यक��के��लए�सीट��के�आर�ण�के��लए�संयु���नवा�चन।�
3. सं�वधान�म��भारत�के�लोग��के��लए�मौ�लक�अ�धकार��का��ावधान।�

ऊपर��दए�गए�कथन��म��से�कौन�सा�सही�है�/ ह��?

a) केवल�2
b) केवल�2 और�3
c) केवल�3
d) 1, 2 और�3
QUESTION 24.
�न�न�ल�खत�कथन��पर��वचार�कर�:

1. 1920 म��, �व�लभाई�पटे ल�, महा�मा�गांधी�और�बाल�गंगाधर��तलक�स�हत�कई��मुख��वतं�ता�सेना�नय��ने�जेल�से�उनक���रहाई�क�


मांग�क�।�
2. उ�ह�ने�‘ �ह���व: �ह���कौन�है�?’ पु�तक��लखी�
3. उनके�नेतृ�व�म��, �ह���महासभा�ने�मु��लम�लीग�के�साथ��मलकर��स�ध�, बंगाल�और�उ�र�प��म�सीमा��ांत�जैसे��ांत��म��सरकार�बनाई।�

भारत�के��वतं�ता�सं�ाम�म���न�न�ल�खत�म��से�कौन�सा�रा�वाद��नेता�उपरो��कथन��से�संब��है�?

a) केशव�ब�लराम�हेडगेवार�
b) मदन�मोहन�मालवीय�
c) लाला�लाजपत�राय�
d) �वनायक�दामोदर�सावरकर�
QUESTION 25.
भले�ही�1857 के��व�ोह�क��पूव�-योजना�पर�संदेह�हो�, ले�कन��व�ोह�शु��होते�ही�योजना�बनाने�के��माण��मलते�ह�।��व�ोह�शु��होने�के�बाद
�व�ो�हय���ारा��कए�गए��यास��क��योजना��न�न�म��से�कौन�सी�है�?

1. सभी�पड़ोसी�रा�य��के�शासक��को�एक�प��संबो�धत��कया�गया�, �जसम��उनका�समथ�न�करने�और�उ�ह��भाग�लेने�के��लए�आमं��त
करने�के��लए�कहा�गया।�
2. �द�ली�म��, �शासक��क��एक�अदालत��था�पत�क��गई�, जो�रा�य�के�सभी�मामल��के��लए�उ�रदायी�थी।�
3. आंदोलन�को�वैधता��दान�करने�के��लए��स�के�मुगल�स�ाट�के�नाम�पर�जारी��कए�गए�तथा�आदे श�जारी��कए�गए।�

सही�कूट�का�चयन�कर�:

a) केवल�2
b) केवल�2 और�3
c) केवल�3
d) 1, 2 और�3
QUESTION 26.
�न�न�ल�खत�म��से��कस�राजनी�तक�दाश��नक�ने�अपने�महान�काय��‘ ए��ह����ऑफ����टश�इं�डया�’ म��भारतीय�इ�तहास�को�तीन�अव�धय�- �ह��
, मु��लम�और����टश�म���वभा�जत��कया�है�?

a) थॉमस�ब�ब�गटन�मैकाले�

IASbaba
Web: http://ilp.iasbaba.com/
Email: ilp@iasbaba.com
Page 39
AIPTS/ILP VETERANS-
Exam Title :
2020 TE...
Email : yadavanurag075@gmail.com
Contact : 8882839768

b) जेरेमी�ब�थम�
c) जे�स��मल�
d) जॉन��टु अट� ��मल�
QUESTION 27.
1600 म��, ई�ट�इं�डया�कंपनी�ने�इं�ल�ड�क��शासक�, महारानी�ए�लजाबेथ�I से�एक�चाट� र��ा�त��कया�, �जससे�उसे�पूव��के�साथ��ापार�करने
का�पूरा�एका�धकार��मल�गया।�यह�एका�धकार�पूरी�तरह�से��न�न�ल�खत�म��से��कस����टश�अ�ध�नयम��ारा�वापस��लया�गया�था�?

a) 1773 का�चाट� र�ए�ट�


b) 1813 का�चाट� र�ए�ट�
c) 1833 का�चाट� र�ए�ट�
d) 1853 का�चाट� र�ए�ट�
QUESTION 28.
अवध�के�नवाब�सआदत�अली�खान�को�1801 म��अपने��े��का�आधा��ह�सा�कंपनी�को�दे ने�के��लए�मजबूर�होना�पड़ा��य��क:

a) सहायक�बल��( subsidiary forces) के��लए�भुगतान�करने�म��उनक���वफलता�, �जसे�कंपनी�संर�ण�के�उ�े �य�से�बनाए�रखने


वाली�थी।�
b) अवध�के�नवाब�पर�कुशासन�का�आरोप।�
c) अं�ेज��के���त�छल�और�कपट�का�आरोप।�
d) इनम��से�कोई�भी�नह�।�
QUESTION 29.
�न�न�ल�खत�म��से��कसने�तीन��गोलमेज�स�मेलन�म��भाग��लया�है:

1. मदन�मोहन�मालवीय�
2. बी�आर�अ�बेडकर�
3. महा�मा�गांधी�
4. तेज�बहा�र�स�ू�

सही�कूट�का�चयन�कर�:

a) केवल�1 और�3
b) केवल�2 और�3
c) केवल�2 और�4
d) केवल�1, 2 और�4
QUESTION 30.
���टश���ारा�राज�व�के��नय�मत��वाह�को�सु�न��त�करने�के��लए�1793 म���थायी�बंदोब�त�शु���कया�गया�था�, ले�कन�इससे�कई�सम�याएं
पैदा���।�इस�संदभ��म���न�न�ल�खत�कथन��पर��वचार�कर�:

1. ज़म�दार��ने�भू�म�के�उ�पादन�म��वृ���से�लाभ�के��लए�भू�म�सुधार�म��भारी��नवेश��कया।�
2. �कसान��ारा�लगान�का�भुगतान�न�करने�क����थ�त�म��उसे�जमीन�से�बेदखल�कर��दया�गया।�
3. ज़म�दार�जो�अं�ेज��को�राज�व�दे ने�म��असफल�रहे�, उ�ह�ने�अपनी�जम�दारी�अ�धकार��को�खो��दया।�

ऊपर��दए�गए�कथन��म��से�कौन�सा�सही��प�से��थायी�बंदोब�त��ारा�बनाई�गई�सम�या��को�उजागर�करता�है�?

a) केवल�2
b) केवल�2 और�3
c) केवल�1 और�3

IASbaba
Web: http://ilp.iasbaba.com/
Email: ilp@iasbaba.com
Page 40
AIPTS/ILP VETERANS-
Exam Title :
2020 TE...
Email : yadavanurag075@gmail.com
Contact : 8882839768

d) केवल�1, 2 और�3
QUESTION 31.
�न�न�ल�खत�म��से�कौन�सा�कथन����टश�भारत�के�तहत�नील�क��खेती�क��मु�य��णा�लय��के�बारे�म��सही�है�/ ह��?

1. ‘ �नज�’ (nij) �णाली�के�तहत�, �लांटर�ने�उन�जमीन��म��नील�का�उ�पादन��कया��जसे�वे�सीधे��नयं��त�करते�थे।�


2. �लांटर��ने�एक�अनुबंध�पर�ह�ता�र��कया�, एक�समझौते�(स�ा) , �जसम��रैयत��के�साथ�‘ रयो�त�’ (ryoti) �णाली�के�तहत�नील�क�
खेती�क��जाती�थी।�

सही�कूट�का�चयन�कर�:

a) केवल�1
b) केवल�2
c) 1 और�2 दोन��
d) न�तो�1 और�न�ही�2
QUESTION 32.
नील��व�ोह�( 1859-61) के�संदभ��म���न�न�ल�खत�कथन��पर��वचार�कर�:

1. �कसान��को��तनक�ठया��णाली�के�तहत�कुल�भू�म�के�3/20 �ह�से�पर�नील�उगाने�के��लए�मजबूर��कया�गया�था।�
2. नील�आयोग�, सरकार�ने�नील�उ�पादन�क���णाली�क��जांच�के��लए��था�पत��कया�, �जसने�नील�का�तकार��के�साथ�जबरद�ती�के
तरीक��का�उपयोग�करने�के��लए��लांटस��क��आलोचना�क�।�
3. नील��व�ोह�के�बाद��लांटस��ने�अपने�नील�के�प�रचालन�को��बहार�से�बंगाल��थानांत�रत�कर��दया�

ऊपर��दए�गए�कथन��म��से�कौन�सा�गलत�है�/ ह��?

a) केवल�1
b) केवल�2 और�3
c) केवल�1 और�3
d) 1, 2 और�3
QUESTION 33.
1857 के��व�ोह�के�संदभ��म���न�न�ल�खत�यु�म��पर��वचार�कर�:

नेता��े��

1. रानी�अवंतीबाई�लोधी�कानपुर�
2. अहम��ला�शाह�फैजाबाद�
3. बेगम�हजरत�महल�लखनऊ�

ऊपर�द��गई�कौन�सी�जोड़ी�गलत�है�/ ह��?

a) केवल�1
b) केवल�2 और�3
c) केवल�1 और�3
d) 1, 2 और�3
QUESTION 34.
नई��द�ली�और�इसक��इमारत��को��डजाइन�करने�के��लए��न�न�ल�खत�म��से��कस�वा�तुकार�को�बुलाया�गया�था�?

a) एडवड��लु�टयन�

IASbaba
Web: http://ilp.iasbaba.com/
Email: ilp@iasbaba.com
Page 41
AIPTS/ILP VETERANS-
Exam Title :
2020 TE...
Email : yadavanurag075@gmail.com
Contact : 8882839768

b) हरबट� �बेकर�
c) रॉबट� �टोर�रसेल�
d) (a) और�( दोन���वक�प�
QUESTION 35.
�न�न�ल�खत�म��से�कौन�' प�रवत�क�नह��' (no changer) था�?

a) सरदार�पटे ल�
b) डॉ. राज����साद�
c) मोतीलाल�नेह��
d) (a) और�( दोन���वक�प�
QUESTION 36.
भारतीय�इ�तहास�के�संदभ��म��‘ �च�ट्ज़�’, ‘ कोसैस�’, ‘ बंद�ा�’ (chintz, cossaes, bandanna) जैसे�श�द��न�न�ल�खत�से�संबं�धत�ह�:

a) मु��त�सूती�व����के��कार�
b) वा�तु�संरचना��के��कार�
c) महीन�रेशमी�व���के��कार�
d) यूरोपीय�लोग��क��मदद�करने�वाले�भारतीय��ापार�एज�ट��के��कार।�
QUESTION 37.
भारत�म���श�ा�पर�गांधी�के����कोण�के�संदभ��म���न�न�ल�खत�कथन��पर��वचार�कर�:

1. उ�ह�ने�माना��क�औप�नवे�शक��श�ा�ने�भारतीय��के�मन�म��हीनता�क��भावना�पैदा�क�।�
2. उ�ह�ने��ढ़ता�से�महसूस��कया��क�भारतीय�भाषा��को�ही��श�ण�का�मा�यम�होना�चा�हए।�
3. उ�ह�ने��श�ा�म���ावसा�यक���श�ण�के�मह�व�पर�जोर��दया।�

ऊपर��दए�गए�कथन��म��से�कौन�सा�सही�है�/ ह��?

a) केवल�1 और�2
b) केवल�1 और�3
c) केवल�2 और�3
d) 1, 2 और�3
QUESTION 38.
ई.वी. रामा�वामी�नायकर�के�बारे�म���न�न�ल�खत�कथन��पर��वचार�कर�:

1. वह�भारतीय�रा�ीय�कां�ेस�( INC) के�सद�य�थे�


2. उ�ह�ने�ज��टस�पाट��क���थापना�क�।�
3. उ�ह�ने�साइमन�कमीशन�का��वागत��कया�तथा�नमक�स�या�ह�का��वरोध��कया�

ऊपर��दए�गए�कथन��म��से�कौन�सा�सही�है�/ ह��?

a) केवल�1
b) केवल�1 और�3
c) केवल�2
d) 2 और�3
QUESTION 39.

IASbaba
Web: http://ilp.iasbaba.com/
Email: ilp@iasbaba.com
Page 42
AIPTS/ILP VETERANS-
Exam Title :
2020 TE...
Email : yadavanurag075@gmail.com
Contact : 8882839768

�न�न�ल�खत�म��से�कौन-सी�कां�ेस�क���थापना�के�शु�आती�वष��म��कां�ेस�क��मांग��थ��?

1. �वधान�प�रषद��को�अ�धक���त�न�ध�बनाया�जाना�चा�हए�, तथा�अ�धक�श����दान��कया�जाना�चा�हए�, और�उन��ांत��म��आरंभ


�कया�जाना�चा�हए�, जहां�कोई�मौजूद�नह��थी।�
2. �सफ��लंदन�म��ही�नह��, ब��क�भारत�म��भी��स�वल�सेवा�क��परी�ाएं�आयो�जत�करना।�
3. �यायपा�लका�को�काय�पा�लका�से�पृथक�करना�

सही�कूट�का�चयन�कर�:

a) केवल�1 और�2
b) केवल�1 और�3
c) केवल�2 और�3
d) 1, 2 और�3
QUESTION 40.
चा�स��वुड्स�एजुकेशन��ड�पैच�, 1854 के�बारे�म���न�न�ल�खत�कथन��पर��वचार�कर�:

1. इसने�भारत�के��लए�‘ डाउनवड���न�पंदन��स�ांत�’ (downward filtration theory) क��अं�ेजी��श�ा�नी�त�क���सफा�रश


क�।�
2. 1857 म��लंदन��व��व�ालय�के�मॉडल�पर�कलक�ा�, बॉ�बे�और�म�ास�म��तीन��व��व�ालय��के��नमा�ण�के�मा�यम�से�उ�च��श�ा�को
बढ़ावा��दया�गया�था।�
3. इसने�म�हला�और��ावसा�यक��श�ा�और��श�क��के���श�ण�पर�जोर��दया।�

ऊपर��दए�गए�कथन��म��से�कौन�सा�सही�है�/ ह��?

a) केवल�1 और�2
b) केवल�1 और�3
c) केवल�2 और�3
d) 1, 2 और�3
QUESTION 41.
�न�न�ल�खत�म��से�कौन�सा�कथन�उ�ीसव��शता�द��के�सामा�जक�और�धा�म�क�सुधार�आंदोलन��क���वशेष��वशेषता��म��से�एक�नह��है�?

a) आंदोलन�एक�संक�ण��सामा�जक��थान�तक�ही�सी�मत�थे।�
b) अ�धकांश�सुधार�आंदोलन��क��अगुवाई�तीन�उ�च�जा�तय��, �ा�ण�, काय�थ�और�बै��से��ई।�
c) ऊपर�से�सुधार�लागू�करने�के��लए�उ�ह�ने�कानून�पर�अ�धक�भरोसा��कया।�
d) उ�ीसव��सद��के�शु�आती�दौर�के�सुधारक��ने�औप�नवे�शक�शासन�के�क�र��वरोध�का��दश�न��कया�
QUESTION 42.
�क�ूर��व�ोह�( 1824) �कससे�संबं�धत�है:

a) रानी�चे��मा�
b) रानी�अवंतीबाई�
c) रानी��गा�वती�
d) रानी����मा�
QUESTION 43.
1875 म��द�कन�दं गे�के�बारे�म���न�न�ल�खत�कथन��पर��वचार�कर�:

1. �कसान��ने��वशेष��प�से�सा�कार��के�क�जे�म��अपने�ऋण��से��नपटने�के��लए�बांड�, फरमान�और�अ�य�संबं�धत�द�तावेज��को�ल��त
�कया।�

IASbaba
Web: http://ilp.iasbaba.com/
Email: ilp@iasbaba.com
Page 43
AIPTS/ILP VETERANS-
Exam Title :
2020 TE...
Email : yadavanurag075@gmail.com
Contact : 8882839768

2. �कसान��ने�उन��कसान��और�बालुटेदार��के��खलाफ�सामा�जक���तबंध�लगाए�जो�सा�कार��के�ब�ह�कार�म��शा�मल�नह��ह�गे।�
3. महा�मा��यो�तराव�फुले�के�स�यशोधक�समाज�ने��कसान��के�कारण�का�समथ�न��कया।�

ऊपर��दए�गए�कथन��म��से�कौन�सा�सही�है�/ ह��?

a) केवल�1 और�2
b) केवल�1 और�3
c) केवल�2 और�3
d) 1, 2 और�3
QUESTION 44.
�न�न�ल�खत�म��से�कौन�सा�नेता�मु�य��प�से�अ�खल�भारतीय�रा�य��के�जन�स�मेलन�( AISPC) के�आयोजन�के��लए�उ�रदायी�थे�?

1. बलवंतराय�मेहता�
2. मा�ण�यलाल�कोठारी�
3. जी.आर. अ�यंकर�

सही�कूट�का�चयन�कर�:

a) केवल�1
b) केवल�2
c) केवल�1 और�2
d) 1, 2 और�3
QUESTION 45.
1927 म��ग�ठत�फेडरेशन�ऑफ�इं�डयन�चै�बस��ऑफ�कॉमस��एंड�इंड����(�फ�क�) के�बारे�म���न�न�ल�खत�कथन��पर��वचार�कर�:

1. �फ�क��क��भू�मका�को��ापार�, वा�ण�य�और�उ�ोग�के�रा�ीय�संर�क�के��प�म��दे खा�गया�


2. यह�आम�तौर�पर�संघष��के�सभी��प�- संवैधा�नक�, जन�आंदोलन�, �ां�तकारी�रणनी�त�आ�द�को�पसंद�करता�था।�
3. इसने�अपने�सद�य��को�गोलमेज�स�मेलन�(आरट�सी) म��भाग�लेने�के��लए��ो�सा�हत��कया�

ऊपर��दए�गए�कथन��म��से�कौन�सा�गलत�है�/ ह��?

a) केवल�1 और�2
b) केवल�1 और�3
c) केवल�2 और�3
d) 1, 2 और�3
QUESTION 46.
कां�ेस-�खलाफत��वराज�पाट��(�वराजवा�दय�) से��ह����के��हत��क��र�ा�के��लए�‘ ��त��यावा�दय��' (responsivists) के��प�म��जाना
जाने�वाला�समूह�था।��न�न�ल�खत�म��से�कौन�सा�नेता�' ��त��यावाद��' (responsivist) नह��है�?

a) लाजपत�राय�
b) मदन�मोहन�मालवीय�
c) एन�सी�केलकर�
d) लोकमा�य��तलक�
QUESTION 47.
�न�न�ल�खत�म��से�कौन�सा�कथन����स��मशन�के��ावधान��पर�कां�ेस�क��आप���को�सही�ढं ग�से�दशा�ता�है�?

1. पूण���वतं�ता�के�बजाय�डो�म�नयन��टे टस�का��ावधान।�

IASbaba
Web: http://ilp.iasbaba.com/
Email: ilp@iasbaba.com
Page 44
AIPTS/ILP VETERANS-
Exam Title :
2020 TE...
Email : yadavanurag075@gmail.com
Contact : 8882839768

2. सं�वधान�सभा�म���रयासत��का���त�न�ध�व�रा�य��के�लोग���ारा�नह��ब��क�शासक��के�नामांकन��ारा��कया�जाता�है।�
3. वह��ावधान�जो��कसी�भी��ांत�को�नए�सं�वधान�को��वीकार�करने�के��लए�तैयार�नह��था�, उसे���टे न�के�साथ�एक�अलग�समझौते�पर
ह�ता�र�करने�का�अ�धकार�होगा�

सही�कूट�का�चयन�कर�:

a) केवल�1 और�2
b) केवल�1 और�3
c) केवल�3
d) 1, 2 और�3
QUESTION 48.
आईएनए�जाँच�( INA trials) 1945 के�बारे�म���न�न�ल�खत�कथन�पर��वचार�कर�:

1. आईएनए�कै�दय��क��र�ा�भुलाभाई�दे साई�, तेज�बहा�र�स�ू�, के.वी. काटजू�, नेह��और�आसफ��ारा�क��गयी�थी�


2. नेह��आईएनए�के�पकड़े�गए�सै�नक��को�पथ���दे शभ��मानते�थे।�
3. मु��लम�लीग�, �ह���महासभा�और�भारतीय�क�यु�न�ट�पाट��ने�आईएनए�कारण�का�समथ�न�नह���कया।�

ऊपर��दए�गए�कथन��म��से�कौन�सा�सही�है�/ ह��?

a) केवल�1
b) केवल�1 और�2
c) केवल�2 और�3
d) 1, 2 और�3
QUESTION 49.
‘ अ�खल�भारतीय�अ�पृ�यता��वरोधी�लीग�, 1932 (All India Anti Untouchability League) �कसके��ारा��था�पत��कया�गया
था:

a) डॉ. बी.आर. अ�बेडकर�


b) एम�के�गांधी�
c) एम.सी. राजा�
d) मदन�मोहन�मालवीय�
QUESTION 50.
���टश�को�कां�ेस- मु��लम�लीग�अवरोध�( Congress- Muslim League logjam) से�बचाने�के��लए�, वेवेल�ने�" �ेकडाउन��लान�"
��ता�वत��कया�, इसका�मु�य���ताव�था:

a) उ�र-प��म�और�उ�र-पूव��के�मु��लम��ांत��म�����टश�सेना�और�अ�धका�रय��क��वापसी�तथा�शेष�दे श�को�कां�ेस�को�स�पना।�
b) उ�र-प��म�और�उ�र-पूव��म��जनमत�सं�ह�क��शु�आत�करना�, जब�क�शेष�दे श�को�कां�ेस�को�स�पना।�
c) भारत�से����टश��क��तेजी�से�वापसी�, इस��कार�समझौता�करने�के��लए�कां�ेस�और�मु��लम�लीग�पर�दबाव�डालना�
d) इनम��से�कोई�भी�नह�।�
QUESTION 51.
�ाचीन�भारतीय�महाका���के�संबंध�म��, ' जय�सं�हता�' और�' सतश�ी�सं�हता�' �कस���स��महाका��से�संब��ह��?

a) उप�नषद��
b) पुराण��
c) रामायण�
d) महाभारत�

IASbaba
Web: http://ilp.iasbaba.com/
Email: ilp@iasbaba.com
Page 45
AIPTS/ILP VETERANS-
Exam Title :
2020 TE...
Email : yadavanurag075@gmail.com
Contact : 8882839768

QUESTION 52.
उ�र-वै�दक�काल�के�सा�ह�य�के�संबंध�म��, �न�न�ल�खत�म��से�कौन�सा�यु�म�सही��प�से�सुमे�लत�है�?

1. �ौतसू�:: य��वे�दय��के��नमा�ण�के��लए�माप�के��व�भ��पहलु��से�संबं�धत�है।�
2. गृ�सू�:: ज�म�, नामकरण�सं�कार�, �ववाह�आ�द�से�जुड़े�घरेलू�अनु�ान��से�संबं�धत�ह��
3. धम��सू�:: आचार�सं�हता�से�संबं�धत�है।�
4. सु�वसू�:: तीन�उ�च�वण��से�संबं�धत�पदाथ��के�पु�ष�और�राजकुमार��के��लए�बड़े�साव�ज�नक�अनु�ान��से�संबं�धत�है।�

सही��वक�प�चुन�:

a) 1 और�4
b) 2 और�3
c) 2, 3 और�4
d) 1, 2, 3 और�4
QUESTION 53.
�न�न�ल�खत�म��से��कसे�संगम�युग�का��ोत�माना�जा�सकता�है�?

1. ��ै बो�और���लनी�के�काय��
2. अशोक�के��शलालेख�
3. हाथीगु�फा��शलालेख�
4. सीलोन�क��पु�तक�' महावंश�' और�' द�पवंश�'

सही��वक�प�चुन��

a) 2 और�3
b) 1 और�4
c) 2, 3 और�4
d) 1, 2, 3 और�4
QUESTION 54.
�न�न�ल�खत�म��से�कौन�सही��प�से�सुमे�लत�है।�

( लेखक) ( पु�तक)

1. बाणभ��हष�च�रत�
2. सं�याकर�नंद��रामच�रत�
3. �ब�हण��व�मंकादे वचा�रत�
4. क�हण�राजतरं�गणी�

सही��वक�प�चुन�:

a) 1 और�2
b) 2 और�3
c) 1, 2 और�4
d) 1, 2, 3 और�4
QUESTION 55.
�न�न�ल�खत�कथन��पर��वचार�कर�:

1. �े न��सांग�और�फा-�ान�दोन��चीनी�या�ी�बौ��थे।�

IASbaba
Web: http://ilp.iasbaba.com/
Email: ilp@iasbaba.com
Page 46
AIPTS/ILP VETERANS-
Exam Title :
2020 TE...
Email : yadavanurag075@gmail.com
Contact : 8882839768

2. फ़ा-�ान�ने�गु�त�काल�म��भारत�क��सामा�जक-आ�थ�क���थ�तय��का�वण�न��कया�जब�क��े न��सांग�हष��के�काल�म��इसी�तरह�के�वृ�ांत
��तुत�करते�ह�।�

उपरो��कथन��म��से�कौन�सा�सही�है�/ ह��?

a) केवल�1
b) केवल�2
c) 1 और�2
d) कोई�नह��
QUESTION 56.
�न�न�ल�खत�म��से�कौन�भारत�म��नवपाषाण�युग�के��थल�ह��?

1. म�क��
2. उ�नूर�( Utnur)
3. ��हगीरी�
4. �मजा�पुर�म��बेलन�घाट��
5. हड़�पा�और�मोहनजोदड़ो�

सही��वक�प�चुन�:

a) 1, 2, 4 और�5
b) 1, 2, 3 और�4
c) 1, 2 और�3
d) 1, 2, 3, 4 और�5
QUESTION 57.
भारत�म��ता�पाषाणकालीन�( Chalcolithic) चरण�के�संबंध�म���न�न�ल�खत�कथन��पर��वचार�कर�:

1. ता�पाषाणकालीन�लोग�पक���ई��ट��से�प�र�चत�थे�, जो�घर��के��नमा�ण�के��लए�उपयोग��कए�जाते�थे।�
2. ता�पाषाणकालीन�लोग��ने��व�भ���कार�के��म���के�बत�न��का�उपयोग��कया।�
3. ता�पाषाणकालीन�लोग��ने�कपास�के�साथ-साथ�गे�ं�और�चावल�का�उ�पादन��कया।�

नीचे��दए�गए�कूट�का�उपयोग�करके�सही�कथन�चुन�:

a) 2 और�3
b) केवल�1
c) 1, 2 और�3
d) कोई�नह��
QUESTION 58.
�स�धु�घाट��स�यता�के�लोग��क��धा�म�क��थाएं�अ��तीय�थी।��न�न�ल�खत�म��से�कौन�संभवतः�इस�स�यता�के�लोग���ारा�पूजा�जाता�था�?

1. पशुप�त�
2. मातृदेवी�
3. कूबड़�वाला�बैल�
4. �व�णु�
5. पीपल�का�पेड़�

सही��वक�प�चुन�:

IASbaba
Web: http://ilp.iasbaba.com/
Email: ilp@iasbaba.com
Page 47
AIPTS/ILP VETERANS-
Exam Title :
2020 TE...
Email : yadavanurag075@gmail.com
Contact : 8882839768

a) 1, 2 और�3
b) 1, 2, 3 और�4
c) 1, 2, 3 और�5
d) 1, 2, 3, 4 और�5
QUESTION 59.
हड़�पा�स�यता�के�दौरान�शहर��का��वकास�एक�उ�लेखनीय��वशेषता�थी।�हड़�पा�स�यता�के�दौरान��न�न�ल�खत�म��से��कस��थान�को�शहर�माना
जा�सकता�है�?

1. कालीबंगा�
2. बनावली�
3. रंगपुर�
4. लोथल�
5. सुरकोटड़ा�

सही��वक�प�चुन�:

a) 3 और�5
b) 1, 2 और�4
c) 1, 2, 3 और�4
d) 1, 2, 3, 4 और�5
QUESTION 60.
ऋ�वेद�म��' ग�व��' (Gavisthi) श�द�का��या�अथ��है�?

a) बढई�
b) आ�दवासी�मु�खया�
c) भू�म�क��इकाई�
d) यु��
QUESTION 61.
भारतीय�उप-महा��प�म��आय��के��वास�के�संबंध�म��, �न�न�ल�खत�कथन��पर��वचार�कर�:

1. आय��ने�भारतीय�उपमहा��प�म��कई�चरण��म���वास��कया।�
2. आय��जनजा�तय��ने�आपस�म��सौहाद� �बनाए�रखा�, �जससे�उ�ह��अ�य�गैर-आय��जनजा�तय��पर�शासन�करने�म��मदद��मली।�

नीचे��दए�गए��वक�प��का�उपयोग�करके�सही�कथन�चुन�:

a) केवल�1
b) केवल�2
c) 1 और�2 दोन��
d) न�तो�1 और�न�ही�2
QUESTION 62.
अशोक�के��शलालेख��व�भ���ल�पय��म���लखे�गए�थे।�अशोक�के��शलालेख��म���यु���ल�पय��म��से�कौन-सी��ल�पयाँ�थ��?

1. �ा�ी�
2. खरो�ी�
3. यूनानी�
4. आरमाइक�

IASbaba
Web: http://ilp.iasbaba.com/
Email: ilp@iasbaba.com
Page 48
AIPTS/ILP VETERANS-
Exam Title :
2020 TE...
Email : yadavanurag075@gmail.com
Contact : 8882839768

नीचे��दए�गए�कूट�का�उपयोग�करके�सही��वक�प�चुन�।�

a) 1 और�3
b) 1, 2 और�3
c) 1, 3 और�4
d) 1, 2, 3 और�4
QUESTION 63.
भारत�म���ह���दश�न�क��छह��णा�लय��म��से�कौन�सी��न�न�ल�खत�ह��?

1. सां�य�
2. �यो�तष�
3. �याय�
4. �ाकरण�
5. योग�
6. मीमांसा�

नीचे��दए�गए�कूट�का�उपयोग�करके�सही��वक�प�चुन�:

a) 1, 2, 3 और�6
b) 1, 3, 5 और�6
c) 1, 2, 4 और�5
d) 1, 2, 3, 4, 5 और�6
QUESTION 64.
5 व��और�6 व��शता�द��ईसा�पूव��के�दौरान�ईरा�नय��के�आ�मण�ने�उ�री�भारत�म��मह�वपूण���वकास�लाए।��न�न�ल�खत�म��से�कौन�सा
घटना�म�ईरानी�आ�मण�से�संबं�धत�है�?

1. खरो�ी��ल�प�का�आ�व�कार।�
2. इसने�मौय�कालीन�मू�त�कला�को��भा�वत��कया।�
3. इसने�भारत�पर��सकंदर�के�आ�मण�का�नेतृ�व��कया।�

सही�कथन�चुन��

a) 1 और�2
b) 1 और�3
c) 2 और�3
d) 1, 2 और�3
QUESTION 65.
उ�री�काले�पो�लश�वाले�मृदभांड�( NBP) चरण�के�संबंध�म���न�न�ल�खत�कथन��पर��वचार�कर�।�

1. एनबीपी�चरण�ने�भारत�म���सरे�शहरीकरण�क��शु�आत�को��च��त��कया।�
2. इसने�पहली�बार�धा��वक�मु�ा�क��शु�आत�दे खी।�

सही�कथन�चुन��

a) केवल�1
b) केवल�2
c) 1 और�2 दोन��

IASbaba
Web: http://ilp.iasbaba.com/
Email: ilp@iasbaba.com
Page 49
AIPTS/ILP VETERANS-
Exam Title :
2020 TE...
Email : yadavanurag075@gmail.com
Contact : 8882839768

d) न�तो�1 और�न�ही�2
QUESTION 66.
वै�दक�काल�म��, श�द�' �न�क�’ और�' शतमान�' का�उ�लेख��कसके�संदभ��म��है�

a) �स�के�
b) जनजा�त�
c) ��स���व�ान�
d) म�हला�पुजारी�
QUESTION 67.
मौय��सा�ा�य�क��आ�थ�क��णाली�के�संबंध�म��, �न�न�ल�खत�कथन��पर��वचार�कर�।�

1. दास��का�उपयोग�कृ�ष�काय��म���कया�जाता�था।�
2. मौय��ने�भंडारण�और�जमा�करने�क��तुलना�म��कर��के�आकलन�को�अ�धक�मह�व��दया।�

सही�कथन�चुन��

a) केवल�1
b) केवल�2
c) 1 और�2 दोन��
d) न�तो�1 और�न�ही�2
QUESTION 68.
मौय��सा�ा�य�के�बाद�भारत�पर�आ�मण�करने�वाले�तथा�बाद�म��बसने�वाले��वदे �शय��के�संबंध�म��, �न�न�ल�खत�कथन��पर��वचार�कर�:

1. कुषाण�शासक��ने��शव�, �व�णु�और�बु��क��पूजा�क�।�
2. सरकार�क��‘ ��प��णाली�’ कुषाण��और�शक�शासक���ारा���तुत��कया�गया�था।�
3. स�ा�का�क���करण�उनके�शासन�क���मुख��वशेषता�थी।�

उपरो��कथन�म��से�कौन�सा�सही�है�/ ह��?

a) 1 और�2
b) 2 और�3
c) केवल�2
d) 1, 2 और�3
QUESTION 69.
सातवाहन��क���शास�नक��णाली�का��ज���न�न�ल�खत�म��से��कसके�संदभ��म���कया�गया�है�?

श�द��ववरण�

1. अमा�य�( Amatya): एक��शास�नक�इकाई�के��मुख।�


2. अहार�( Ahara): �जले�क��इकाई।�
3. गौ��मक�( Gaulmika): �ामीण��े���म��एक�सै�य�रे�जम�ट�के��मुख।�
4. कटवास�( Katavas): सै�य��श�वर।�

नीचे��दए�गए�कूट�का�उपयोग�करके�सही��वक�प�चुन�।�

a) 1 और�2
b) 1 और�3

IASbaba
Web: http://ilp.iasbaba.com/
Email: ilp@iasbaba.com
Page 50
AIPTS/ILP VETERANS-
Exam Title :
2020 TE...
Email : yadavanurag075@gmail.com
Contact : 8882839768

c) केवल�1
d) 1, 2, 3 और�4
QUESTION 70.
‘ गाथास�तसती�’ पु�तक�का��ेय��कस�सातवाहन�राजा�को�है�?

a) नहपान�
b) गौतमीपु��सातकण��
c) हाल�
d) �समुक�
QUESTION 71.
भारत�म��महापाषाण�कालीन�चरण�या�महापाषाणीय�( Magaliths) के�संबंध�म���न�न�ल�खत�कथन��पर��वचार�कर�:

1. प�थर��के�बड़े�टु कड़��से��घरी��ई�महापाषणीय�क���थ�।�
2. महापाषाणीय�लोग��ने�एक�उ�त��कार�क��कृ�ष�का�अ�यास��कया।�
3. कुछ�महापाषाणीय�लोग��ने�ग���म��लाल��म���के�बत�न��से�बने�कलश��म��मृतक��के�कंकाल�दफन��कए।�

उपरो��कथन�म��से�कौन�सा�सही�है�/ ह��?

a) केवल�1
b) 1 और�3
c) 1 और�2
d) 1, 2 और�3
QUESTION 72.
भारत�म��गु�त�काल�के�सा�ह�य�के�संबंध�म��, �न�न�ल�खत�कथन��पर��वचार�कर�:

1. इस�अव�ध�के�दौरान��न�म�त�सभी�नाटक�हा�य�थे�तथा�उनम��कोई�भी��ासद��नह��थी।�
2. उ�च�और��न�न�वग��के�वण��समान�भाषा�नह��बोलते�ह�।�

उपरो��कथन��म��से�कौन�सा�सही�है�/ ह��?

a) केवल�1
b) केवल�2
c) 1 और�2 दोन��
d) न�तो�1 और�न�ही�2
QUESTION 73.
' र�नावली�', ' नागानंद�' और�' ��यद�श�का�' भारत�के��कस���स��राजा�के�सा�ह��यक�काय��ह��?

a) समु�गु�त�
b) च��गु�त���तीय�
c) क�न�क�
d) हष�वध�न�
QUESTION 74.
पां��सा�ा�य�के�संबंध�म���न�न�ल�खत�कथन��पर��वचार�कर�।�

1. अशोक�के��शलालेख��म��केवल�पां��सा�ा�य�के�बारे�म��उ�लेख�है�तथा�इसम��चोल�या�चेर�रा�य��का�कोई�उ�लेख�नह��है।�
2. पां��सा�ा�य�मातृस�ा�मक�था।�

IASbaba
Web: http://ilp.iasbaba.com/
Email: ilp@iasbaba.com
Page 51
AIPTS/ILP VETERANS-
Exam Title :
2020 TE...
Email : yadavanurag075@gmail.com
Contact : 8882839768

3. पां���क��अथ��व�था�ने�मोती�( pearls) म��मह�वपूण���ापार�दश�या�है।�

उपरो��कथन�म��से�कौन�सा�सही�है�/ ह��?

a) 1 और�3
b) 1 और�2
c) केवल�1
d) 2 और�3
QUESTION 75.
भारत�म��हष��के�शासन�के�संबंध�म���न�न�ल�खत�कथन��पर��वचार�कर�।�

1. हष��का��शासन�गु�त��क��तुलना�म��अ�धक�सामंती�और��वक���कृत�था।�
2. भारत�म��हष��के�शासनकाल�के�दौरान�नालंदा�महायान�बौ��धम��का�सबसे���स��क���था।�

उपरो��कथन�म��से�कौन�सा�सही�है�/ ह��?

a) केवल�1
b) केवल�2
c) 1 और�2 दोन��
d) न�तो�1 और�न�ही�2
QUESTION 76.
जैन�धम��म��, �न�न�ल�खत�म��से��कसे�" �न���थ�" (Nirgranthis) कहा�जाता�है�?

a) वह�जो�मं�दर��म��रहता�हो�
b) वह�जो�सभी�बंधन��से�मु��हो�
c) अनाथ��को�
d) वह�जो�स�यास�लेता�है�
QUESTION 77.
�न�न�ल�खत�कथन��पर��वचार�कर�:

1. ��ांड�के�उ�च�चरण�को�उ�सप�नी�( Utsarpani) कहा�जाता�है�जब�क���ांड�के��न�न�चरण�को�अ�सरपानी�( Avsarpani) क


हा�जाता�है।�
2. आ�मा�जी�वत�और��नज�व�त�व��दोन��म��मौजूद�है।�

उपरो��म��से�कौन�सा�कथन�जैन�दश�न�के�अनुसार�सही�है�/ ह��?

a) केवल�1
b) केवल�2
c) 1 और�2 दोन��
d) न�तो�1 और�न�ही�2
QUESTION 78.
�न�न�ल�खत�कथन��पर��वचार�कर�:

1. �दगंबर�सं�दाय�म��पु�ष��भ�ु�कपड़े�नह��पहनते�ह��, जब�क�म�हला��भ�ु��बना��सली�साद��सफेद�साड़ी�पहनती�ह�।�
2. �ेता�बर�5 �त��म��से�केवल�4 �त��का�पालन�करते�ह��, वे�" अप�र�ह�" का�पालन�नह��करते�ह��

उपरो��कथन��म��से�कौन�सा�सही�है�/ ह��?

IASbaba
Web: http://ilp.iasbaba.com/
Email: ilp@iasbaba.com
Page 52
AIPTS/ILP VETERANS-
Exam Title :
2020 TE...
Email : yadavanurag075@gmail.com
Contact : 8882839768

a) केवल�1
b) केवल�2
c) 1 और�2 दोन��
d) न�तो�1 और�न�ही�2
QUESTION 79.
�न�न�ल�खत�यु�म��पर��वचार�कर�:

बु��क��मु�ाएं�श�द���त�न�ध�व�करते�ह��

1. वरद�मु�ा�सभी�कामना��क��पू�त��, दान�का�भाव�
2. अभय�मु�ा�आ�ासन�, आशीवा�द�और�र�ा�
3. �वतक��मु�ा�बौ��क�तक��, चचा�।�

उपरो��म��से�कौन�सी�जोड़ी�सही�ढं ग�से�सुमे�लत�है�/ ह��?

a) केवल�2
b) केवल�1 और�2
c) केवल�2 और�3
d) 1, 2 और�3
QUESTION 80.
�न�न�ल�खत�कथन��पर��वचार�कर�:

1. बौ��धम��के�अनुसार�, आठ�महान�स�य�(आय��स�य) ह��


2. वसु�म��और�अ�घोष�क��अ�य�ता�म��पाट�लपु��म��चौथी�बौ��प�रषद�आयो�जत�क��गयी�थी�

उपरो��कथन��म��से�कौन�सा�सही�है�/ ह��?

a) केवल�1
b) केवल�2
c) 1 और�2 दोन��
d) न�तो�1 और�न�ही�2
QUESTION 81.
�न�न�ल�खत�कथन��पर��वचार�कर�:

1. �ेत��च�ीदार�लाल�बलुआ�प�थर�( White spotted Red sandstone) का�उपयोग�गंधार��कूल�कला�म���कया�जाता�है�जब�क


सफेद�संगमरमर�का�उपयोग�अमरावती��कूल�कला�म���कया�जाता�है�
2. मथुरा��कूल�कला�म��बु��के��च��अभयमु�ा�और�धम�च�प�रवत�न�मु�ा�म��दे खे�जा�सकते�ह��

उपरो��कथन��म��से�कौन�सा�गलत�है�/ ह��?

a) केवल�1
b) केवल�2
c) 1 और�2 दोन��
d) न�तो�1 और�न�ही�2
QUESTION 82.

IASbaba
Web: http://ilp.iasbaba.com/
Email: ilp@iasbaba.com
Page 53
AIPTS/ILP VETERANS-
Exam Title :
2020 TE...
Email : yadavanurag075@gmail.com
Contact : 8882839768

�न�न�ल�खत��थान��पर��वचार�कर�:

1. क�ल�ग�
2. काक�नाडा�
3. सुर�कारा�
4. �ग�रनगर�

उपयु����थान��म��से�अशोक��ारा�कहाँ��तंभ�लेख�का��नमा�ण��कया�गया�था�?

a) केवल�1 और�2
b) केवल�2 और�3
c) केवल�1, 2, और�3
d) 1, 2, 3 और�4
QUESTION 83.
�न�न�ल�खत�कथन��पर��वचार�कर�:

1. वध�मान�ने�एक�साल�के�पेड़�के�नीचे�सव��च��ान�(कैव�य) �ा�त��कया�
2. 498 ई.पू. �रझुपा�लका�नद��के�तट�पर����बी�ाम�नामक��थान�पर�
3. वध�मान�महावीर�के�दामाद�उनके�अं�तम��श�य�थे�

उपरो��कथन��म��से�कौन�सा�सही�है�/ ह��?

a) केवल�1
b) केवल�2
c) 1 और�2 दोन��
d) न�तो�1 और�न�ही�2
QUESTION 84.
�न�न�ल�खत�कथन��पर��वचार�कर�:

1. �वपरीत��ल�ग�को�नह��दे खना।�
2. �वपरीत��ल�ग�से�बात�नह��करना।�
3. �वपरीत��ल�ग�को�नह��छू ना।�
4. �वपरीत��ल�ग�के�बारे�म��महसूस�नह��करना।�

जैन�धम��के�पाँचव���स�ांत��के�अनुसार�उपरो��म��से�कौन�से�कथन�का�पालन�नह���कया�जाता�है�?

a) केवल�1 और�2
b) केवल�2 और�3
c) 1, 2, 3 और�4
d) कोई�नह��
QUESTION 85.
�न�न�ल�खत�पर��वचार�कर�:

1. स�यक��ान�
2. मो��क���ा��त�
3. स�यक�आचरण�( Right action)
4. सही�दश�न�( Right faith)

IASbaba
Web: http://ilp.iasbaba.com/
Email: ilp@iasbaba.com
Page 54
AIPTS/ILP VETERANS-
Exam Title :
2020 TE...
Email : yadavanurag075@gmail.com
Contact : 8882839768

उपरो��म��से�कौन�सा�कथन�जैन�धम��का���र�न�माना�जाता�है�?

a) केवल�1, 2 और�3
b) केवल�1, 3 और�4
c) केवल�2, 3 और�4
d) कोई�नह��
QUESTION 86.
�न�न�ल�खत�यु�म��पर��वचार�कर�:

बु��के�जीवन�काल�के�चरण��त�नध�व�श�द�

1. महा�भ�न�कण��आ�म�ान�( Enlightenment)
2. �नवा�ण�घर�छोड़ने�क����या�
3. धम��च��प�र�तन�ग�त�च��क���थापना�( Setting the wheel in motion)

उपरो��म��से�कौन�सी�जोड़ी�सही�ढं ग�से�सुमे�लत�है�/ ह��?

a) केवल�3
b) केवल�1 और�3
c) केवल�2 और�3
d) 1, 2 और�3
QUESTION 87.
राजा�चं�गु�त�मौय��के�वंश�का�वण�न�करने�वाले��वशाखद��के�सा�ह��यक�काय��म��से��न�न�ल�खत�कौन�सा�है�?

a) �व�मोव�शीयम्�
b) कुमारसंभवम�
c) ऋतुसंहार�
d) मु�ारा�स�
QUESTION 88.
दसव��और�तेरहव��शता�द��ई. के�बीच�भारत�आने�वाले�या�ी�थे�

a) अल- ब�नी�, �े न-�सांग�, और�मेग�थनीज़�


b) फा-�ान�, �े न-�सांग�और�मेग�थानीज़�
c) अल-ब�नी�, अल-मसुद��और�माक��पोलो�
d) फ़ा-�ान�, अल-ब�नी�और�अल-ब�नी�
QUESTION 89.
�न�न�ल�खत�कथन��पर��वचार�कर�:

1. रामानंद�पहले�भ���संत�थे�तथा�उ�र�भारत�के�भ���आंदोलन�के�सं�थापक�थे।�
2. वह�शंकराचाय��के��श�य�थे�

उपरो��कथन��म��से�कौन�सा�गलत�है�/ ह��?

a) केवल�1
b) केवल�2
c) 1 और�2 दोन��

IASbaba
Web: http://ilp.iasbaba.com/
Email: ilp@iasbaba.com
Page 55
AIPTS/ILP VETERANS-
Exam Title :
2020 TE...
Email : yadavanurag075@gmail.com
Contact : 8882839768

d) न�तो�1 और�न�ही�2
QUESTION 90.
�न�न�ल�खत�यु�म��पर��वचार�कर�:

लेखक�काय��

1. �व�णु�शमा��पंच�स�ां�तका�
2. वराह�म�हर�पंचतं��
3. वा��यायन�कामसू��

उपरो��म��से�कौन�सी�जोड़ी�सही�ढं ग�से�सुमे�लत�है�/ ह��?

a) केवल�1 और�3
b) केवल�2 और�3
c) 1, 2 और�3
d) केवल�3
QUESTION 91.
�न�न�ल�खत�कथन��पर��वचार�कर�:

1. बा-शरा�( Bashara) सूफ�वाद�के��सल�सल��म��से�एक�है�, �जसका�अथ��है�जो�इ�लामी�कानून��का�पालन�करते�ह�।�


2. सुहरावद��बाशरा�के�अंतग�त�बारह��सल�सल��म��से�एक�ह��
3. सूफ��श�द�म��, उस��का�अथ��अनुयायी�है�

उपरो��कथन��म��से�कौन�सा�सही�है�/ ह��?

a) केवल�1 और�3
b) केवल�2 और�3
c) केवल�1
d) केवल�3
QUESTION 92.
�न�न�ल�खत�कथन��पर��वचार�कर�:

1. ��प�ीय�संघष��म��पूव��पाल�से�संबं�धत�है�, द�कन�म��रा�कूट�, उ�री��े��म���द�ली�स�तनत�थी�


2. �व�म�शला��व��व�ालय�क���थापना�धम�पाल�ने�क��थी।�

उपरो��कथन��म��से�कौन�सा�सही�है�/ ह��?

a) केवल�1
b) केवल�2
c) 1 और�2 दोन��
d) न�तो�1 और�न�ही�2
QUESTION 93.
�न�न�म��से�कौन�सी�पु�तक�तुगलक�वंश�का��ववरण�दे ती�है�?

a) तुगलक-नामा�
b) फतवा-ए-जहांदारी�
c) �कताब-फ�-तहक़�क़�

IASbaba
Web: http://ilp.iasbaba.com/
Email: ilp@iasbaba.com
Page 56
AIPTS/ILP VETERANS-
Exam Title :
2020 TE...
Email : yadavanurag075@gmail.com
Contact : 8882839768

d) कमस�( Qamas)
QUESTION 94.
म�लक�अहमद��न�न�ल�खत�म��से��कस�रा�य�के�सं�थापक�थे�?

a) इमाद�शाही�राजवंश�
b) आ�दल�शाही�वंश�
c) �नज़ाम�शाही�राजवंश�
d) बरीद�शाही�वंश�
QUESTION 95.
�न�न�ल�खत�पर��वचार�कर�:

1. गुलाम�वंश�
2. सै�यद�वंश�
3. तुगलक�वंश�
4. लोधी�वंश�
5. �खलजी�वंश�

�न�न�ल�खत�म��से�कौन��द�ली�स�तनत�का�सही�कालानु��मक��म�है�?

a) 1, 2,3 4 और�5
b) 1, 5, 3, 2 और�4
c) 1, 2, 5, 3 और�4
d) 1, 3, 2, 5 और�4
QUESTION 96.
�न�न�ल�खत�कथन��पर��वचार�कर�:

1. नीले�पानी�क��नी�त�को��ां�स�को�डी�अ�मीडा�ने���तुत��कया�था�
2. नीनो�दा�कु�हा��ारा�भारत�म��त�बाकू�और�काजू�को���तुत��कया�गया�था�

उपरो��कथन��म��से�कौन�सा�सही�है�/ ह��?

a) केवल�1
b) केवल�2
c) 1 और�2 दोन��
d) न�तो�1 और�न�ही�2
QUESTION 97.
�न�न�ल�खत�कथन��पर��वचार�कर�:

1. अ���ला�खान�और��सैन�अली�मुगल�काल�के�दौरान�राजा��नमा�ता�के��प�म��जाने�जाते�ह��
2. जहांदर�शाह�ने�इज़ारा�( Izara) को�समा�त�कर��दया�था�

उपरो��कथन��म��से�कौन�सा�गलत�है�/ ह��?

a) केवल�1
b) केवल�2
c) 1 और�2 दोन��
d) न�तो�1 और�न�ही�2

IASbaba
Web: http://ilp.iasbaba.com/
Email: ilp@iasbaba.com
Page 57
AIPTS/ILP VETERANS-
Exam Title :
2020 TE...
Email : yadavanurag075@gmail.com
Contact : 8882839768

QUESTION 98.
�न�न�ल�खत�कथन��पर��वचार�कर�:

1. डच��ने�1623 म��मसूलीप�नम�म��अपनी�पहली�फै�����था�पत��कया�जब�क�अं�ेजी�ने�1605 म��मसूलीप�नम�म��अपनी�फै����क�


�थापना�क��
2. 1632 म��गोलकुंडा�के�सु�तान��ारा�अं�ेजी�कंपनी�को��व�ण�म�फरमान�जारी��कया�गया�

उपरो��कथन��म��से�कौन�सा�सही�है�/ ह��?

a) केवल�1
b) केवल�2
c) 1 और�2 दोन��
d) न�तो�1 और�न�ही�2
QUESTION 99.
�न�न�ल�खत�कथन��पर��वचार�कर�:

1. �थम�बमा��यु��का�अंत�यांडाबो�क��सं�ध�के�साथ��आ�
2. तृतीय�आं�ल-मैसूर�यु��के�दौरान�हैदर�अली�क��मृ�यु�हो�गई�, �जसके�कारण�ट�पू�ने�आगे�यु��को�नेतृ�व��दया�

उपरो��कथन��म��से�कौन�सा�सही�है�/ ह��?

a) केवल�1
b) केवल�2
c) 1 और�2 दोन��
d) न�तो�1 और�न�ही�2
QUESTION 100.
��तीय�आं�ल-�सख�यु��लड़ने�वाला�गवन�र�जनरल�कौन�था�?

a) लॉड��डलहौजी�
b) लॉड��कान�वा�लस�
c) लॉड��वेलेजली�
d) लॉड��कै�न�ग�

IASbaba
Web: http://ilp.iasbaba.com/
Email: ilp@iasbaba.com
Page 58
AIPTS/ILP VETERANS-
Exam Title :
2020 TE...
Email : yadavanurag075@gmail.com
Contact : 8882839768

Review in Hindi
QUESTION 1.
भरत�इ�तहास�समशोधक�मंडल�( BISM) क���थापना�1910 म���कसके��ारा��ई�थी:

a) �व�नाथ�काशीनाथ�राजवाड़े�
b) रामकृ�ण�गोपाल�भंडारकर�
c) राज���लाल��म�ा�
d) के.ए. नीलकंठ�शा��ी�
Correct Answer: A
Your Answer:
Explanation

Solution (a)

Basic Information:

• भारत�इ�तहास�समाशोधक�मंडल�( BISM) के�सं�थापक��व�नाथ�काशीनाथ�राजवाडे�थे।�


• उ�ह�ने�1910 म��के. सी. मह�दले�( K C Mehendale) के�सहयोग�से�पुणे�म��इस�सं�था�क���थापना�क�।�
• �व�नाथ�काशीनाथ�राजवाड़े�( 24 जून�1863 - 31 �दसंबर�1926), जो�इ�तशाचाय��राजवाडे�के�नाम�से���स��ह��, महारा��के
इ�तहासकार�, �व�ान�, लेखक�, समालोचक�और�लेखक�थे।�
• 1895 म��उ�ह�ने�एक�मराठ��प��का�शु��क��, �जसका�नाम�भाषांतार�( ‘ अनुवाद�’) है�, �जसके�मा�यम�से�वे�प��मी�इ�तहासकार�
तथा��लेटो�, अर�तू�और�एडवड���ग�बन�जैसे��व�ान��और�मराठ��म��शंकराचाय��जैसे�भारतीय��व�ान��के�काय��सामने�लाये।�
• 1926 म��उनक��आक��मक�मृ�यु�के�बाद�, धुले�म��' राजवाडे�संशोधक�मंडल�' क���थापना��ई�
• उनक���व�यात�कृ�त: मरा�ान�या�इ�तशाची�साधने�(मराठ��का�इ�तहास) , राधा�माधव��वलास�चंपू�(शाहजी�क��जीवनी) , भारतीय
�ववाह�सं�थान�इ�तहस�(भारतीय�वैवा�हक�जीवन�का�इ�तहास) , ऐ�तहा�सक���तावना�(ऐ�तहा�सक��थान)
• BISM ने�अपनी�रजत�जयंती�मनाने�के��लए�1935 म��एक�अ�खल�भारतीय�कां�ेस�का�आयोजन��कया।�इस�घटना�के
प�रणाम�व�प�भारतीय�इ�तहास�कां�ेस�( IHC) क��शु�आत��ई।�
• भंडारकर�ओ�रएंटल��रसच��इं�ट��ूट�क���थापना�1917 म��आर�जी�भंडारकर�ने�क��थी।�

QUESTION 2.
भीमा-कोरेगांव�क��लड़ाई��न�न�ल�खत�म��से��कस�यु��का��ह�सा�थी�?

a) चौथा�आं�ल-मैसूर�यु��
b) �सरा�आं�ल-मराठा�यु��
c) तीसरा�आं�ल-मराठा�यु��
d) इनम��से�कोई�भी�नह��
Correct Answer: C
Your Answer:
Explanation

Solution (c)

Basic Information:

• 1818 के�भीमा-कोरेगांव�लड़ाई�क��202 व��वष�गांठ�1 जनवरी�, 2020 को�मनाई�गई�थी।�


• तृतीय�आं�ल-मराठा�यु��क��अं�तम�लड़ाई�1 जनवरी�1818 को�भीमा�नद��के�तट�पर�कोरेगाँव�म��लड़ी�गई�थी�, इस�लए�भीमा
कोरेगाँव�लड़ाई�का�नाम��दया�गया�
• ���टश�ई�ट�इं�डया�कंपनी�और�मराठा�सा�ा�य�के�बीच�तीन�यु���को�महान�मराठा�यु���या�आं�ल�-मराठा�यु���के��प�म��जाना�जाता
है।�

IASbaba
Web: http://ilp.iasbaba.com/ Score:
Email: ilp@iasbaba.com 3.00 / 200
Page 109
AIPTS/ILP VETERANS-
Exam Title :
2020 TE...
Email : yadavanurag075@gmail.com
Contact : 8882839768

• तृतीय�आं�ल-मराठा�यु��( 1817-1818) अं�ेज��और��प�डा�रय��के�बीच�संघष��के�कारण��आ�था।�अं�ेज��को�संदेह�था��क�मराठा


�प�डा�रय��को�सहायता��दान�कर�रहे�थे�, जो��क�मराठ��के��लए�लड़ने�वाले�भाड़े�के�सै�नक�थे।�
• पेशवा�ने�पुणे�पर�आ�मण�करने�का��वचार��कया�, ले�कन����टश�ने��श�र�से�सै�नक��को�भेज��दया�, �जसके�प�रणाम�व�प�1 जनव
री�1818 को�भीमा-कोरेगांव�क��लड़ाई��ई।�
• कंपनी�बल���ारा��नणा�यक��प�से�लड़ाई�जीती�गई�थी।�भारतीय�मूल�के�मृत�कंपनी�सै�नक��म��22 महार�, 16 मराठा�, 8 राजपूत�, 2
मु��लम�और�2 य�द��शा�मल�थे।�
• 1 जनवरी�1927 को�बी.आर. अंबेडकर�ने�भीमा-कोरेगांव�के�आधार�पर�खड़ी��मारक�ओ�ब�ल�क�का�दौरा��कया�, �जसम��लगभग�दो
दज�न�महार�सै�नक��स�हत�मृतक��के�नाम�ह�।�

(Source: https://www.thehindu.com/thread/politics-and-policy/what-the-battle-of-bhima-
koregaon-meant/article22378086.ece )

QUESTION 3.
�न�न�ल�खत�कथन��पर��वचार�कर�:

1. वह�एक��श�क�बनने�वाली�पहली�भारतीय�म�हला�थ�।�
2. उ�ह�ने�क�या��ूण�ह�या�को�रोकने�तथा�गृह�, बालह�या���तबंधक�गृह��था�पत�करने�क���दशा�म��भी�काम��कया।�
3. उसने�बावन�काशी�सुबोध�र�नाकर��लखी।�

उपयु���कथन��न�न�ल�खत�म��से��कस�समाज�सुधारक�से�जुड़े�ह�:

a) फा�तमा�शेख�
b) पं�डत�रमाबाई�
c) रमाबाई�रानाडे�
d) सा�व�ीबाई�फुले�
Correct Answer: D
Your Answer:
Explanation

Solution (d)

Basic Information:

• सा�व�ीबाई�फुले�का�ज�म�3 जनवरी�1831 को�महारा��के�सतारा��जले�के�नायगांव�म���आ�था।�वह�ल�मी�और�खंडोजी�नीवशे


पा�टल�क��सबसे�बड़ी�बेट��थी�, दोन��माली�समुदाय�के�थे�, जो�अब�एक�अ�य��पछड़ी�जा�त�(ओबीसी) है।�9 साल�क��उ��म��, उसक�
शाद��13 वष�य��यो�तराव�फुले�से��ई�थी।�
• 21 साल�क��उ��म���यो�तबा�और�17 साल�क��सा�व�ी�ने�1848 म��म�हला��के��लए�एक��कूल�खोला।�यह�भारतीय���ारा�शु�
�कया�गया�म�हला��के��लए�दे श�का�पहला��कूल�था।�
• सा�व�ीबाई�फुले�को��यो�तबा�ने�उनके�घर�पर�पढ़ाया�था।�बाद�म��, उ�ह�ने�अहमदनगर�म��एक�अमे�रक���मशनरी��ारा�संचा�लत�सं�थान
म��तथा�पुणे�के�नॉम�ल��कूल�म��एक��श�क���श�ण�पा��म��लया।�
• फा�तमा�शेख�सा�व�ीबाई�के�साथ�नॉम�ल��कूल�गई�थी�और�दोन��ने�साथ�म���नातक��कया।�फा�तमा�और�सा�व�ीबाई�ने�1849 म��पुणे
म��उ�मान�शेख�के�घर�म��एक��कूल�खोला।�
• 1850 के�दशक�म��, फुले�दं प�त�ने�दो�शै��क���ट�- द�ने�टव�फ�मेल��कूल�, पुणे�और�महार��क���श�ा�को�बढ़ावा�दे ने�के��लए
सोसाइट��, म��स�एंड�एटसेटेरस�( Mangs and Etceteras)- क��शु�आत�क��, �जसके�तहत�उनके�कई��कूल�थे।�
• सा�व�ीबाई�एक�उ��लेखक�और�कव�य�ी�भी�थ�।�उ�ह�ने�1854 म��का�ा�फुले�और�1892 म��बावन�काशी�सुबोध�र�नाकर��का�शत
�कया।�
• 1852 म��, एक�क�र�म�हलावाद��, सा�व�ीबाई�ने�म�हला��के�अ�धकार��के�बारे�म��जाग�कता�बढ़ाने�के��लए�म�हला�सेवा�मंडल�क�
शु�आत�क�।�
• उ�ह�ने�अपने�घर�म���शशु�ह�या�क��रोकथाम�के��लए�आवासीय�सु�वधा�भी�आरंभ��कया�, एक�ऐसी�जगह�जहाँ��ा�ण��वधवाएँ�अपने
ब�च��को�सुर��त��प�से�ज�म�दे �सकती�थ�।�
• उ�ह�ने�पहले�स�यशोधक��ववाह- दहेज�र�हत�, �ा�ण�पुजा�रय��या��ा�णवाद��अनु�ान��के��बना��ववाह�क��शु�आत�क�।�

IASbaba
Web: http://ilp.iasbaba.com/ Score:
Email: ilp@iasbaba.com 3.00 / 200
Page 110
AIPTS/ILP VETERANS-
Exam Title :
2020 TE...
Email : yadavanurag075@gmail.com
Contact : 8882839768

• सा�व�ीबाई�ने�अपने�प�त�के�अं�तम�सं�कार�क��अगुवाई�करते�समय�एक�और�वज�ना�तोड़ी।�

(Source: https://www.livemint.com/Leisure/DmR1fQSnVD62p4D3eyq9mO/The-life-and-times-of-
Savitribai-Phule.html )

QUESTION 4.
�न�न�ल�खत�कथन��पर��वचार�कर�:

1. उ�ह�ने�1899 के�कलक�ा�नगर�संशोधन�अ�ध�नयम�के�मा�यम�से�कलक�ा��नगम�के��नवा��चत���त�न�धय��क��सं�या�म��कमी�क�।�
2. 1904 के�भारतीय��व��व�ालय��अ�ध�नयम�के�मा�यम�से�उ�ह�ने�भारत�म���व��व�ालय��पर�सरकारी��नयं�ण�बढ़ा��दया।�
3. 1904 के�भारतीय�आ�धका�रक�गोपनीयता�संशोधन�अ�ध�नयम�के�तहत�उ�ह�ने��ेस�क���वतं�ता�को���तबं�धत�कर��दया।�

�न�न�ल�खत�म��से�कौन�सा�गवन�र�जनरल�उपरो��कथन��से�संब��है�?

a) लॉड��डलहौजी�
b) लॉड��कज�न�
c) लॉड���लटन�
d) लॉड���म�टो�
Correct Answer: B
Your Answer:
Explanation

Solution (b)

Basic Information:

लॉड��कज�न�( 1899-1905) ने�कई�अलोक��य��वधायी�और��शास�नक�उपाय��क��शु�आत�क��, �जससे��श��त�भारतीय��क�


संवेदनशीलता��भा�वत��ई।�

• 1899 के�कलक�ा�नगर�संशोधन�अ�ध�नयम�के�मा�यम�से�कलक�ा��नगम�के�पुनग�ठन�ने�इसम���नवा��चत���त�न�धय��क��सं�या�को
कम�कर��दया�;
• 1904 के�भारतीय��व��व�ालय�अ�ध�नयम�ने�कलक�ा��व��व�ालय�पर�अ�य�धक�सरकारी��नयं�ण�बढ़ा��दया�;
• 1904 के�भारतीय�आ�धका�रक�गोपनीयता�संशोधन�अ�ध�नयम�ने��ेस�क���वतं�ता�को���तबं�धत�कर��दया।�
• �ृंखला�म��अं�तम�1905 म��बंगाल�का��वभाजन�था�, �जसे�क�थत��प�से�कां�ेस�को��नयं��त�करने�वाले�बंगाली�रा�वा�दय��को
कमजोर�करने�के��लए��व�न�म�त��कया�गया�था।�

उनक��कुछ�लोक��य�पहल:

• उ�ह�ने�पूसा�(�बहार�- बंगाल��ेसीड�सी) म��कृ�ष�अनुसंधान�सं�थान�क���थापना�क��


• उ�ह�ने�1902 म��सर�एं�यू��ेज़र�क��अ�य�ता�म��एक�पु�लस�आयोग�क���थापना�क�।�
• उ�ह�ने��ाचीन��मारक�अ�ध�नयम�, 1904 नामक�एक�कानून�पा�रत��कया�

(Source: “From Plassey to Partition: A History of Modern India”)

QUESTION 5.
मदन�मोहन�मालवीय�के�संदभ��म���न�न�ल�खत�कथन��पर��वचार�कर�:

1. उ�ह�ने�बनारस��ह����व��व�ालय�क���थापना�म��योगदान��दया।�
2. उ�ह�ने�स�वनय�अव�ा�आंदोलन�म��भाग�नह���लया�
3. उ�ह��मरणोपरांत�दे श�के�सव��च�नाग�रक�स�मान�भारत�र�न�से�स�मा�नत��कया�गया�था।�

ऊपर��दए�गए�कथन��म��से�कौन�सा�सही�है�/ ह��?

IASbaba
Web: http://ilp.iasbaba.com/ Score:
Email: ilp@iasbaba.com 3.00 / 200
Page 111
AIPTS/ILP VETERANS-
Exam Title :
2020 TE...
Email : yadavanurag075@gmail.com
Contact : 8882839768

a) केवल�1 और�2
b) केवल�2 और�3
c) केवल�1 और�3
d) 1, 2 और�3
Correct Answer: C
Your Answer:
Explanation

Solution (c)

Basic Information:

• 25 �दसंबर�को�मदन�मोहन�मालवीय�, ��स��भारतीय��श�ा�वद्�और��वतं�ता�सेनानी��ज�ह��' महामना�' भी�कहा�जाता�है�, क�


जयंती�है।�
• वह��वतं�ता�सं�ाम�के�नेतृ�वकता���म��से�एक�थे�- कां�ेस�म��नरमपं�थय��और�उ�वा�दय��के�बीच�एक�सेतु�के��प�म��काय���कया।�
• राजनी�त�के��लए�आक�ष�त�, मालवीय�1886 म��इसके�कलक�ा�स��म��भारतीय�रा�ीय�कां�ेस�म��शा�मल��ए�- इसक���थापना�एक
साल�पहले�मुंबई�के�गोकुलदास�तेजपाल�सं�कृत�कॉलेज�म��क��गई�थी।�
• मालवीय�का�कद�बढ़ा�, और�चार�बार�अ�य��बने�- 1909 ( लाहौर) म��, 1918 म��(�द�ली) , 1930 ( �द�ली) म��, और�1932 (
कलक�ा) म�।�मालवीय�लगभग�50 वष��तक�कां�ेस�का��ह�सा�थे।�
• मालवीय��ह���महासभा�के�शु�आती�नेता��म��से�एक�थे�तथा�उ�ह�ने�इसक���थापना�म��मदद�क�।�

कथन��व�ेषण:

कथन�1 कथन�2 कथन�3

स�य� अस�य� स�य�

2015 म��, सरकार�ने


उ�ह�ने�ऐनी�बेस�ट�और�अ�य�लोग��के�साथ� 1930 म��, जब�महा�मा�गांधी�ने�नमक मालवीय�को�उनक��मृ�यु�के�
1916 म��बनारस��ह����व��व�ालय�क� स�या�ह�और�स�वनय�अव�ा�आंदोलन�शु� 68 वष��बाद�भारत�के
�थापना�क��, उ�ह�ने�1919 से�1938 त �कया�, तो�उ�ह�ने�इसम��भाग��लया�और सव��च�नाग�रक�स�मान�,
क�कुलप�त�के��प�म��काय���कया।� �गर�तारी�द�।� भारत�र�न�से�स�मा�नत
�कया।�

(Source: https://indianexpress.com/article/explained/who-was-pandit-madan-mohan-malaviya-
freedom-fighter-and-educationist-6184723/ )

QUESTION 6.
�न�न�ल�खत�कथन��पर��वचार�कर�:

1. भारतीय�इ�तहास�कां�ेस�( IHC) का�ज�म�पुणे�, महारा��म��लगभग�50 ��त�न�धय��के�साथ��आ�था।�


2. नवंबर�1919 म��पहला�अ�खल�भारतीय��ा�य�स�मेलन�( All India Oriental Conference) बंगाल�म��आयो�जत��कया�गया
था।�

ऊपर��दए�गए�कथन��म��से�कौन�सा�गलत�है�/ ह��?

IASbaba
Web: http://ilp.iasbaba.com/ Score:
Email: ilp@iasbaba.com 3.00 / 200
Page 112
AIPTS/ILP VETERANS-
Exam Title :
2020 TE...
Email : yadavanurag075@gmail.com
Contact : 8882839768

a) केवल�1
b) केवल�2
c) 1 और�2 दोन��
d) न�तो�1 और�न�ही�2
Correct Answer: B
Your Answer:
Explanation

Solution (b)

Basic Information:

• हाल�ही�म��, भारतीय�इ�तहास�कां�ेस�( IHC) का�80 वां�स��28 और�30 �दसंबर�2019 के�बीच�क�ूर�, केरल�म��आयो�जत��कया


गया�था।�
• IHC का�ज�म�लगभग�50 ��त�न�धय��के�साथ�एक�अकादमी�के��प�म���आ�था�, जो�भारत�म���कये�गए�ऐ�तहा�सक�काय��के�मानक
को��व�नय�मत�करते�ह��तथा��न�प��और�मूल�इ�तहास�को�बढ़ावा�दे ते�ह�।�
• द�ो�वामन�पो�ार�, सुर���नाथ�सेन�(जो�बाद�म��भारत�के�रा�ीय�अ�भलेखागार�के�पहले��नदे शक�बने) , और�सर�शफ़ात�अहमद�खान
जैसे�इ�तहासकार��ने�पहले�स��म��भाग��लया�
• वा�तव�म��, 1946 म��, सरकार�ने�या�चकाकता���को�अ�भलेखागार�तक�प�ंचने�क��अनुम�त�दे ने�के��लए�या�चका�दायर�क�।�बाद�म��,
1977 म��, IHC ने�साव�ज�नक�जीवन�म��सां�दा�यक�बयानबाजी�के�उपयोग�और��मारक��के�राजनी�तक�आरोप�के�खतर��के���त
आगाह��कया।�

कथन��व�ेषण:

कथन�1 कथन�2

स�य� अस�य�

BISM ने�अपनी�रजत�जयंती�मनाने�के��लए�1935 म��एक


भंडारकर�सं�थान�के�त�वावधान�म��पुणे�म��नवंबर�1919 म�
अ�खल�भारतीय�कां�ेस�का�आयोजन��कया।�इस�घटना�के
पहला�अ�खल�भारतीय��ा�य�स�मेलन�, �वशेष��प�से��ाचीन
प�रणाम�व�प�IHC क��शु�आत��ई।�काय��म�का�आयोजन
भारतीय�इ�तहास�पर�क���त�था।�
पुणे�के�परशुरामभाऊ�कॉलेज�के�अस�बली�हॉल�म���कया�गया�

(Source: https://indianexpress.com/article/opinion/columns/indian-history-congress-that-
wasnt-5514300/ )

QUESTION 7.
आय��समाज�के�संदभ��म���न�न�ल�खत�कथन��पर��वचार�कर�:

1. आय��समाज�ने�शु���क��अवधारणा�को��वक�सत��कया�, �जसका�उ�े �य�ईसाई�धम��, इ�लाम�और��सख�धम��से�पुनःप�रवत�न�का�था।�


2. इसने�मू�त�पूजा�, ब�दे ववाद�, बाल��ववाह�, �वधवा���चय��, �ाहमण��के��भु�व�और�जा�त��व�था�जैसी�मौजूदा��ह����था��क�
तीखी�आलोचना�क�।�
3. लाला�लाजपत�राय�आय��समाज�के�सं�थापक��ववेकानंद�के�अनुयायी�बन�गए�

ऊपर��दए�गए�कथन��म��से�कौन�सा�सही�है�/ ह��?

IASbaba
Web: http://ilp.iasbaba.com/ Score:
Email: ilp@iasbaba.com 3.00 / 200
Page 113
AIPTS/ILP VETERANS-
Exam Title :
2020 TE...
Email : yadavanurag075@gmail.com
Contact : 8882839768

a) केवल�1 और�2
b) केवल�2 और�3
c) केवल�1 और�3
d) 1, 2 और�3
Correct Answer: A
Your Answer:
Explanation

Solution (a)

Basic Information:

• आय��समाज�आंदोलन�, पुन��थानवाद��के��प�म��य��प�साम�ी�म��नह��है�, प��मी��भाव��क����त��या�का�प�रणाम�था।�इसके


सं�थापक�दयानंद.सर�वती�(या�मूलशंकर�, 1824-83) का�ज�म�गुजरात�के�पुराने�मोरवी�रा�य�म���आ�था।�
• दयानंद�के��वचार��को�उनके���स��काय��, स�याथ���काश�(स�य��ा�या) म���का�शत��कया�गया�था।�
• उ�ह�ने�वेद��से��ेरणा�ली�तथा�उ�ह��" भारत�क��युग��क��च�ान�" (India's Rock of Ages) माना�, जो��क��ह���धम��का�अचूक
और�वा�त�वक�मूल�था।�
• उ�ह�ने�" वेद��क��ओर�लौटो�" का�नारा��दया।�
• आय��समाज�ने�मू�त�पूजा�, ब�दे ववाद�, बाल��ववाह�, �वधवा���चय��, �वदे श�या�ा�, �ा�ण��का��भु�व�और�जा�त��व�था�जैसी
मौजूदा��ह����था��क��तीखी�आलोचना�क�।�
• लाला�लाजपत�राय�आय��समाज�से�जुड़े�थे�तथा�आय��गजट�के�संपादक�थे�, �जसे�उ�ह�ने�अपने�छा��काल�के�दौरान��था�पत��कया�था।�

कथन��व�ेषण:

कथन�1 कथन�2 कथन�3

स�य� स�य� अस�य�

आय��समाज��ारा�शु���कए उ�ह�ने��नरथ�क�सं�कार��क���न�दा�क��, �व�भ�


गए�शु���काय��म�का दे वी-दे वता��क��छ�वय��क��पूजा�क��, �ज�ह�
उ�े �य�उन�लोग��के��लए��ह�� ने�लोग��को�कई�जुझा��सं�दाय��म���वभा�जत आय��समाज�क���थापना�1875 म�
धम��के��ार�खोलना�था कर��दया�, �ा�ण��के�अ�धकार�का�खंडन दयानंद�सर�वती�ने�क��थी।�
�ज�ह�ने�अ�य�धम��को �कया�तथा�सभी�धा�म�क�अंध�व�ास��के
अपनाया�था।� �खलाफ�धम�यु��का�शुभारंभ��कया।�

(Source: Spectrum)

QUESTION 8.
“ पावट��एंड�अन���टश��ल�इन�इं�डया�” �कसके��ारा��लखा�गया�था�

a) दादाभाई�नौरोजी�
b) एम�जी�रानाडे�
c) आर�सी�द��
d) महा�मा�गांधी�

IASbaba
Web: http://ilp.iasbaba.com/ Score:
Email: ilp@iasbaba.com 3.00 / 200
Page 114
AIPTS/ILP VETERANS-
Exam Title :
2020 TE...
Email : yadavanurag075@gmail.com
Contact : 8882839768

Correct Answer: A
Your Answer:
Explanation

Solution (a)

Basic Information:

• 4 �सतंबर�, 2019 को�दादाभाई�नौरोजी�, " भारत�के�वयोवृ��पु�ष�” (Grand Old Man of India) क��194 व��जयंती�थी
, जो�दे श�म��रा�ीय�चेतना�जगाने�वाले�आरं�भक�नेता��म��से�थे।�
• 1825 म��नवसारी�म��ज�मे�, वत�मान�गुजरात।�
• वह�अपने�शु�आती�चरण�म��भारतीय�रा�ीय�कां�ेस�के�साथ��नकटता�से�जुड़े�थे�, और����टश�संसद�के�पहले�भारतीय�सद�य�के��प�म�
काय���कया।�
• 1895 म��, वह�भारतीय�खच��पर�रॉयल�आयोग�के�सद�य�बने।�
• नौरोजी�का��थायी�बौ��क�योगदान�‘ �नकासी��स�ांत�’ (Drain Theory) को�उनक��1901 क��पु�तक�‘ पावट��एंड�अन���टश
�ल�इन�इं�डया�’ म���सा�रत�करना�था।�
• नौरोजी�ने�तक���दया��क�सा�ा�यवाद����टे न�भारत�क��संप���को�शोषणकारी�आ�थ�क�नी�तय��के�मा�यम�से�ले�जा�रहा�है�, �जसम�
�वदे �शय���ारा�भारत�का�शासन�भी�शा�मल�है�; भारत�म�����टश�नाग�रक�और�सै�य�तं��का�भारी��व�ीय�बोझ�; मु���ापार�के�कारण
दे श�का�शोषण�; गैर-भारतीय��ने�भारत�म��जो�पैसा�कमाया�था�, उसे�ले�गए�; तथा�भारत�ने���टे न�म��आयो�जत�अपने�साव�ज�नक�ऋण
पर�जो��याज��दया�था।�

(Source: https://indianexpress.com/article/explained/explained-remembering-dadabhai-naoroji-
on-his-birthday-5966633/ )

QUESTION 9.
पानीपत�क��तीसरी�लड़ाई�( 1761) के�संदभ��म���न�न�ल�खत�कथन��पर��वचार�कर�:

1. यह�मराठा�सा�ा�य�और��रा�नी�सा�ा�य�(अफगा�न�तान) क��सेना��के�बीच�लड़ा�गया�था।�
2. बाजीराव�पेशवा�मराठा�सेना�के��मुख�कमांडर�थे।�
3. शाह�आलम���तीय�यु��के�दौरान�मुगल�स�ाट�था।�

ऊपर��दए�गए�कथन��म��से�कौन�सा�सही�है�/ ह��?

a) केवल�1
b) केवल�2 और�3
c) केवल�1 और�3
d) 1, 2 और�3
Correct Answer: C
Your Answer:
Explanation

Solution (c)

Basic Information:

• अ�दाली�को�दो�भारतीय�सहयो�गय�- रो�हला�नजीब-उद-दौला�, दोआब��े��के�अफगान��और�शुजा-उद-दौला-अवध�के�नवाब��ारा


सम�थ�त��कया�गया�था।�
• मराठ��को�यु��म��परा�जत��कया�गया�, �जसम��उनके�40-50,000 सै�नक�मारे�गए�, जब�क�अ�दाली�क��सेना�को�लगभग�20,000
हताहत��का�सामना�करना�पड़ा।�
• मराठा�सा�ा�य�क��संभावना��पर�इसका�ब�त��भाव�पड़ा।�यु��म��भाऊ�, पेशवा�के�बेटे�, �व�ास�राव�, जसवंत�राव�पवार�, तुकोजी
�स��धया�आ�द�स�हत�कई�मह�वपूण��मराठा��मुख�मारे�गए।�
• मराठा�उ�थान�रोक�लग�गई�थी�ले�कन�उ�ह�ने�पेशवा�माधवराव�के�नेतृ�व�म��दस�साल�बाद��द�ली�पर�वापस��भु�व�बढ़ा��लया�था।�

IASbaba
Web: http://ilp.iasbaba.com/ Score:
Email: ilp@iasbaba.com 3.00 / 200
Page 115
AIPTS/ILP VETERANS-
Exam Title :
2020 TE...
Email : yadavanurag075@gmail.com
Contact : 8882839768

• 1526 म��पानीपत�क��पहली�लड़ाई�, बाबर�और�इ�ा�हम�लोधी�के�बीच�लड़ी�गई�थी।�यह��भारत�म��मुगल�सा�ा�य�क��न�व�रखी�गयी


थी।�
• 1556 म��पानीपत�क���सरी�लड़ाई�, अकबर�और�राजा�हेमू�‘ �व�मा�द�य�’ के�बीच�लड़ी�गई�थी।�

कथन��व�ेषण:

कथन�1 कथन�2 कथन�3

स�य� अस�य� स�य�

पानीपत�क��तीसरी�लड़ाई�जो
सदा�शवराव�भाऊ�मराठा�सेना�के�कमांडर�थे
अफगा�न�तान�के��रा�नी�सा�ा�य�के शाह�आलम���तीय�मुगल�स�ाट�था�और
तथा�यु��के�समय�बाजीराव�पेशवा�के
शासक�अहमद�शाह�अ�दाली�और�मराठ� मराठ��के�संर�ण�म��था।�
उ�रा�धकारी�बालाजी�बाजीराव�थे।�
के�बीच�लड़ी�गई�थी।�

(Source: https://theprint.in/opinion/panipat-was-a-bloody-military-debacle-for-marathas-will-
patriotism-high-india-see-the-film/317220/ )

QUESTION 10.
�न�न�ल�खत�कथन��पर��वचार�कर�:

1. झांसी�क��रानी�क��मृ�यु�कै�टन�हे�गे�( Captain Heneage) के�नेतृ�व�म��8 व���सर��( 8th Hussars) के�एक�द�ते�से�लड़ाई


के�दौरान��ई�
2. झाँसी�पहली�ऐसी��रयासत�थी��जसे�अं�ेज़��ने��पगत�के��स�ांत�के�तहत�अ�ध��हत��कया�था।�

ऊपर��दए�गए�कथन��म��से�कौन�सा�सही�है�/ ह��?

a) केवल�1
b) केवल�2
c) 1 और�2 दोन��
d) न�तो�1 और�न�ही�2
Correct Answer: A
Your Answer:
Explanation

Solution (a)

Basic Information:

• रानी�ल�मीबाई�का�ज�म�19 नवंबर�, 1828 को�वाराणसी�म��म�णक�ण�का�तांबे�( Manikarnika Tambe) के��प�म���आ�था।


रानी�का��ववाह�झाँसी�के�राजा�, राजा�गंगाधर�नेवलकर�से�1842 म���आ�था।�
• उसने�1851 म��एक�लड़के�को�ज�म��दया�, �जसका�नाम�दामोदर�राव�रखा�गया�, �जसक��चार�महीने�बाद�मृ�यु�हो�गई�थी।�महाराजा�ने
एक��दन�पहले�गंगाधर�राव�के�चचेरे�भाई�के�बेटे�आनंद�राव�नामक�एक�ब�चे�को�गोद��लया�था�, �जसका�नाम�महाराजा�के�मरने�से
पहले�दामोदर�राव�रखा�गया�था।�

IASbaba
Web: http://ilp.iasbaba.com/ Score:
Email: ilp@iasbaba.com 3.00 / 200
Page 116
AIPTS/ILP VETERANS-
Exam Title :
2020 TE...
Email : yadavanurag075@gmail.com
Contact : 8882839768

• गवन�र-जनरल�लॉड��डलहौज़ी�के�अंतग�त�, �पगत�का��स�ांत�लागू��कया�गया�था�, �जसने�दामोदर�राव�के��स�हासन�के�दावे�को


खा�रज�कर��दया�और�रा�य�को�अपने��े���म��वापस�कर��लया�था।�
• माच��1854 म��, रानी�ल�मीबाई�को�60,000 �पये�क��वा�ष�क�प�शन�द��गई�तथा�महल�और��कले�को�छोड़ने�का�आदे श��दया।�
• �पगत�का��स�ांत�एक��वलोपन�नी�त�थी�, �जसके�ऊपर�लॉड��डलहौजी�ने��ापक��प�से�भारत�के�गवन�र-जनरल�के��प�म��1848
से�1856 तक�काय���कया।�
• इसके�तहत�, ई�ट�इं�डया�कंपनी�के���य��या�अ��य��(एक�जागीरदार) �नयं�ण�के�तहत�कोई�भी��रयासत�जहां�शासक�के�पास
कानूनी�पु�ष�वा�रस�नह��था�, कंपनी��ारा�अ�ध��हत��कया�जाएगा।�
• साथ�ही�, भारतीय�शासक�के��कसी�भी�द�क�पु��को�रा�य�का�उ�रा�धकारी�घो�षत�नह���कया�जा�सकता�था।�

कथन��व�ेषण:

कथन�1 कथन�2

स�य� अस�य�

�वा�लयर�के�फूल�बाग�के�पास�कोताह-क�-सराय�म��17 सतारा�( 1848) पहली��रयासत�थी�, उसके�बाद�जैतपुर�और


जून�को�, कै�टन�हेनगेड�के�नेतृ�व�म��8 व��(राजा�क� संबलपुर�( 1849), बघाट�( 1850), छोटा�उदयपुर�( 1852
रॉयल�आय�रश) �सस��क��एक�टु कड़ी�, रानी�ल�मीबाई ), झांसी�( 1853) और�नागपुर�( 1854) �पगत��स�ांत�के
�ारा��नद� �शत�बड़ी�भारतीय�सेना�से�लड़ी।� तहत�अ�ध��हत��कया�गया�था।�

(Spectrum)

QUESTION 11.
�न�न�ल�खत�म��से��कस�भारतीय��ां�तकारी�ने�कन�ल�रे�जना�ड�एडवड��हैरी�डायर�क��ह�या�क��थी�?

a) मदन�लाल�ढ�गरा�
b) उधम��स�ह�
c) खुद�राम�बोस�
d) इनम��से�कोई�भी�नह��
Correct Answer: D
Your Answer:
Explanation

Solution (d)

Basic Information:

• डायर�क��1927 म��सेरे�ल�र���ाव�और�धमनीका�ठ�य�( cerebral haemorrhage and arteriosclerosis) से�मृ�यु


�ई�थी।�
• 13 माच��, 1940 को�उधम��स�ह�ने�ई�ट�इं�डया�एसो�सएशन�और�रॉयल�स��ल�ए�शयन�सोसाइट��ऑफ़�के�सटन��हल�क��बैठक�म��ओ�
' डायर�क��ह�या�क��थी।�वह�एक�राजनी�तक�काय�कता��थे�, जो�अमे�रका�म��रहते��ए�ग़दर�पाट��से�जुड़े��ए�थे।�
• उ�ह�ने�1919 म��पंजाब�के�ले��टन�ट-गवन�र�के��प�म��ओ�' डायर�को�वा�त�वक�अपराधी�माना�था।�
• 13 अ�ैल�, 1919 को�, बैसाखी�के��दन�, ��गे�डयर�जनरल�(अ�थायी�र�क) रे�गना�ड�डायर�ने�चेतावनी�द���क�रॉलेट�ए�ट�के�तहत
सैफु��न��कचलू�जैसे�नेता��क���गर�तारी�के��वरोध�म��ज�लयांवाला�बाग�म��एक��त�भीड़�पर�गो�लयां�चलाने�का�आदे श��दया�जाए।�

IASbaba
Web: http://ilp.iasbaba.com/ Score:
Email: ilp@iasbaba.com 3.00 / 200
Page 117
AIPTS/ILP VETERANS-
Exam Title :
2020 TE...
Email : yadavanurag075@gmail.com
Contact : 8882839768

• 1919 का�अराजक�और��ां�तकारी�अपराध�अ�ध�नयम�, �जसे�सामा�य��प�से�रौलट�ए�ट�के��प�म��जाना�जाता�है�, ज�लयांवाला


बाग�म��नरसंहार�से�एक�महीने�पहले�लागू��आ�था।�
• लॉड��हंटर�क��अ�य�ता�म��एक�अ�नयमता�जांच�स�म�त�का�गठन��कया�गया�था।�स�म�त�ने�डायर�क��सी�मत�आलोचना�क��ले�कन
ले��टन�ट�गवन�र�माइकल�ओ�' डायर�को�छोड़��दया�गया�था।�

(Source: Spectrum)

QUESTION 12.
�न�न�ल�खत�कथन��पर��वचार�कर�:

1. आज़ाद��ह�द�फ़ौज�का�गठन�दो�बार�- कै�टन�मोहन��स�ह�और�सुभाष�चं��बोस�के�अंतग�त��आ�था।�
2. कै�टन�मोहन��स�ह�भारतीय�रा�ीय�सेना�के�पहले�कमांडर�बने।�
3. बोस�ने�जम�नी�क��रे�डयो�सेवा�के��ह�से�के��प�म��आज़ाद��ह�द�रे�डयो�शु���कया�, जो�पहली�बार�7 जनवरी�1942 को��स�गापुर�म�
�सा�रत��आ।�

ऊपर��दए�गए�कथन��म��से�कौन�सा�सही�है�/ ह��?

a) केवल�1 और�2
b) केवल�2
c) केवल�1 और�3
d) केवल�2 और�3
Correct Answer: A
Your Answer:
Explanation

Solution (a)

Basic Information:

• जापान�ने�मेजर�फु�जवारा�को��वासी�भारतीय��से�संपक��करने�के��लए�द��ण-पूव��ए�शया�भेजा�, जो��ीतम��स�ह�जैसे�लोग��के�नेतृ�व�म�
�वयं�को�भारतीय��वतं�ता�लीग�म��संग�ठत�कर�रहे�थे।�
• �सतंबर�1942 तक�, आईएनए�औपचा�रक��प�से�मोहन��स�ह�के�अंतग�त�अ��त�व�म��था।�
• जैसा��क�मोहन��स�ह�ने��वाय�ता�और�संब����थ�त�पर�जोर��दया�, उ�ह��कमान�से�हटा��दया�गया�तथा��गर�तार�कर��लया�गया�था।
रास��बहारी�बोस�ने�कुछ�समय�के��लए�इस�पर�अ�धकार�करने�क��को�शश�क��, ले�कन�वह�तब�इस�काय��के��लए�वृ��थे।�
• मई�1943 म��एक�लंबी�और�क�ठन�पनडु �बी�या�ा�के�बाद�, बोस�द��ण-पूव��ए�शया�म��प�ंचे।�
• वह�अपनी�सेना�, आजाद��ह�द�फौज�(���इं�डया�आम�) या�भारतीय�रा�ीय�सेना�के�सव��च�कमांडर�बन�गए�, �ज�ह�ने�1945 तक
लगभग�चालीस�हजार�लोग��क��भत��क��थी�, �जसम��1857 क����स��झांसी�क��रानी�के�नाम�पर�एक�म�हला�रे�जम�ट�भी�थी।�
• अ�टू बर�1943 म��, उ�ह�ने��वतं��भारत�क��एक�अनं�तम�सरकार�क���थापना�क��, �जसे�तुरंत�जापान�और�बाद�म��जम�नी�और
फ़ा�स�ट�इटली�स�हत�आठ�अ�य�सरकार��ने�मा�यता�द�।�

कथन��व�ेषण:

कथन�1 कथन�2 कथन�3

स�य� स�य� अस�य�

IASbaba
Web: http://ilp.iasbaba.com/ Score:
Email: ilp@iasbaba.com 3.00 / 200
Page 118
AIPTS/ILP VETERANS-
Exam Title :
2020 TE...
Email : yadavanurag075@gmail.com
Contact : 8882839768

�दसंबर�1941 म��, कै�टन�मोहन��स�ह�,


जापान�के�साथ��मलकर�भारत�को��वतं� ���टश�भारतीय�सेना�के�पंजाब�रे�जम�ट�के�एक सुभाष�चं��बोस�ने�आज़ाद��ह�द�रे�डयो�,
करने�के��लए�जापा�नय��के�साथ�POWs युवा�अ�धकारी�, कै�टन�मोहन��स�ह�, �ज�ह�ने माच��1942 म��ब�ल�न�से��सा�रत��कया।
( यु��बंद�) के�साथ�एक�भारतीय�सेना जापान�म��आ�मसमप�ण�कर��दया�था�, ने बोस�ने�अपना��सारण�और��वदे शी
जुटाने�म��सहयोग�करने�के��लए�सहमत भारत�को�आजाद�कराने�के��लए�जापा�नय��के �सारण�जम�नी�से�भारत�म��जारी�रखा�,
�ए।�ले�कन�, 1943 क��शु�आत�म� साथ�माच��करने�के��लए�POWs के�साथ तथा�अंततः�द��ण-पूव��ए�शया�म��, जून�
पहला�INA �योग�लगभग��व�त�हो�गया। एक�भारतीय�सेना�जुटाने�के��लए�फु�जवारा�के 1945 तक�, तथा�अपनी�भारतीय
मई�1943 म��बोस�द��ण�पूव��ए�शया साथ�सहयोग�करने�पर�सहम�त����क�।�इस रा�ीय�सेना�के�1944 म��अं�ेज���ारा
प�ंचे�और�तुरंत�आईएनए�के��सरे�अवतार �कार�INA के�पहले�कमांडर�बन�गए।� परा�जत�होने�के�बाद�भी�जारी�रखा।�
पर��नयं�ण�कर��लया।�

QUESTION 13.
" सव�ट्स�ऑफ�इं�डया�सोसाइट��" �कसके��ारा��था�पत��कया�गया�था�

a) बाल�गंगाधर��तलक�
b) एम�के�गांधी�
c) गोपाल�कृ�ण�गोखले�
d) दादाभाई�नौरोजी�
Correct Answer: C
Your Answer:
Explanation

Solution (c)

Basic Information:

• 1905 म��, गोखले�ने���श�ण��दान�करने�के�उ�े �य�से�' सव�ट्स�ऑफ़�इं�डया�सोसाइट��' क���थापना�क��, ता�क�वे��वयं�को�भारत


क��सेवा�म��रा�ीय��मशन�रय��के��प�म��सम�प�त�कर�सक��तथा�सभी�संवैधा�नक�मा�यम��से�भारतीय�लोग��के�रा�ीय��हत��को�बढ़ावा�दे
सक�।�
• अकाल�राहत�, संघ�संगठन�, सहकारी�स�म�तय��और�आ�दवा�सय��के�उ�थान�के��े��म��, सोसाइट��ने�सराहनीय�काय���कया।�
• 1911 से�, यह�नागपुर�से�अं�ेजी�म��" �हतवाद�" शीष�क�से�अपना�समाचार�प��भी��का�शत��कया।�यह�सोसाइट��अभी�भी�मौजूद�है
और�इसका�मु�यालय�पुणे�म��है।�
• गोखले�क��मृ�यु�( 1915) के�बाद�, �ी�नवास�शा��ी�ने�अ�य��पद�संभाला।�
• गोपाल�कृ�ण�गोखले�का�ज�म�9 मई�1866 को�र�ना�गरी�के�काटु लक�म���आ�था।�गोखले�एम.जी.रानाडे�से�ब�त��भा�वत�थे�, �ज�ह�
वे�राजनी�तक�और�साव�ज�नक�जीवन�म��अपने�गु��के��प�म��मानते�थे�, दादाभाई�नौरोजी�और��फरोजशाह�मेहता�जो�उनके�नायक�थे।�
• गोखले�जीवन�कालीन�सद�य�के��प�म��द�कन�एजुकेशन�सोसायट��, पूना�म��शा�मल��ए।�जब�1885 म��फ�यू�सन�कॉलेज�खोला�गया
, तो�उ�ह��वहां��ा�यान�दे ने�के��लए�बुलाया�गया।�वह�1902 म���वशेष��प�से��वयं�को�साव�ज�नक�जीवन�के��लए�सम�प�त�करने�के
�लए�सेवा�नवृ���ए।�

(Source: Spectrum)

QUESTION 14.
1946 म��अंत�रम�सरकार�के�गठन�के�संदभ��म���न�न�ल�खत�कथन��पर��वचार�कर�:

1. यह�भारत�के�इ�तहास�का�एकमा��ऐसा�मं��मंडल�था��जसम��क�र���त�ं ���कां�ेस�और�मु��लम�लीग�ने�क���म��स�ा�साझा�क�।�
2. आ�खरकार�मु��लम�लीग�के���त�न�धय���ारा�मुसलमान��के��लए�आर��त�सभी�पाँच�मं�ालय��पर�अ�धकार�कर��लया�गया।�
3. �ारंभ�म��, मौलाना�अबुल�कलाम�आज़ाद�को�मं��मंडल�म��शा�मल�नह���कया�गया�था।�

ऊपर��दए�गए�कथन��म��से�कौन�सा�सही�है�/ ह��?

IASbaba
Web: http://ilp.iasbaba.com/ Score:
Email: ilp@iasbaba.com 3.00 / 200
Page 119
AIPTS/ILP VETERANS-
Exam Title :
2020 TE...
Email : yadavanurag075@gmail.com
Contact : 8882839768

a) केवल�1 और�2
b) केवल�2 और�3
c) केवल�1 और�3
d) 1, 2 और�3
Correct Answer: D
Your Answer:
Explanation

Solution (d)

Basic Information:

• 1946 म��, जवाहरलाल�नेह��के�नेतृ�व�म��भारत�क��अंत�रम�सरकार�का�गठन��कया�गया�था।�यह�भारत�के�इ�तहास�म��एकमा��ऐसी


कै�बनेट�थी�, �जसम��क�र���त�ं ���कां�ेस�और�मु��लम�लीग�ने�क���म��स�ा�साझा�क��थी।�
• अंत�रम�सरकार�ने��वाय�ता�क��एक�बड़ी��ड�ी�के�साथ�काम��कया�, तथा����टश�शासन�के�अंत�तक�स�ा�म��बनी�रही�, �जसके�बाद
भारत�और�पा�क�तान�के�डो�म�नयन��को�बनाया�गया।�
• 1942 म�����स��मशन�ने�भारत�म��अंत�रम�सरकार�बनाने�के��लए�कई��यास��कए।�
• 1946 म��, ���टश��धानमं�ी��लीम�ट�एटली��ारा�भेजे�गए����टश�कै�बनेट��मशन�के���ताव��के�बाद�सं�वधान�सभा�के�चुनाव��ए।�
• वायसराय�वेवेल�ने�बाद�म��भारतीय���त�न�धय��से�अंत�रम�सरकार�म��शा�मल�होने�का�आ�ान��कया।�
• अंत�रम�सरकार�ने�1919 के�पुराने�भारत�सरकार�अ�ध�नयम�के�अनुसार�काय���कया।�

कथन��व�ेषण:

कथन�1 कथन�2 कथन�3

स�य� स�य� अस�य�

लीग�के�बाहर�रहने�का�फैसला�करने�के�बाद�, मु
सलमान��के��लए�आर��त�पाँच�मं�ालय��म��से
तीन�पर�आसफ़�अली�, सर�शफ़ात�अहमद�खान
मु��लम�लीग�ने�आरंभ�म��सरकार�से
, और�सैयद�अली�ज़हीर�, सभी�गैर-मु��लम �दसंबर�, 1946 म��मौलाना�अबुल
बाहर�रहने�का�फैसला��कया।�ले�कन
मु��लम���त�न�धय��ने�अ�धकार�कर��लया।�दो कलाम�आज़ाद�को�मं��मंडल�म��शा�मल
आ�खरकार�इस�तरह�से�क�र
पद�खाली�रह�गए।�ले�कन�बाद�म��लॉड��वेवेल�ने �कया�गया।�आरंभ�म��वह�कै�बनेट�म��नह�
��त�ं ��य��का�एक�साथ�आने�के��लए
मु��लम�लीग�को�सभी�पाँच�आर��त��वभाग��को थे�
सहमत�हो�गयी।�
आवं�टत�करने�पर�सहम�त����क��, अगर�वह
सहयोग�करने�के��लए�सहमत�हो�जाए�, जो�बाद
म��शा�मल�हो�गयी।�

(Source: https://indianexpress.com/article/explained/september-2-when-indias-interim-govt-was-
formed-in-1946-5959889/ )

QUESTION 15.
�न�न�म��से�कौन�सा�����व�मुंबई�क��ए�शया�टक�सोसाइट��का�सं�थापक�है�?

a) सर��व�लयम�जो�स�

IASbaba
Web: http://ilp.iasbaba.com/ Score:
Email: ilp@iasbaba.com 3.00 / 200
Page 120
AIPTS/ILP VETERANS-
Exam Title :
2020 TE...
Email : yadavanurag075@gmail.com
Contact : 8882839768

b) जे�स����सेप�
c) सर�चा�स���व��कंस�
d) सर�जे�स�मै�कनटोस�
Correct Answer: D
Your Answer:
Explanation

Solution (d)

Basic Information:

• मु�बई�क��ए�शया�टक�सोसाइट��ने�अपने�अ��त�व�के�215 वष��म��पहली�म�हला�रा�प�त��ोफेसर��व�पी�बालापो�रया�को�चुना�है।�
• मुब
ं ई�के�जय��ह�द�कॉलेज�म���ो��व�पी�बलापो�रया��व�ज�ट�ग�फैक�ट��है�, जहाँ�वे�पहली�उप-�ाचाय��और�अं�ेजी��वभाग�क���मुख�थ��,
उस�सं�थान�का�नेतृ�व�कर�गी�जो�उ�लेखनीय�ऐ�तहा�सक�कलाकृ�तय��का�भंडार�है।�
• ए�शया�टक�सोसाइट��ने�1804 म��सा�ह��यक�सोसायट��ऑफ�बॉ�बे�के��प�म��अपनी�या�ा�आरंभ�क�।�यह�एक��कॉ�टश�औप�नवे�शक
�शासक�सर�जे�स�मै�कनटोश��ारा��था�पत��कया�गया�था�, �जनक���ा�य�अ�ययन�म��गहरी���च�थी।�
• 1841 म��, सोसाइट��ने�जन�ल�ऑफ�द�बॉ�बे��ांच�ऑफ�द�रॉयल�ए�शया�टक�सोसाइट��नामक�एक�प��का�का��काशन�शु���कया�,
जो�मुंबई�के�ए�शया�टक�सोसाइट��के�नाम�से��का�शत�होती�है।�
• यह�आरंभ�म��गोरे�यूरोपीय�लोग���ारा�और�उनके��लए�शु���कया�गया�था�, ले�कन�वष��1841 म��सोसायट��ने�भारतीय��को�सद�य�के
�प�म���वीकार�करना�आरंभ��कया।�
• 1826 म��, सा�ह��यक�स�म�त�लंदन���थत�रॉयल�ए�शया�टक�सोसाइट��ऑफ़��ेट���टे न�और�आयरल�ड�क��मुंबई�शाखा�बन�गई�तथा
इसे�रॉयल�ए�शया�टक�सोसाइट��( BBRAS) क��बॉ�बे�शाखा�कहा�जाने�लगा।�
• 1954 म��, सं�था�को�लंदन�के�अपने�मूल�से�अलग�कर��दया�गया�तथा�ए�शया�टक�सोसाइट��ऑफ�बॉ�बे�बन�गयी।�2002 म��, इसने
अपना�वत�मान�नाम��ा�त�कर��लया�था।�

(Source: https://indianexpress.com/article/explained/explained-now-with-first-woman-president-
vispi-balaporia-what-is-asiatic-society-mumbai-5962907/ )

QUESTION 16.
�न�न�ल�खत�कथन��पर��वचार�कर�:

1. वह�भारत�के�पहले�उपरा�प�त�और��सरे�रा�प�त�थे।�
2. उ�ह�ने�अ�ै त�वेदांत�क��सही��श�ा��का��सार�करने�क��मांग�क�।�
3. उनके��मुख�काय��म��भारतीय�दश�न�, लेख�‘ द�ए�थ�स�ऑफ�द�भगवद�गीता�और�कांट�’ शा�मल�ह�।�

उपयु���कथन��से�कौन-सा�����व�संब��है�?

a) लाल�बहा�र�शा��ी�
b) डॉ. सव�प�ली�राधाकृ�णन�
c) वराह�ग�र�व�कट��ग�र�
d) डॉ. जा�कर��सैन�
Correct Answer: B
Your Answer:
Explanation

Solution (b)

Basic Information:

• �श�क��दवस�5 �सतंबर�को���तवष��मनाया�जाता�है�, �य��क�डॉ. राधाकृ�णन�का�ज�म��सतंबर�5,1888 को��आ�था।�

IASbaba
Web: http://ilp.iasbaba.com/ Score:
Email: ilp@iasbaba.com 3.00 / 200
Page 121
AIPTS/ILP VETERANS-
Exam Title :
2020 TE...
Email : yadavanurag075@gmail.com
Contact : 8882839768

• जब�वह�भारत�के�रा�प�त�बने�, तो�उनके�कुछ�छा���और�दो�त��ने�उनसे�5 �सतंबर�को�उ�ह��अपना�ज�म�दन�मनाने�क��अनुम�त�दे ने�का


अनुरोध��कया।�उ�ह�ने�जवाब��दया�, " मेरा�ज�म�दन�मनाने�के�बजाय�, 5 �सतंबर�को��श�क��दवस�के��प�म��मनाया�जाए�तो�यह�मेरा
गौरवपूण��सौभा�य�होगा।�"
• उ�ह��कलक�ा��व��व�ालय�म���क�ग�जॉज��पंचम�के�मान�सक�और�नै�तक��व�ान��नकाय�के�अ�य��के��प�म��दश�नशा���के��ोफेसर�के
�प�म���नयु���कया�गया�था।�
• उ�ह��ऑ�सफोड���व��व�ालय�म��पूव��धम��और�नै�तकता�के�एक��ोफेसर�के��प�म���नयु���कया�गया�था।�
• वह�बनारस��ह����व��व�ालय�के�कुलप�त��नयु��होने�वाले�पहले�भारतीय�थे।�
• राधाकृ�णन�को�सा�ह�य�म��नोबेल�पुर�कार�के��लए�16 बार�और�अ�य�नोबल�पुर�कार�के��लए�11 बार�नामां�कत��कया�गया�था।�
• डॉ. सव�प�ली�राधाकृ�णन�को�वष��1931 म��नाइट�बैचलर�के��प�म���नयु���कया�गया�था।�उ�ह��1938 म�����टश�अकादमी�के�फेलो
के��प�म��चुना�गया�था।�उ�ह��1954 म��भारत�र�न�और�1963 म��ऑड�र�ऑफ�मे�रट�से�भी�स�मा�नत��कया�गया�था।�

कथन��व�ेषण:

कथन�1 कथन�2 कथन�3

राधाकृ�णन�को�1952
म��भारत�का�पहला उ�ह�ने�उन�लोग��के�बीच�अ�ै त�वेदांत�क��सही
उनक��सा�ह��यक�कृ�तय��म��शा�मल�ह��- ए
उपरा�प�त�चुना�गया�था। �श�ा��को�फैलाने�क��मांग�क��, जो��ह���धम��के
सोस��बुक�इन�इं�डयन��फलॉसफ��, द�व�ड�
वह�(राज����साद�1950 वा�त�वक�सार�के�बारे�म��अ�ान�थे।�उ�ह�ने�वेदांत
�े जरी�ऑफ�मॉडन���रली�जयस�, आइ�डय
से�1962 तक) के�बाद� क��नै�तकता�पर�एक�थी�सस�तैयार�क��, �जसका
�ल�ट��ू�ऑफ�लाइफ�, �र�लजन�एंड
1962 से�1967 तक उ�े �य�इस�आरोप�का�उ�र�दे ना�था��क�वेदांत
क�चर�, आ�द।�
भारत�के��थम�रा�प�त �णाली�म��नै�तकता�के��लए�कोई�जगह�नह��थी।�
बने।�

(Source: https://theprint.in/theprint-profile/sarvepalli-radhakrishnan-the-president-who-
defended-hinduism-against-western-criticism/222458/ and https://
economictimes.indiatimes.com/news/politics-and-nation/philosopher-teacher-president-
remembering-dr-s-radhakrishnan/proficient-statesman/slideshow/60374997.cms

QUESTION 17.
�न�न�ल�खत�म��से�कौन�सी�घटना�स�वनय�अव�ा�आंदोलन�क��समा��त�के�बाद�नह���ई�?

a) जय�काश�नारायण�, अचुत�पटवध�न�, अशोक�मेहता�, यूसुफ�मेहरअली�, नर���दे व�और�मीनू�मसानी�ने��वतं��पाट��(सीएसपी) का�गठन


�कया।�
b) रा�ीय�आंदोलन�क��भावी�रणनी�त�को�लेकर�भारतीय�रा�ीय�कां�ेस�म��आंत�रक�मतभेद�उभर�कर�सामने�आए।�
c) गांधी�जी�ने�कां�ेस�से�अपने�इ�तीफे�क��घोषणा��वचार�, श�द�और�कम��म��बेहतर�तरीके�से�करने�के��लए�क�।�
d) दोन���वक�प�( और�(
Correct Answer: A
Your Answer:
Explanation

Solution (a)

Basic Information:

• भारतीय�रा�ीय�कां�ेस�ने�मई�1933 म��स�वनय�अव�ा�आंदोलन�को�आ�धका�रक��प�से��नलं�बत�कर��दया�और�मई�1934 म��इसे


वापस�ले��लया।�

IASbaba
Web: http://ilp.iasbaba.com/ Score:
Email: ilp@iasbaba.com 3.00 / 200
Page 122
AIPTS/ILP VETERANS-
Exam Title :
2020 TE...
Email : yadavanurag075@gmail.com
Contact : 8882839768

• 1934 म��CSP का�गठन�INC के�भीतर��आ�तथा�उसने�समाजवाद��काय��म�के���त�अपने�उ�मुखीकरण�को�बदलने�क��को�शश


क��और�साथ�ही�साथ���ढ़वाद��' द��णपंथी�' के��भु�व�को�भी�दबाये�रखा।�
• 1936 म��लखनऊ�कां�ेस�म��दो�मु���, यानी�, प�रषद�म���वेश�और�काया�लय�क���वीकृ�त�पर�आमने-सामने�आ�गए�
• बॉ�बे�म��AICC क��बैठक�(अग�त�1936) ने�चुनाव�लड़ने�के�प��म���नण�य��लया�, ले�कन�चुनाव�समा�त�होने�तक�काया�लय��वीकृ�त
पर��नण�य�को��थ�गत�कर��दया।�
• माच��1937 म��AICC ने�नेह��और�अ�य�CSP नेता��क��आप��य��को�दर�कनार�कर�काया�लय�क���वीकृ�त��दान�क�।�
• अ�ैल�1936 म��कां�ेस�के�लखनऊ�अ�धवेशन�म��अ�खल�भारतीय��कसान�सभा�( AIKS) का�गठन�, सहजानंद�सर�वती�को�अपना
पहला�अ�य��चुना�गया।�

कथन��व�ेषण:

�वक�प�a �वक�प�b �वक�प�c

अस�य� स�य� स�य�

INC के�साथ�बु��जी�वय�
समाजवाद��और�अ�य�वामपंथी�त�व-उनम� गैर-जन�संघष��के�चरण�के�दौरान�रा�ीय का�एक�बड़ा�वग��संसद�य
सबसे�मह�वपूण��जय�काश�नारायण�, अचुत आंदोलन�क��भ�व�य�क��रणनी�त�पर�दो- राजनी�त�का�प�धर�था�, गां
पटवध�न�, अशोक�मेहता�, यूसुफ�मेहरअली�, न चरणीय�बहस��ई�- गांधी�जी�के धी�जी�का�जोर�चरखे�पर�था�,
र���दे व�और�मीनू�मसानी�ने�मई�1934 म� रचना�मक�काय��और�प�रषद���व��।�1 वे�इसे�" रा��के��सरे�फेफड़े�"
कां�ेस�समाजवाद��पाट��(सीएसपी) का�गठन 937 म��, �ांतीय�चुनाव��के�संदभ��म� के��प�म��मानते�थे।�नेह��से
�कया।�सी. राजगोपालाचारी�ने�1959 म� काया�लय��वीकृ�त�के����पर�मतभेद संघष��- �वराम�-संघष��( S-T-
�वतं��पाट��क���थापना�क�।� उभर�कर�सामने�आए� S) रणनी�त�को�लेकर�भी
गांधी�जी�के�साथ�मतभेद�थे�

(Source: From Plassey to Partition: A History of Modern India)

QUESTION 18.
�न�न�ल�खत�म��से��कस�जन�आंदोलन�क��शु�आत�करते��ए�, गांधी�ने�इसे�" मेरे�जीवन�क��सबसे�बड़ी�लड़ाई�" कहा�?

a) स�वनय�अव�ा�आंदोलन�
b) सां�दा�यक�अवाड��के��खलाफ�उपवास�
c) असहयोग�आंदोलन�
d) भारत�छोड़ो�आंदोलन�
Correct Answer: D
Your Answer:
Explanation

Solution (d)

Basic Information:

• 7 अग�त�, 1942 को�मुंबई�के�गोवा�लया�ट� क�मैदान�म��अ�खल�भारतीय�कां�ेस�कमेट��( AICC) के�उ�ाटन�के��दन�; गांधी�ने�कहा


, " म��पहले�क��तरह�अ�ह�सा�के��स�ांत�पर�कायम��ं।�अगर�आप�इससे�थक�गए�ह��तो�आपको�मेरे�साथ�नह��आने�क��ज�रत�है।�उस
समय�जब�म��अपने�जीवन�क��सबसे�बड़ी�लड़ाई�शु��करने�जा�रहा��ं।�मेरे��दल�म��अं�ेज��के��लए�कोई�नफरत�नह��है�”

IASbaba
Web: http://ilp.iasbaba.com/ Score:
Email: ilp@iasbaba.com 3.00 / 200
Page 123
AIPTS/ILP VETERANS-
Exam Title :
2020 TE...
Email : yadavanurag075@gmail.com
Contact : 8882839768

• 8 अग�त�1942 को�बॉ�बे�म��AICC �ारा�अपनाया�गया�" भारत�छोड़ो�" ��ताव�, गांधी�के��नद� शन�म��इस�जन�स�वनय�अव�ा�को


शु��करने�का���ताव�था�
• उ�ह�ने�मु�य��प�से�नेह��और�राजगोपालाचारी�क��ओर�से���य��कार�वाई�के��व���कां�ेस�के�भीतर�से�सभी��वरोध��को�रोक��दया
और�अं�तम�संघष��, " मेरे�जीवन�क��सबसे�बड़ी�लड़ाई�" के��लए�पाट��को�तैयार��कया।�
• भारत�छोड़ो�आंदोलन�क��77 व��वष�गांठ�8 अग�त�, 2019 को�मनाई�गई�थी।�हर�साल�8 अग�त�को�भारत�म��अग�त��ां�त��दवस�के
�प�म��मनाया�जाता�है।�

(Source: From Plassey to Partition: A History of Modern India )

QUESTION 19.
लाल�बहा�र�शा��ी�के�संदभ��म���न�न�ल�खत�कथन��पर��वचार�कर�:

1. वे�लाला�लाजपत�राय��ारा��था�पत�सव�ट्स�ऑफ�द�पीपुल�सोसाइट��(लोक�सेवक�मंडल) के�आजीवन�सद�य�बने।�
2. उ�ह�ने�असहयोग�आंदोलन�और�नमक�स�या�ह�म��भाग��लया।�
3. नेह��क��बीमारी�के�दौरान�, वह�भारत�के�र�ा�मं�ी�थे।�

ऊपर��दए�गए�कथन��म��से�कौन�सा�सही�है�/ ह��?

a) केवल�1 और�2
b) केवल�1 और�3
c) केवल�3
d) केवल�2 और�3
Correct Answer: A
Your Answer:
Explanation

Solution (a)

Basic Information:

• लाल�बहा�र�शा��ी�का�ज�म�2 अ�टू बर�, 1904 को�मुगलसराय�, वाराणसी�, उ�र��दे श�म���आ�था।�


• वे�महा�मा�गांधी�से�ब�त��भा�वत�थे।�
• लाल�बहा�र�शा��ी�वाराणसी�म��काशी��व�ा�पीठ�म��शा�मल�हो�गए�, जो����टश�शासन�क��अवहेलना�म���था�पत�कई�रा�ीय�सं�थान��म�
से�एक�है।�
• ' शा��ी�' �व�ा�पीठ��ारा�उ�ह���दान�क��गई��नातक�क��उपा�ध�थी�, ले�कन�लोग��के�मन�म��उनके�नाम�के�भाग�के��प�म��रह�गई।�
• उ�ह�ने�एक�चरखा�और�कुछ�गज�के�कपड़े�को�दहेज�के��प�म���वीकार��कया।�
• 1946 म��जब�कां�ेस�सरकार�बनी�, तो�उ�ह��अपने�गृह�रा�य�उ�र��दे श�म��संसद�य�स�चव��नयु���कया�गया�और�ज�द�ही�गृह�मं�ी�के
पद�पर�आसीन��ए।�
• उ�ह�ने�रेल�मं�ी�के��प�म��अपने�पद�से�इ�तीफा�दे ��दया��य��क�उ�ह�ने�एक�रेलवे��घ�टना�के��लए�उ�रदायी�महसूस��कया�था�, �जसम�
कई�लोग��क��जान�चली�गई�थी।�अभूतपूव��काय��को�संसद�और�दे श�ने�ब�त�सराहा�गया।�
• वह��वतं��भारत�के��सरे��धान�मं�ी�थे�, जो�नेह��के�उ�रा�धकारी�थे।�
• उ�ह�ने��ेत��ां�त�को�बढ़ावा��दया�और�" जय�जवान�जय��कसान�" नारा��दया�
• उ�ह��1966 म��मरणोपरांत�भारत�र�न�का�भारत�का�सव��च�नाग�रक�पुर�कार��दया�गया।�
• ताशकंद�म��उ�ह���दल�का�दौरा�पड़ा�और�उनका��नधन�हो�गया।�

कथन��व�ेषण:

कथन�1 कथन�2 कथन�3

IASbaba
Web: http://ilp.iasbaba.com/ Score:
Email: ilp@iasbaba.com 3.00 / 200
Page 124
AIPTS/ILP VETERANS-
Exam Title :
2020 TE...
Email : yadavanurag075@gmail.com
Contact : 8882839768

स�य� स�य� अस�य�

लाल�बहा�र�शा��ी�सोलह�वष��के�थे�जब
उ�ह�ने�क���य�मं��मंडल�म��कई��वभाग�
गांधी�जी�ने�अपने�दे शवा�सय��से�असहयोग
लोक�सेवा�मंडल�म��उ�ह�ने��पछड़े�वग��के को�संभाला�- रेल�मं�ी�; प�रवहन�और
आंदोलन�म��शा�मल�होने�का�आ�ान��कया।
उ�थान�के��लए�काम�करना�शु���कया�और संचार�मं�ी�; वा�ण�य�और�उ�ोग�मं�ी�;
उ�ह�ने��वयं�नमक-स�या�ह�म��भाग��लया
बाद�म��वे�उस�सोसाइट��के�अ�य��बने।� �ह�मं�ी�; और�पोट� फो�लयो�के��बना
और�कई�अ�भयान��का�नेतृ�व��कया�और
नेह��क��बीमारी�के�दौरान।�
���टश�जेल��म��कुल�सात�वष���बताए।�

(Source: https://www.pmindia.gov.in/en/former_pm/shri-lal-bahadur-shastri/ )

QUESTION 20.
होम��ल�आंदोलन�का�उ�े �य��या�था�?

a) ���टश�कानून��क��अव�ा�करना�तथा�शासन�करना�क�ठन�बना�दे ना।�
b) भारतीय��को�स�ा�का�पूण��ह�तांतरण।�
c) ���टश�सा�ा�य�के�भीतर�डो�म�नयन��टे टस��ा�त�करना।�
d) �वदे शी�व�तु��का�ब�ह�कार�
Correct Answer: C
Your Answer:
Explanation

Solution (c)

Basic Information:

• होम��ल�आंदोलन��थम��व��यु��के��लए�भारतीय���त��या�थी।�
• इं�डयन�होम��ल�लीग�का�आयोजन�आय�रश�होम��ल�लीग�क��तज��पर��कया�गया�था�तथा�उ�ह�ने�आ�ामक�राजनी�त�क��एक�नई
�वृ���के�उ�व�का���त�न�ध�व��कया।�
• एनी�बेस�ट�और��तलक�इस�नई��वृ���के�अ�णी�थे।�
• कारक�जैसे�- मोल�-�म�टो�सुधार�उदारवाद��का�मोहभंग�कर�रहे�थे�, लोग�यु��के��ख��के�बोझ�को�महसूस�कर�रहे�थे�, आ�द�ने�होम��ल
आंदोलन�के��लए�आधार�का�नेतृ�व��कया।�
• मूल�उ�े �य�थे�; �व-शासन�(होम��ल) क��रा�ीय�मांग�, और�उस�समय�ऑ��े �लया�, कनाडा�, द��ण�अ��का�, �यूजील�ड�और
�यूफ़ाउंडल�ड��ारा��ा�त����टश�सा�ा�य�के�भीतर�डो�म�नयन�का�दजा���ा�त�करना।�
• दो�होम��ल�लीग�क���थापना�क��गई�, एक�अ�ैल�1916 म��पुणे�म��बी.जी.�तलक��ारा�, जो�महारा��(बॉ�बे�शहर�को�छोड़कर) , कर्
नाटक�, म�य��ांत�और�बरार�तक�सी�मत�थी।��सतंबर�1916 म��म�ास�म��एनी�बेस�ट��ारा�अ�य�लीग�ने�शेष�भारत�(बॉ�बे�शहर�स�हत)
को�कवर��कया।�
• होम��ल�आंदोलन�म��बाद�म��मोतीलाल�नेह��, जवाहरलाल�नेह��, भूलाभाई�दे साई�, �चतरंजन�दास�, मदन�मोहन�मालवीय�, मोह�मद
अली��ज�ा�, तेज�बहा�र�स�ू�और�लाला�लाजपत�राय�शा�मल��ए।�

(Source: Spectrum)

QUESTION 21.
�न�न�ल�खत�म��से�कौन�सा�1939 म��सात��ांत��म��कां�ेस�के�मं��य��के�इ�तीफे�का�सही�कारण�है�?

a) दे श�म��बढ़ते�समाजवाद��और�सा�यवाद���भाव�ने�कां�ेस�के�मं��य��के��लए�काम�करना�मु��कल�कर��दया।�

IASbaba
Web: http://ilp.iasbaba.com/ Score:
Email: ilp@iasbaba.com 3.00 / 200
Page 125
AIPTS/ILP VETERANS-
Exam Title :
2020 TE...
Email : yadavanurag075@gmail.com
Contact : 8882839768

b) दे श�म���ापक�सां�दा�यक�तनाव।�
c) भारतीय��के�परामश��के��बना�वायसराय�के�भारत�को���तीय��व��यु��के��लए�पाट��घो�षत�करने�के��वरोध�म��
d) सं�वधान�सभा�का�गठन�करने�के�कां�ेस�के���ताव�को�अ�वीकार�करने�के�कारण।�
Correct Answer: C
Your Answer:
Explanation

Solution (c)

Basic Information:

• कां�ेस�ने�1161 म��से�716 सीट��पर�चुनाव�लड़ा�था।�बंगाल�, असम�, पंजाब�, �स�ध�और�उ�र�प��म�सीमांत��ा�त�को�छोड़कर


सभी��ांत��म��इसे�ब�मत��मला�तथा�बंगाल�, असम�और�उ�र�प��म�सीमांत��ा�त�म��सबसे�बड़ी�पाट��के��प�म��उभरी।�
• कां�ेस�के�मं�ालय�बॉ�बे�, म�ास�, म�य��ांत�, उड़ीसा�, संयु���ांत�, �बहार�और�बाद�म��उ�र�प��म�सीमांत��ा�त�और�असम�म��भी
बनाए�गए�थे।�
• 1 �सतंबर�, 1939 को: जम�नी�ने�पोल�ड�पर�आ�मण��कया�, उसके�बाद���तीय��व��यु��शु���आ।�
• 3 �सतंबर�, 1939 को: ��टे न�ने�जम�नी�के��खलाफ�यु��क��घोषणा�क��और�भारतीय�राय�से�परामश���कए��बना�यु��के��लए�भारत�के
समथ�न�क��घोषणा�क�।�
• कां�ेस�काय��स�म�त�( CWC) के���ताव�ने�फासीवाद��आ�मण�क���न�दा�क�।�ले�कन�यह�भी�तक���दया�गया��क: भारत�लोकतां��क
�वतं�ता�के��लए�यु��नह���कया�जा�सकता�है�, जब�क�भारत�को��वतं�ता�से�वं�चत��कया�जा�रहा�है�; य�द���टे न�लोकतं��और
�वतं�ता�के��लए�लड़�रहा�था�, तो�उसे�अपने�उप�नवेश��म��सा�ा�यवाद�को�समा�त�करने�और�भारत�म��पूण��लोकतं��क���थापना�करके
इसे�सा�बत�करना�चा�हए।�
• कां�ेस�नेतृ�व�वायसराय�और����टश�सरकार�को�हर�मौका�दे ना�चाहता�था।�
• सरकार�क����त��या�पूरी�तरह�से�नकारा�मक�थी।��लन�लथगो�ने�अपने�बयान�( 17 अ�टू बर�, 1939) म��मु��लम�लीग�और�कां�ेस
के��खलाफ��रयासत��का�इ�तेमाल�करने�क��को�शश�क�।�
• 23 अ�टू बर�, 1939 को�, कां�ेस�काय��स�म�त�क��बैठक�ने�यु��का�समथ�न�नह��करने�का�फैसला��कया�, तथा�कां�ेस�के�मं�ालय�
से��ांत��म��इ�तीफा�दे ने�का�आ�ान��कया�

(Source: Spectrum)

QUESTION 22.
ई�र�चं���व�ासागर�के�संदभ��म���न�न�ल�खत�कथन��पर��वचार�कर�:

1. उ�ह�ने�सं�कृत��वभाग�म���मुख�पं�डत�के��प�म��फोट� ��व�लयम�कॉलेज�म���वेश��लया।�
2. उ�ह��1851 म��सं�कृत�कॉलेज�का�����सपल��नयु���कया�गया�था।�
3. उनका��मुख�योगदान�1829 म��सती��था�को�समा�त�करने�वाले�कानून�को�लागू�करने�के��लए����टश�सरकार�को�राजी�करना�था।�

ऊपर��दए�गए�कथन��म��से�कौन�सा�सही�है�/ ह��?

a) केवल�1 और�2
b) केवल�2 और�3
c) केवल�1 और�3
d) 1, 2 और�3
Correct Answer: A
Your Answer:
Explanation

Solution (a)

IASbaba
Web: http://ilp.iasbaba.com/ Score:
Email: ilp@iasbaba.com 3.00 / 200
Page 126
AIPTS/ILP VETERANS-
Exam Title :
2020 TE...
Email : yadavanurag075@gmail.com
Contact : 8882839768

Basic Information:

• ई�र�चं��बंदोपा�याय�का�ज�म�26 �सतंबर�, 1820 को�बंगाल�म���मदनापुर��जले�के��बर�स�घा�गाँव�म���आ�था।�


• उ�ह�ने�1829 से�1841 के�दौरान�सं�कृत�कॉलेज�म��वेदांत�, �ाकरण�, सा�ह�य�, �मृ�त�और�नै�तकता�सीखी।�
• उ�ह�ने�1839 म��सं�कृत�म��एक���तयो�गता�परी�ण��ान�म��भाग��लया�तथा��ान�के�महासागर�' �व�ासागर�' का�शीष�क�अ�ज�त
�कया।�उसी�वष��ई�र�चं���व�ासागर�ने�सफलतापूव�क�अपनी��व�ध�परी�ा�उ�ीण��क�।�
• उ�ह�ने�दो�पु�तक��‘ उप�मो�नका�’ और�‘ �ाकरण�कौमुद��’ �लख��, आसान�सुग�य�बंगाली�भाषा�म��सं�कृत��ाकरण�क��ज�टल
धारणा��क���ा�या�क�।�
• वे�नारी��श�ा�के��बल�प�धर�थे।�उ�ह�ने�7 मई�, 1849 को�भारत�म��बे�यून��कूल�के�पहले��थायी�लड़�कय��के��कूल�क���थापना�के
�लए�जॉन�इ�लयट����कवाटर�बे�यून�का�समथ�न��कया।�
• उनक���थायी��वरासत�बंगाली�वण�माला�सीखने�के��लए��ाथ�मक��तर�क��पु�तक�‘ बोन��पो�रचोय�’ के�साथ�बनी��ई�है।�
• उ�ह�ने�ब��ववाह�के��खलाफ�अ�भयान�चलाया।�
• 14 अ�टू बर�, 1855 को�, �व�ासागर�ने�भारत�सरकार�से��ह����वधवा��के��ववाह�म��आने�वाली�सभी�बाधा��को��र�करने�तथा
ऐसे�सभी��ववाह��के�मु�े�को�वैध�घो�षत�करने�के��लए�एक�कानून�पा�रत�करने�क���ाथ�ना�क�।�

कथन��व�ेषण:

कथन�1 कथन�2 कथन�3

स�य� स�य� अस�य�

वह�कॉलेज�के�अ�धका�रय��के�अनुरोध�पर राजाराम�मोहन�रॉय�ने�18
सं�कृत�कॉलेज�म���ोफेसर�बन�गए�ले�कन�एक 29 म��सती��था�को
शत��लगा�द���क�उ�ह���णाली�को��फर�से समा�त�करने�वाले�कानून
1841 म��, इ�क�स�साल�क��उ��म��, ई�र �डज़ाइन�करने�क��अनुम�त�द��जाए।�वह�18 को�पा�रत�करने�म�
चं��ने�सं�कृत��वभाग�म���मुख�पं�डत�के��प 51 म��सं�कृत�कॉलेज�के��धानाचाय��बने।�1 मह�वपूण��भू�मका��नभाई।
म��फोट� ��व�लयम�कॉलेज�म���वेश��लया।� 855 म��, उ�ह�ने�अ�त�र���भार�वाले��कूल� �व�ासागर�ने��वधवा��के
के��वशेष��नरी�क�के��प�म���ज�मेदा�रय��को पुन�व�वाह�अ�ध�नयम�,
संभाला�तथा��श�ा�क��गुणव�ा�क��दे खरेख 1856 म���मुख�भू�मका
के��लए�बंगाल�के�सु�र�गांव��क��या�ा�क�।� �नभाई�

(Source: https://indianexpress.com/article/explained/vidyasagar-history-of-indian-social-reform-
kolkata-violence-5729699/ )

QUESTION 23.
भारतीय��वतं�ता�सं�ाम�क��अव�ध�के�संदभ��म��, नेह���रपोट� �म���न�न�ल�खत�म��से��कसक���सफा�रश�क��गई�थी�?

1. भारत�के��लए�पूण���वतं�ता�
2. अ�पसं�यक��के��लए�सीट��के�आर�ण�के��लए�संयु���नवा�चन।�
3. सं�वधान�म��भारत�के�लोग��के��लए�मौ�लक�अ�धकार��का��ावधान।�

ऊपर��दए�गए�कथन��म��से�कौन�सा�सही�है�/ ह��?

a) केवल�2

IASbaba
Web: http://ilp.iasbaba.com/ Score:
Email: ilp@iasbaba.com 3.00 / 200
Page 127
AIPTS/ILP VETERANS-
Exam Title :
2020 TE...
Email : yadavanurag075@gmail.com
Contact : 8882839768

b) केवल�2 और�3
c) केवल�3
d) 1, 2 और�3
Correct Answer: B
Your Answer:
Explanation

Solution (b)

Basic Information:

• 1927 म����टे न�क��कंजव��टव�सरकार�ने�भारतीय�सां�व�धक�आयोग�क���नयु���क��, �जसे�साइमन�कमीशन�के�नाम�से�जाना�जाता


है।�
• �कसी�भी�भारतीय�को�एक�ऐसे��नकाय�क��सेवा�के�लायक�नह��माना�गया��जसने�भारत�के�राजनी�तक�भ�व�य�का�फैसला�करने�के
अ�धकार�का�दावा��कया।�
• लॉड��बीरकेनहेड�, कंज़व��टव�से�ेटरी�ऑफ��टे ट�ने�संवैधा�नक�सुधार��क��एक�ठोस�योजना�तैयार�करने�के��लए�भारतीय��क���मता�पर
���उठाया।�
• एक���त��या�के��प�म��, फरवरी�, मई�और�अग�त�1928 म��सभी�दल��का�स�मेलन�आयो�जत��कया�गया�था�ता�क�एक�योजना�को
अं�तम��प��दया�जा�सके�, �जसे�लोक��य�लेखक�मोतीलाल�नेह��के�नाम�पर�नेह���रपोट� �के��प�म��जाना�जाने�लगा।�
• नेह���रपोट� �( 1928) संवैधा�नक�योजना�का�मसौदा�तैयार�करने�का�पहला�भारतीय��यास�था।�
• मु��लम�लीग�और�यहां�तक��क��ज�ा�भी�मु��लम�ब�ल��ांत��म��मुसलमान��के��लए�सीट��के�आर�ण�क��मांग�नह��छोड़ा।�
• आगे�कोई��रयायत�नह��होने�के�कारण�, �ज�ा�ने��रपोट� �के��लए�अपना�समथ�न�वापस�ले��लया�तथा�अपने���स��’ चौदह�सू��’ का
��ताव�करने�के��लए�आगे�बढ़�गए।�
• अ�य��सफा�रश��म��शा�मल�ह�: क���के�साथ�अव�श��श��य��वाली�सरकार�का�एक�संघीय��प�, मं�ालय��वधा�यका�के���त�उ�रदायी
होगा�, एक�सु�ीम�कोट� �के��नमा�ण�के��लए�एक���ताव�, �ांत��को�भाषाई�आधार�के�साथ�बनाया�जाएगा�, आ�द।�

कथन��व�ेषण:

कथन�1 कथन�2 कथन�3

अस�य� स�य� स�य�

इसने�अलग�सां�दा�यक�मतदाता��के
19 मौ�लक�अ�धकार��जैसे��क��रपोट� �म�
�स�ांत�को�भी�खा�रज�कर��दया�, �जस
साव�भौ�मक�वय�क�मता�धकार�, म�हला�
इस��रपोट� �ने�डो�म�नयन��टे टस�को�भारत पर��पछले�संवैधा�नक�सुधार�आधा�रत�थे।
के��लए�समान�अ�धकार�, यू�नयन��के�गठन
�ारा�वां�छत�सरकार�के��प�म��प�रभा�षत क���और��ांत��म��सीट� �मु��लम��के��लए
क���वतं�ता�और��कसी�भी��प�म��धम��से
�कया।� आर��त�ह�गी�, �जसम��वे�अ�पसं�यक�थे
रा�य�के�पृथ�करण�आ�द�क���सफा�रश�क�
, ले�कन�उन��थान��म��नह��, जहां�उनके
गई�थी।�
पास�सं�या�मक�ब�मत�था।�

(Source: India’s Struggle for Independence)

QUESTION 24.

IASbaba
Web: http://ilp.iasbaba.com/ Score:
Email: ilp@iasbaba.com 3.00 / 200
Page 128
AIPTS/ILP VETERANS-
Exam Title :
2020 TE...
Email : yadavanurag075@gmail.com
Contact : 8882839768

�न�न�ल�खत�कथन��पर��वचार�कर�:

1. 1920 म��, �व�लभाई�पटे ल�, महा�मा�गांधी�और�बाल�गंगाधर��तलक�स�हत�कई��मुख��वतं�ता�सेना�नय��ने�जेल�से�उनक���रहाई�क�


मांग�क�।�
2. उ�ह�ने�‘ �ह���व: �ह���कौन�है�?’ पु�तक��लखी�
3. उनके�नेतृ�व�म��, �ह���महासभा�ने�मु��लम�लीग�के�साथ��मलकर��स�ध�, बंगाल�और�उ�र�प��म�सीमा��ांत�जैसे��ांत��म��सरकार�बनाई।�

भारत�के��वतं�ता�सं�ाम�म���न�न�ल�खत�म��से�कौन�सा�रा�वाद��नेता�उपरो��कथन��से�संब��है�?

a) केशव�ब�लराम�हेडगेवार�
b) मदन�मोहन�मालवीय�
c) लाला�लाजपत�राय�
d) �वनायक�दामोदर�सावरकर�
Correct Answer: D
Your Answer:
Explanation

Solution (d)

Basic Information:

• �वनायक�दामोदर�सावरकर�उफ���वातं�यवीर�सावरकर�को��ह���रा�वाद�के��पता�के��प�म��जाना�जाता�है�, ले�कन�यह�सामा�जक�और
धा�म�क�सुधारक��क��महारा��क��गौरवशाली�परंपरा�का�भी���त�न�ध�व�करता�है।�
• सावरकर�ने�एक�हाई��कूल�के�छा��के��प�म��अपनी�राजनी�तक�ग�त�व�धय��क��शु�आत�क��और�पुणे�के�फ�यू�सन�कॉलेज�म��ऐसा
करना�जारी�रखा।�
• उ�ह�ने�और�उनके�भाई�ने�अ�भनव�भारत�सोसायट��नामक�एक�गु�त�समाज�क���थापना�क�।�
• वे�अपने�कानून�अ�ययन�के��लए�यूनाइटे ड��क�गडम�गए�, उ�ह�ने�खुद�को�इं�डया�हाउस�और����इं�डया�सोसाइट��जैसे�संगठन��से�जोड़ा।
• उन�पर�ना�सक�कले�टर�जै�सन�क��ह�या�के�आरोप�म��और�राजा�स�ाट�के��खलाफ�भारतीय�दं ड�सं�हता�121- ए�के�तहत�एक
सा�जश�रचने�का�आरोप�लगाया�गया�था।�
• 28 साल�क��उ��म��, उ�ह��दोषी�ठहराया�गया�और�50 साल�के�कारावास�क��सजा�सुनाई�गई�तथा�4 जुलाई�1911 को�अंडमान�और
�नकोबार���प�समूह�म��कु�यात�सेलुलर�जेल�म��ले�जाया�गया।�
• अपनी�पु�तक�' द��ह����ऑफ�द�वॉर�ऑफ�इं�डयन�इं�डप�ड�स�' म��, अं�ेज��के��खलाफ�आजाद��के��लए�भारत�के�पहले�यु��का
आ�ान�करने�वाले�पहले�लेखक��म��से�एक�बने।�
• �ह���व�के�एक�भावुक��वत�क�होने�के�नाते�भी�, वह�कभी�भी�गाय�के�उपासक�नह��थे।�
• जब�कां�ेस�ने�1942 म��भारत�छोड़ो�आंदोलन�शु���कया�, तो�सावरकर�ने�इसक��आलोचना�क��और��ह����से�यु��के��यास�म�
स��य�रहने�और�सरकार�क��अव�ा�नह��करने�को�कहा।�
• 1964 म��, उ�ह�ने�महसूस��कया��क�भारत�ने��वतं�ता�के�अपने�ल�य�को��ा�त��कया�है�और�उ�ह�ने�समा�ध��ा�त�करने�क��अपनी
इ�छा�क��घोषणा�क�।�उ�ह�ने�1 फरवरी�, 1966 को�भूख�हड़ताल�शु��क��और�26 फरवरी�, 1966 को�उनका��नधन�हो�गया।�

(Source: Wikipedia)

QUESTION 25.
भले�ही�1857 के��व�ोह�क��पूव�-योजना�पर�संदेह�हो�, ले�कन��व�ोह�शु��होते�ही�योजना�बनाने�के��माण��मलते�ह�।��व�ोह�शु��होने�के�बाद
�व�ो�हय���ारा��कए�गए��यास��क��योजना��न�न�म��से�कौन�सी�है�?

1. सभी�पड़ोसी�रा�य��के�शासक��को�एक�प��संबो�धत��कया�गया�, �जसम��उनका�समथ�न�करने�और�उ�ह��भाग�लेने�के��लए�आमं��त
करने�के��लए�कहा�गया।�
2. �द�ली�म��, �शासक��क��एक�अदालत��था�पत�क��गई�, जो�रा�य�के�सभी�मामल��के��लए�उ�रदायी�थी।�
3. आंदोलन�को�वैधता��दान�करने�के��लए��स�के�मुगल�स�ाट�के�नाम�पर�जारी��कए�गए�तथा�आदे श�जारी��कए�गए।�

सही�कूट�का�चयन�कर�:

IASbaba
Web: http://ilp.iasbaba.com/ Score:
Email: ilp@iasbaba.com 3.00 / 200
Page 129
AIPTS/ILP VETERANS-
Exam Title :
2020 TE...
Email : yadavanurag075@gmail.com
Contact : 8882839768

a) केवल�2
b) केवल�2 और�3
c) केवल�3
d) 1, 2 और�3
Correct Answer: D
Your Answer:
Explanation

Solution (d)

Basic Information:

• �या��व�ोह�एक�संग�ठत�और��व�धपूव�क��नयो�जत��व�ोह�या�एक�सहज��व�ोह�था�, अभी�भी�बहस�का��वषय�है।�
• �व�ो�हय���ारा�पीछे �छोड़��दए�गए��कसी�भी��व�सनीय�सा�य�क��अनुप��थ�त�ने��म�को��र�करना�मु��कल�बना��दया।�
• जब��सपाही�मेरठ�से�प�ंचे�, तो�लगता�है��क�बहा�र�शाह�आ�य�च�कत�हो�गए�और�उ�ह�ने�तुरंत�आगरा�के�ले��टन�ट�गवनर�को�खबर
प�ंचा�द�।�
• झांसी�क��रानी�ल�मीभाई��ज�ह�ने��व�ो�हय��के�साथ�खुले�तौर�पर�शा�मल�होने�से�पहले�, कुछ�समय��लया�था।�
• चपा�तय��और�कमल�के�फूल��के��चलन�से����संदेश�भी�अ�न��त�है।�
• ले�कन�यहां�तक��क�अगर��व�ोह�से�पहले�कोई�योजना�और�संगठन�नह��था�, तो�यह�मह�वपूण��था��क�यह�, एक�बार�शु��होने�के�बाद
�कया�गया�था�
• �द�ली�पर�क�जा�करने�के�तुरंत�बाद�सभी�पड़ोसी�रा�य��और�राज�थान�के�शासक��को�एक�प��संबो�धत��कया�गया��जसम��उ�ह��अपना
समथ�न�दे ने�और�उ�ह��भाग�लेने�के��लए�आमं��त��कया।�
• �द�ली�म��, �शासक��क��एक�अदालत��था�पत�क��गई�जो�रा�य�के�सभी�मामल��के��लए��ज�मेदार�थी।�अदालत�म��दस�सद�य�, सेना
के�छह�और�नाग�रक��वभाग��के�चार�शा�मल�थे।�सभी��नण�य�ब�मत�के�मत��ारा��लए�गए�थे।�
• अदालत�ने�स�ाट�के�नाम�पर�रा�य�के�मामल��का�संचालन��कया।�
• खान�बहा�र�खान�ने�मुगल�स�ाट�के�नाम�पर��शासन�का�संचालन��कया।�

(Source: India’s Struggle for Independence)

QUESTION 26.
�न�न�ल�खत�म��से��कस�राजनी�तक�दाश��नक�ने�अपने�महान�काय��‘ ए��ह����ऑफ����टश�इं�डया�’ म��भारतीय�इ�तहास�को�तीन�अव�धय�- �ह��
, मु��लम�और����टश�म���वभा�जत��कया�है�?

a) थॉमस�ब�ब�गटन�मैकाले�
b) जेरेमी�ब�थम�
c) जे�स��मल�
d) जॉन��टु अट� ��मल�
Correct Answer: C
Your Answer:
Explanation

Solution (c)

Basic Information:

• 1817 म��, �कॉ�टश�अथ�शा��ी�और�राजनी�तक�दाश��नक�, जे�स��मल�, ने�एक��वशाल�तीन-वॉ�यूम�का�काय��, ए��ह����ऑफ़����टश


इं�डया��का�शत��कया।�इसम��उ�ह�ने�भारतीय�इ�तहास�को�तीन�कालखंड��म���वभा�जत��कया- �ह���, मु��लम�और����टश।�इस�काम�के
आधार�पर�, �म�स�ने�शाही�ई�ट�इं�डया�कंपनी�म��एक�परी�क�के�पद�को�हा�सल��कया�
• �मल�ने�सोचा��क�सभी�ए�शयाई�समाज�यूरोप�क��तुलना�म��स�यता�के��नचले��तर�पर�थे।�उनके�इ�तहास�के�अनुसार�, अं�ेज��के�भारत
आने�से�पहले�, दे श�म���ह���और�मु��लम��नरंकुश�शासक�थे।�

IASbaba
Web: http://ilp.iasbaba.com/ Score:
Email: ilp@iasbaba.com 3.00 / 200
Page 130
AIPTS/ILP VETERANS-
Exam Title :
2020 TE...
Email : yadavanurag075@gmail.com
Contact : 8882839768

• �मल�को�लगा��क����टश�शासन�भारत�को�स�य�बना�सकता�है।�
• �मल�ने�, वा�तव�म��, सुझाव��दया��क����टश�लोग��को�भारत�म��सभी��े���को�जीतना�चा�हए�ता�क�भारतीय�लोग��के��ान�और�खुशी
को�सु�न��त��कया�जा�सके।����टश�मदद�के��बना�भारत��ग�त�के��लए�स�म�नह��था।�
• इ�तहासकार��ने�भारतीय�इ�तहास�को�' �ाचीन�', ' म�यकालीन�' और�' आधु�नक�' म���वभा�जत��कया�है।�

(Source: Our Past – III Part – 1, Class VIII NCERT)

QUESTION 27.
1600 म��, ई�ट�इं�डया�कंपनी�ने�इं�ल�ड�क��शासक�, महारानी�ए�लजाबेथ�I से�एक�चाट� र��ा�त��कया�, �जससे�उसे�पूव��के�साथ��ापार�करने
का�पूरा�एका�धकार��मल�गया।�यह�एका�धकार�पूरी�तरह�से��न�न�ल�खत�म��से��कस����टश�अ�ध�नयम��ारा�वापस��लया�गया�था�?

a) 1773 का�चाट� र�ए�ट�


b) 1813 का�चाट� र�ए�ट�
c) 1833 का�चाट� र�ए�ट�
d) 1853 का�चाट� र�ए�ट�
Correct Answer: C
Your Answer:
Explanation

Solution (c)

Basic Information:

• 1833 के�चाट� र�ए�ट�ने�वा�ण��यक��नकाय�के��प�म��ई�ट�इं�डया�कंपनी�क��ग�त�व�धय��को�समा�त�कर��दया�और�इसे��वशु���प�से


�शास�नक��नकाय�बना��दया।�
• 1813 के�चाट� र�ए�ट�ने�भारत�म��ई�ट�इं�डया�कंपनी�के�एका�धकार�को�समा�त�कर��दया�, हालाँ�क�कंपनी�का�चीन�के�साथ��ापार
और�चाय�म���ापार�का�एका�धकार�था।�
• इसके�तहत�कंपनी�ने�चीन�के�साथ�अपना��ापार�और�चाय�म���ापार�का�एका�धकार�खो��दया।�
• 1833 के�चाट� र�ए�ट�ने�ई�ट�इं�डया�कंपनी�को�बीस�वष��क��अव�ध�के��लए�एक�और�प�ा��दया�जो��क�भारतीय��े���को�" महाम�हम
महाम�हम�, और�उ�रा�धका�रय��के��लए��व�ास�म��" था।�
• इसने�भारतीय��शासन�को�क���कृत��कया�और�बंगाल�के�गवन�र-जनरल�को�भारत�का�गवन�र-जनरल�ना�मत��कया।�इस��कार�, लॉड�
�व�लयम�ब��टक�" ���टश�भारत�के�पहले�गवन�र�जनरल�" बन�गये।�
• इसने�बॉ�बे�और�म�ास�के�गवन�र��को�उनक���वधायी�श��य��से�वं�चत�कर��दया।�
• अ�ध�नयम�ने�काउं�सल�के�गवन�र�जनरल�को�भारत�म��दासता�क����थ�त�को�कम�करने�के�उपाय��को�अपनाने�का��नद� श��दया।�

(Source: Our Past – III Part – 1, Class VIII NCERT)

QUESTION 28.
अवध�के�नवाब�सआदत�अली�खान�को�1801 म��अपने��े��का�आधा��ह�सा�कंपनी�को�दे ने�के��लए�मजबूर�होना�पड़ा��य��क:

a) सहायक�बल��( subsidiary forces) के��लए�भुगतान�करने�म��उनक���वफलता�, �जसे�कंपनी�संर�ण�के�उ�े �य�से�बनाए�रखने


वाली�थी।�
b) अवध�के�नवाब�पर�कुशासन�का�आरोप।�
c) अं�ेज��के���त�छल�और�कपट�का�आरोप।�
d) इनम��से�कोई�भी�नह�।�
Correct Answer: A
Your Answer:
Explanation

Solution (a)

IASbaba
Web: http://ilp.iasbaba.com/ Score:
Email: ilp@iasbaba.com 3.00 / 200
Page 131
AIPTS/ILP VETERANS-
Exam Title :
2020 TE...
Email : yadavanurag075@gmail.com
Contact : 8882839768

Basic Information:

• कंपनी�ने�रा�य��को�" सहायक�गठबंधन�" के��लए�मजबूर��कया।�इस�गठबंधन�क��शत��के�अनुसार�, भारतीय�शासक��को�अपने��वतं�


सश���बल�रखने�क��अनुम�त�नह��थी।�
• उ�ह��कंपनी��ारा�संर��त��कया�जाना�था�, ले�कन�कंपनी�को�" सहायक�बल��" के��लए�भुगतान�करना�था�जो�कंपनी�को�इस�संर�ण
के�उ�े �य�के��लए�बनाए�रखना�था।�
• य�द�भारतीय�शासक�भुगतान�करने�म��असफल�रहे�, तो�उनके��े��का��ह�सा�दं ड�के��प�म��ले��लया�गया।�
• उदाहरण�के��लए�, जब��रचड��वेले�ली�गवन�र�जनरल�( 1798-1805) थे�, तो�अवध�के�नवाब�को�1801 म��अपने��े��का�आधा
�ह�सा�कंपनी�को�दे ने�के��लए�मजबूर�होना�पड़ा�, �य��क�वे�" सहायक�बल��" के��लए�भुगतान�करने�म���वफल�रहे�थे।�
• हैदराबाद�को�भी�इसी�तरह�के�आधार�पर�उ�री�सरकार�के�इलाक��को�दे ने�के��लए�मजबूर��कया�गया�था।�
• 1856 म��, लॉड��डलहौज़ी�ने�कुशासन�के�आरोप��के�तहत�अवध�के�शेष�आधे��ह�से�को�अ�ध��हत�कर��लया।�वा�जद�अली�शाह�, तत्
कालीन�नवाब�को�कैद�कर��लया�गया�था�, और��फर�कंपनी��ारा�कलक�ा�(बंगाल) म���नवा��सत�कर��दया�गया�था।�

(Source: Our Past – III Part – 1, Class VIII NCERT)

QUESTION 29.
�न�न�ल�खत�म��से��कसने�तीन��गोलमेज�स�मेलन�म��भाग��लया�है:

1. मदन�मोहन�मालवीय�
2. बी�आर�अ�बेडकर�
3. महा�मा�गांधी�
4. तेज�बहा�र�स�ू�

सही�कूट�का�चयन�कर�:

a) केवल�1 और�3
b) केवल�2 और�3
c) केवल�2 और�4
d) केवल�1, 2 और�4
Correct Answer: C
Your Answer:
Explanation

Solution (c)

Basic Information:

• तेज�बहा�र�स�ू�और�डॉ. बी. आर. अ�बेडकर�, बेगम�जहाँआरा�शाहनवाज़�(म�हला���त�न�ध�व) , यहाँ�तक��क�मु��लम�लीग�के�नेता


आगा�खान�III ने�तीन��गोलमेज�स�मेलन��म��भाग��लया।�
• लंदन�म��रॉयल�गैलरी�हाउस�ऑफ�लॉड् �स�म��12 नवंबर�, 1930 को�महाम�हम�जॉज��पंचम��ारा�आ�धका�रक�तौर�पर��थम�गोलमेज
स�मेलन�का�उ�ाटन��कया�गया�था�तथा�इसक��अ�य�ता��धान�मं�ी�रामसे�मैकडोना�ड�ने�क��थी।�इसम��आगा�खान�III, �ज�ा�, आ
�द�जैसे�मु��लम�लीग�के�सद�य��ने�भाग��लया�; तेज�बहा�र�स�ू��ारा���तुत�उदारवाद��; बी. आर. अ�बेडकर��ारा�द�लत�वग��; और
अ�य।�
• 7 �सतंबर�, 1931 को��सरा�गोलमेज�स�मेलन��आ�, �जसम��एम�के�गांधी�ने�कां�ेस�के�एकमा����त�न�ध�के��प�म��भाग��लया�; तेज
बहा�र�स�ू��ारा���तुत�उदारवाद��; बी. आर. अ�बेडकर��ारा�शो�षत�वग��, मु��लम�लीग�के�नेता�आगा�खान�III; मदन�मोहन
मालवीय�आ�द।�
• 17 नवंबर�, 1932 को�इक�े ��ए�तीसरे�गोलमेज़�स�मेलन�म��आगा�खान�तृतीय�, बी. आर. अ�बेडकर�(द�लत�वग�) , बेगम�जहाँआरा
शाहनवाज़�(म�हला) , ए. पी. पा�ो�, तेज�बहा�र�स�ू�, आ�द�ने�भाग��लया।�

(Source: Wikipedia)

IASbaba
Web: http://ilp.iasbaba.com/ Score:
Email: ilp@iasbaba.com 3.00 / 200
Page 132
AIPTS/ILP VETERANS-
Exam Title :
2020 TE...
Email : yadavanurag075@gmail.com
Contact : 8882839768

QUESTION 30.
���टश���ारा�राज�व�के��नय�मत��वाह�को�सु�न��त�करने�के��लए�1793 म���थायी�बंदोब�त�शु���कया�गया�था�, ले�कन�इससे�कई�सम�याएं
पैदा���।�इस�संदभ��म���न�न�ल�खत�कथन��पर��वचार�कर�:

1. ज़म�दार��ने�भू�म�के�उ�पादन�म��वृ���से�लाभ�के��लए�भू�म�सुधार�म��भारी��नवेश��कया।�
2. �कसान��ारा�लगान�का�भुगतान�न�करने�क����थ�त�म��उसे�जमीन�से�बेदखल�कर��दया�गया।�
3. ज़म�दार�जो�अं�ेज��को�राज�व�दे ने�म��असफल�रहे�, उ�ह�ने�अपनी�जम�दारी�अ�धकार��को�खो��दया।�

ऊपर��दए�गए�कथन��म��से�कौन�सा�सही��प�से��थायी�बंदोब�त��ारा�बनाई�गई�सम�या��को�उजागर�करता�है�?

a) केवल�2
b) केवल�2 और�3
c) केवल�1 और�3
d) केवल�1, 2 और�3
Correct Answer: B
Your Answer:
Explanation

Solution (b)

Basic Information:

• लॉड��कॉन�वॉ�लस��ारा�1793 म���थायी�बंदोब�त�आरंभ��कया�गया�था।�
• राजा��और�तालुकदार��को�जम�दार��के��प�म��मा�यता�द��गई�थी�तथा�वे��कसान��से��कराए�लेने�और�कंपनी�को�राज�व�दे ने�के��लए
उ�रदायी�थे।�
• भुगतान�क��जाने�वाली�रा�श��थायी��प�से�तय�क��गई�थी।�
• यह�महसूस��कया�गया��क�इससे�कंपनी�के�खजाने�म��राज�व�का��नय�मत��वाह�सु�न��त�होगा�तथा�साथ�ही�, ज़म�दार��को�भू�म�म�
सुधार�के��लए��नवेश�करने�के��लए��ो�सा�हत��कया�जाएगा।�
• ले�कन�ज�द�ही�कंपनी�के�अ�धका�रय��को�बंदोब�त��ारा�बनाई�गई�सम�या��का�एहसास��आ�

कथन��व�ेषण:

कथन�1 कथन�2 कथन�3

अस�य� स�य� स�य�

गाँव��, का�तकार��ने��व�था�को�अ�यंत
�थायी��प�से�तय��कया�गया�राज�व
दमनकारी�पाया।�भुगतान��कया�जाने�वाला
इतना�अ�धक�था��क�जम�दार��को�भुगतान
ज़म�दार�भू�म�सुधार�म���नवेश�नह��कर �कराया�अ�धक�था�और�भू�म�पर�उसका
करना�मु��कल�हो�गया।�इस�लए�राज�व
रहे�थे।� अ�धकार�असुर��त�था।�ऋण�चुकाने�म��बाधा
का�भुगतान�करने�म���वफलता�के�कारण
, उ�च��कराए�के�कारण��कराए�का�भुगतान
जम�दारी�का�नुकसान��आ�
नह���आ�और�भू�म�से�बेदखल�कर��दया�गया�

(Source: Our Past – III Part – 1, Class VIII NCERT)

IASbaba
Web: http://ilp.iasbaba.com/ Score:
Email: ilp@iasbaba.com 3.00 / 200
Page 133
AIPTS/ILP VETERANS-
Exam Title :
2020 TE...
Email : yadavanurag075@gmail.com
Contact : 8882839768

QUESTION 31.
�न�न�ल�खत�म��से�कौन�सा�कथन����टश�भारत�के�तहत�नील�क��खेती�क��मु�य��णा�लय��के�बारे�म��सही�है�/ ह��?

1. ‘ �नज�’ (nij) �णाली�के�तहत�, �लांटर�ने�उन�जमीन��म��नील�का�उ�पादन��कया��जसे�वे�सीधे��नयं��त�करते�थे।�


2. �लांटर��ने�एक�अनुबंध�पर�ह�ता�र��कया�, एक�समझौते�(स�ा) , �जसम��रैयत��के�साथ�‘ रयो�त�’ (ryoti) �णाली�के�तहत�नील�क�
खेती�क��जाती�थी।�

सही�कूट�का�चयन�कर�:

a) केवल�1
b) केवल�2
c) 1 और�2 दोन��
d) न�तो�1 और�न�ही�2
Correct Answer: C
Your Answer:
Explanation

Solution (c)

Basic Information:

• यूरोप�म��नील�क��बढ़ती�मांग�का�सामना�करते��ए�, भारत�म��कंपनी�ने�नील�क��खेती�के�तहत��े��का��व�तार�करने�के�तरीक��क�
तलाश�क�।�
• उ�च�लाभ�क��संभावना�से�आक�ष�त�, कई��कॉट् समैन�और�अं�ेज�भारत�आए�और��लांटस��बन�गए।�
• नील�क��खेती�क��दो�मु�य��णा�लयाँ�थ��- �नज�( nij) और�रैयती�( ryoti) ।
• �नज�खेती�क���णाली�के�भीतर�, �लांटस��ने�उन�जमीन��म��नील�का�उ�पादन��कया��जसे�उ�ह�ने�सीधे��नयं��त��कया�था।�उसने�या�तो
ज़मीन�खरीद��या�उसे�अ�य�ज़म�दार��से��कराए�पर��लया�और�सीधे�तौर�पर��कराए�के�मज़�र��को��नयु��करके�नील�का�उ�पादन
�कया।�
• रैयती��णाली�के�तहत�, �लांटस��ने�रैयत��को�एक�अनुबंध�, एक�समझौते�(स�ा) पर�ह�ता�र�करने�के��लए�मजबूर��कया।�कई�बार
उ�ह�ने��ाम��धान��पर�अनुबंध�पर�ह�ता�र�करने�के��लए�दबाव�डाला।�अनुबंध�पर�ह�ता�र�करने�वाल��को�नील�का�उ�पादन�करने�के
�लए�कम��याज�दर��पर��लांटस��से�नकद�अ��म��मला।�

(Source: Our Past – III Part – 1, Class VIII NCERT)

QUESTION 32.
नील��व�ोह�( 1859-61) के�संदभ��म���न�न�ल�खत�कथन��पर��वचार�कर�:

1. �कसान��को��तनक�ठया��णाली�के�तहत�कुल�भू�म�के�3/20 �ह�से�पर�नील�उगाने�के��लए�मजबूर��कया�गया�था।�
2. नील�आयोग�, सरकार�ने�नील�उ�पादन�क���णाली�क��जांच�के��लए��था�पत��कया�, �जसने�नील�का�तकार��के�साथ�जबरद�ती�के
तरीक��का�उपयोग�करने�के��लए��लांटस��क��आलोचना�क�।�
3. नील��व�ोह�के�बाद��लांटस��ने�अपने�नील�के�प�रचालन�को��बहार�से�बंगाल��थानांत�रत�कर��दया�

ऊपर��दए�गए�कथन��म��से�कौन�सा�गलत�है�/ ह��?

a) केवल�1
b) केवल�2 और�3
c) केवल�1 और�3
d) 1, 2 और�3
Correct Answer: C
Your Answer:

IASbaba
Web: http://ilp.iasbaba.com/ Score:
Email: ilp@iasbaba.com 3.00 / 200
Page 134
AIPTS/ILP VETERANS-
Exam Title :
2020 TE...
Email : yadavanurag075@gmail.com
Contact : 8882839768

Explanation

Solution (c)

Basic Information:

• 1859-60 म��बंगाल�म��नील��व�ोह�(नील��बदरोहो) �आ�था�तथा����टश��लांटस��के��खलाफ��व�ोह�था�, �जसने�उ�ह��उन�शत��के


तहत�नील�उगाने�के��लए�मजबूर��कया�था�जो��कसान��के��लए�ब�त���तकूल�थी।�
• �कसान��को�इस�उ�े �य�के��लए�ऋण��दया�गया�था।�एक�बार�जब��कसान��ने�कज���लया�, तो�वे��याज�क��उ�च�दर��के�कारण�इसे�कभी
नह��चुका�सकते�थे।�
• सरकार�ने�हमेशा��लांटस��का�समथ�न��कया�, �ज�ह�ने�कई��वशेषा�धकार�और��या�यक���तर�ा�का�आनंद��लया।�
• नील��कसान��ने�बंगाल�के�ना�दया��जले�म��नील�उगाने�से�इंकार�कर��दया।�
• �कसान��का�नेतृ�व�ना�दया�के��ब�वास�बंधु��, मालदा�के�रफ�क�म�डल�और�पाबना�के�कादर�मु�ला�ने��कया।��व�ोह�को�कई�जम�दार�
से��वशेष��प�से�नारेल�के�रामरतन�म�लक�का�समथ�न��मला।�
• द�नबंधु��म�ा��ारा�1858 - 59 म���लखे�गए�नील�दप�ण�(द��मरर�ऑफ�इं�डगो) के�नाटक�ने��कसान��क����थ�त�को�सट�क��प�से
�च��त��कया�है।�
• �व�ोह�से��च��तत�, �लांटस��को�हमले�से�बचाने�के��लए�सरकार�ने�सेना�बुलाई�, और�नील�उ�पादन�क���व�था�क��जांच�करने�के��लए
नील�आयोग�क���थापना�क��
• �व�ोह�के�बाद�, बंगाल�म��नील�उ�पादन��व�त�हो�गया।�ले�कन��लांटस��ने�अब�अपना�संचालन��बहार�म���थानांत�रत�कर��दया।�
• जब�महा�मा�गांधी�द��ण�अ��का�से�लौटे �, तो��बहार�के�एक��कसान�ने�उ�ह��चंपारण�आने�के��लए�राजी��कया�तथा�वहां�के��कसान�
क���द� शा��दखाई।�1917 म��महा�मा�गांधी�क��या�ा�ने�नील��लांटस��के��खलाफ�चंपारण�आंदोलन�क��शु�आत�क�।�

कथन��व�ेषण:

कथन�1 कथन�2 कथन�3

अस�य� स�य� अस�य�

नील�आयोग�ने��लांटस��क��आलोचना�क��, यह नील��व�ोह�बंगाल�म���आ
�तनक�ठया��णाली�चंपारण�स�या�ह�से घो�षत��कया��क�नील�उ�पादन��कसान��के��लए , तथा��व�ोह�के�बाद
संबं�धत�है।� लाभदायक�नह��था।�इसने�रैयत��को�अपने �लांटस���बहार�म�
मौजूदा�अनुबंध��को�पूरा�करने�के��लए�कहा।� �थानांत�रत�हो�गए।�

(Source: Our Past – III Part – 1, Class VIII NCERT

QUESTION 33.
1857 के��व�ोह�के�संदभ��म���न�न�ल�खत�यु�म��पर��वचार�कर�:

नेता��े��

1. रानी�अवंतीबाई�लोधी�कानपुर�
2. अहम��ला�शाह�फैजाबाद�
3. बेगम�हजरत�महल�लखनऊ�

ऊपर�द��गई�कौन�सी�जोड़ी�गलत�है�/ ह��?

IASbaba
Web: http://ilp.iasbaba.com/ Score:
Email: ilp@iasbaba.com 3.00 / 200
Page 135
AIPTS/ILP VETERANS-
Exam Title :
2020 TE...
Email : yadavanurag075@gmail.com
Contact : 8882839768

a) केवल�1
b) केवल�2 और�3
c) केवल�1 और�3
d) 1, 2 और�3
Correct Answer: A
Your Answer:
Explanation

Solution (a)

Basic Information:

• 10 मई�को�, 3 ने�टव�कैवेलरी�के�सै�नक��ने�अपने�कैद��कए�गए�सा�थय��को��रहा�कर��दया�, उनके�अ�धका�रय��को�मार�डाला�और


�व�ोह�के�बैनर�को�उकसाया।�वे�सूया��त�के�बाद��द�ली�के��लए�रवाना��ए�और�इस�तरह�1857 के��व�ोह�का�उदय��आ।�
• रे�जम�ट�के�बाद�अ�य�रे�जम�ट�ने��व�ोह��कया�तथा��द�ली�, कानपुर�और�लखनऊ�जैसे��व�ोह�के�नोडल��ब����पर�अ�य�सै�नक��को
शा�मल�होने�के��लए�रवाना��कया।�
• म�य��दे श�के�मंडला��े��म��रामगढ़�क��रानी�अवंतीबाई�लोधी�ने�अं�ेज��के��खलाफ�चार�हजार�क��सेना�खड़ी�क��और�नेतृ�व��कया।�
• नाना�साहेब�ने��वग�य�पेशवा�बाजीराव���तीय�के�द�क�पु��, जो�कानपुर�के�पास�रहते�थे�, सश���बल��को�इक�ा��कया�और�शहर�से
���टश�गैरीसन�को�बाहर��नकाल��दया�और��वयं�को�पेशवा�घो�षत��कया।�
• लखनऊ�म���बर�जस�का�दर�, अपद�थ�नवाब�वा�जद�अली�के�पु��को�उनक��माँ�बेगम�हज़रत�महल�के�अधीन�नए�नवाब�के��प�म�
घो�षत��कया�गया�था।�उसने��व�ोह�म��स��य�भाग��लया।�
• कई�नए�नेता�सामने�आए�जैसे�अहम��ला�शाह�एक�मौलवी�, फैजाबाद�से�, ने�भ�व�यवाणी�क���क�अं�ेज��का�शासन�ज�द�ही�समा�त
हो�जाएगा।�उ�ह�ने�समथ�क��क��भारी�ताकत�जुटाई।�
• झाँसी�म��, रानी�ल�मीबाई��व�ोही��सपा�हय��म��शा�मल�हो�ग��और�ता�या�टोपे�के�साथ�लड़�।�
• बरेली�के�एक�सै�नक�ब�त�खान�ने��द�ली�म��आने�वाले�सेना�नय��के�एक�बड़े�दल�का�काय�भार�संभाला।�
• �बहार�के�एक�पुराने�जम�दार�कुंवर��स�ह��व�ोही��सपा�हय��म��शा�मल�हो�गए�तथा�उ�ह�ने�अं�ेज��के�साथ�यु���कया।�

(Source: Our Past – III Part – 1, Class VIII NCERT))

QUESTION 34.
नई��द�ली�और�इसक��इमारत��को��डजाइन�करने�के��लए��न�न�ल�खत�म��से��कस�वा�तुकार�को�बुलाया�गया�था�?

a) एडवड��लु�टयन�
b) हरबट� �बेकर�
c) रॉबट� �टोर�रसेल�
d) (a) और�( दोन���वक�प�
Correct Answer: D
Your Answer:
Explanation

Solution (d)

Basic Information:

• 1911 म��, जब�इं�ल�ड�म���क�ग�जॉज��पंचम�क��ताजपोशी��ई�, तो�इस�अवसर�को�मनाने�के��लए��द�ली�म��एक�दरबार�का�आयोजन


�कया�गया।�इस�दरबार�म��भारत�क��राजधानी�को�कलक�ा�से��द�ली��थानांत�रत�करने�के��नण�य�क��घोषणा�क��गई।�
• नई��द�ली�का��नमा�ण�मौजूदा�शहर�के�द��ण�म��रायसीना��हल�पर�10 वग��मील�के�शहर�के��प�म���कया�गया�था।�
• दो�आ�क�टे�ट�, एडवड��लु�टयन�और�हब�ट��बेकर�, �ज�ह��नई��द�ली�और�इसक��इमारत��को��डजाइन�करने�के��लए�कहा�गया।�
• नई��द�ली�म��सरकारी�प�रसर�म��दो-मील�एवे�यू�, �क��सवे�(अब�राजपथ) शा�मल�है�, �जसके�कारण�वायसराय�का�महल�(अब�रा�प�त
भवन) , राज�व�के�दोन��ओर�स�चवालय�भवन��के�साथ�था।�

IASbaba
Web: http://ilp.iasbaba.com/ Score:
Email: ilp@iasbaba.com 3.00 / 200
Page 136
AIPTS/ILP VETERANS-
Exam Title :
2020 TE...
Email : yadavanurag075@gmail.com
Contact : 8882839768

• इन�इमारत��क��सु�वधा��को�भारत�के�शाही�इ�तहास�के��व�भ��अव�धय��से�उधार��लया�गया�था�, ले�कन�सम���प�से��ला�सकल
�ीस�थी।�
• वाइसराय�के�महल�के�क���य�गुंबद�को�सांची�म��बौ���तूप�से�कॉपी��कया�गया�था�, तथा�लाल�बलुआ�प�थर�और�न�काशीदार����न�या
जा�लय��को�मुगल�वा�तुकला�से�उधार��लया�गया�था।�
• वा�तुकार�ने�यह�सु�न��त��कया��क�वायसराय�का�महल�शाहजहाँ�क��जामा�म��जद�से�ऊँचा�हो�, जो����टश��भु�व�/ मह�व�का�दावा
करता�था।�
• नई��द�ली�को�बनने�म��लगभग�20 साल�लगे।�
• आ�क�टे�ट�रॉबट� �टोर�रसेल�ने�कनॉट��लेस�को��डज़ाइन��कया�था��जसका�नाम��ूक�ऑफ़�कनॉट�के�नाम�पर�रखा�गया�था।�1929 से�
1933 तक��नमा�ण�लगभग�4 वष��तक�चला।�

(Source: Our Past – III Part – 1, Class VIII NCERT)

QUESTION 35.
�न�न�ल�खत�म��से�कौन�' प�रवत�क�नह��' (no changer) था�?

a) सरदार�पटे ल�
b) डॉ. राज����साद�
c) मोतीलाल�नेह��
d) (a) और�( दोन���वक�प�
Correct Answer: C
Your Answer:
Explanation

Solution (c)

Basic Information:

• गांधी�क���गर�तारी�(माच��1922) के�बाद�, रा�वाद��नेतृ�व�के�बीच��वघटन�, अ�व�था��ई।�


• सं�मण�काल�के�दौरान�, यानी�आंदोलन�के��न���य�चरण�के�दौरान��या�करना�है�, इस�पर�कां�े�सय��म��बहस�शु��हो�गई।�
• सी.आर. दास�, मोतीलाल�नेह��और�अजमल�खान�के�नेतृ�व�म��एक�वग���वधायी�प�रषद��के�ब�ह�कार�का�अंत�चाहता�था�ता�क
रा�वाद��इन��वधानसभा��क��बु�नयाद��कमजो�रय��को�उजागर�कर�सक��तथा�इन�प�रषद��का�उपयोग�राजनी�तक�संघष��के�अखाड़े�के
�प�म��लोक��य�उ�साह�के��प�म��कर�सक�।�।�
• �वधान�प�रषद��म���वेश�क��वकालत�करने�वाल��को��वराजवा�दय��के��प�म��जाना�जाता�है।�व�लभभाई�पटे ल�, राज����साद�, सी.
राजगोपालाचारी�और�एम. ए. अंसारी�के�नेतृ�व�म���वचार�के��सरे�लोग��को�' नो-च�जस��' के��प�म��जाना�जाने�लगा।�
• ' नो-च�जस��' ने�काउं�सल�म���वेश�का��वरोध��कया�, वकालत�क��, रचना�मक�काम�पर�एका�ता�और�ब�ह�कार�और�असहयोग�को
जारी�रखा�तथा��नलं�बत�स�वनय�अव�ा�काय��म�को��फर�से�शु��करने�के��लए�चुपचाप�तैयारी�क�।�
• �वचार�के�दो��कूल��के�बीच�प�रषद�के��वेश�के����पर�मतभेद�कां�ेस�के�गया�स��म��(�दसंबर�1922) प�रषद��के��वराजवा�दय��के
��ताव�को�' समा�त�या�संशो�धत�' करने�क��हार�के�प�रणाम�व�प��आ।�
• सी. आर. दास�और�मोतीलाल�नेह��ने�कां�ेस�के��मशः�अ�य��और�स�चव�पद�से�इ�तीफा�दे ��दया�तथा�सी. आर. दास�के�अ�य�
और�मोतीलाल�नेह��के�स�चव�के��प�म��कां�ेस-�खलाफत��वरा�य�पाट��के�गठन�क��घोषणा�क�।�
• �सतंबर�1923 म���द�ली�म��एक�बैठक�म��समझौता��कया�गया।��वराजवा�दय��को�कां�ेस�के�भीतर�एक�समूह�के��प�म��चुनाव�लड़ने
क��अनुम�त�द��गई।�

(Source: Spectrum)

QUESTION 36.
भारतीय�इ�तहास�के�संदभ��म��‘ �च�ट्ज़�’, ‘ कोसैस�’, ‘ बंद�ा�’ (chintz, cossaes, bandanna) जैसे�श�द��न�न�ल�खत�से�संबं�धत�ह�:

a) मु��त�सूती�व����के��कार�
b) वा�तु�संरचना��के��कार�

IASbaba
Web: http://ilp.iasbaba.com/ Score:
Email: ilp@iasbaba.com 3.00 / 200
Page 137
AIPTS/ILP VETERANS-
Exam Title :
2020 TE...
Email : yadavanurag075@gmail.com
Contact : 8882839768

c) महीन�रेशमी�व���के��कार�
d) यूरोपीय�लोग��क��मदद�करने�वाले�भारतीय��ापार�एज�ट��के��कार।�
Correct Answer: A
Your Answer:
Explanation

Solution (a)

Basic Information:

• लगभग�1750 म��, ���टश�बंगाल�पर��वजय�पाने�से�पहले�, भारत�सूती�व����का�संसार�का�सबसे�बड़ा�उ�पादक�था।�


• वे�द��ण�पूव��ए�शया�(जावा�, सुमा�ा�और�पेनांग) और�प��म�और�म�य�ए�शया�के�साथ�बड़े�पैमाने�पर�कारोबार�करते�थे।�
• ‘ �च�ट् ज़�’,‘ कॉसैस�’( ख़ास) , ‘ बंद�ा�’ मु��त�सूती�कपड़े�थे।�
• �च�ट्ज़�को��ह�द��श�द�छ�ट�से��लया�गया�है�, जो�छोटे �और�रंगीन�फूल��के��डजाइन�वाला�एक�कपड़ा�है।�रानी�स�हत�इं�ल�ड�के�अमीर
लोग��ने��वयं�भारतीय�कपड़े�पहने�थे�
• बंद�ा�श�द�अब��कसी�भी�चमक�ले�और�मु��त��प�े �के�शीष��या��सर�के��लए�संद�भ�त�करता�है।�मूल��प�से�, श�द�" बंधना�" से��लया
गया�है।�
• मस�लन�, कै�लको�यूरोपीय�के�साथ�लोक��य�अ�य�व���ह�।�

(Source: Our Past – III Part 2, VIII NCERT))

QUESTION 37.
भारत�म���श�ा�पर�गांधी�के����कोण�के�संदभ��म���न�न�ल�खत�कथन��पर��वचार�कर�:

1. उ�ह�ने�माना��क�औप�नवे�शक��श�ा�ने�भारतीय��के�मन�म��हीनता�क��भावना�पैदा�क�।�
2. उ�ह�ने��ढ़ता�से�महसूस��कया��क�भारतीय�भाषा��को�ही��श�ण�का�मा�यम�होना�चा�हए।�
3. उ�ह�ने��श�ा�म���ावसा�यक���श�ण�के�मह�व�पर�जोर��दया।�

ऊपर��दए�गए�कथन��म��से�कौन�सा�सही�है�/ ह��?

a) केवल�1 और�2
b) केवल�1 और�3
c) केवल�2 और�3
d) 1, 2 और�3
Correct Answer: D
Your Answer:
Explanation

Solution (d)

Basic Information:

• बे�सक�रा�ीय��श�ा�योजना�नामक��श�ा�पर�गांधी�के��वचार��को�वधा��म��22, 23 अ�टू बर�, 1937 को�अ�खल�भारतीय�रा�ीय


�श�ा�स�मेलन�म��आगे�रखा�गया�है।�
• महा�मा�गांधी�ने�तक���दया��क�औप�नवे�शक��श�ा�ने�भारतीय��के�मन�म��हीनता�क��भावना�पैदा�क�।�इसने�उ�ह��प��मी�स�यता�को��े�
के��प�म��दे खा�, और�अपनी�सं�कृ�त�म��उनके�गौरव�को�न���कया।�
• प��म��ारा�बढ़ावा��दया�गया�, प��म�से�आने�वाली�हर�चीज�क��सराहना�करते��ए�, इन�सं�थान��म���श��त�भारतीय����टश�शासन
क���शंसा�करने�लगे।�
• महा�मा�गांधी�एक�ऐसी��श�ा�चाहते�थे�जो�भारतीय��को�उनक��ग�रमा�और��वा�भमान�क��भावना�को�बहाल�करने�म��मदद�कर�सके।�
• महा�मा�गांधी�ने��ढ़ता�से�महसूस��कया��क�भारतीय�भाषा��को��श�ण�का�मा�यम�होना�चा�हए।�

IASbaba
Web: http://ilp.iasbaba.com/ Score:
Email: ilp@iasbaba.com 3.00 / 200
Page 138
AIPTS/ILP VETERANS-
Exam Title :
2020 TE...
Email : yadavanurag075@gmail.com
Contact : 8882839768

• प��मी��श�ा�, महा�मा�गांधी�ने�कहा�, मौ�खक��ान�के�बजाय�पढ़ने�और��लखने�पर��यान�क���त��कया�; यह�जी�वत�अनुभव�और


�ावहा�रक��ान�के�बजाय�पा�पु�तक��को�मह�व�दे ती�है।�
• उ�ह�ने�तक���दया��क��श�ा�को�एक�����के��दमाग�और�आ�मा�को��वक�सत�करना�चा�हए।�
• उ�ह�ने�दावा��कया��क�लोग��को�अपने�हाथ��से�काम�करना�था�, एक��श�प�सीखना�था�, और�यह�जानना�था��क��व�भ��चीज��कैसे
संचा�लत�होती�ह�।�इससे�उनके��दमाग�और�उनक��समझने�क���मता�का��वकास�होगा।�

(Source: Our Past – III Part 2, VIII NCERT)

QUESTION 38.
ई.वी. रामा�वामी�नायकर�के�बारे�म���न�न�ल�खत�कथन��पर��वचार�कर�:

1. वह�भारतीय�रा�ीय�कां�ेस�( INC) के�सद�य�थे�


2. उ�ह�ने�ज��टस�पाट��क���थापना�क�।�
3. उ�ह�ने�साइमन�कमीशन�का��वागत��कया�तथा�नमक�स�या�ह�का��वरोध��कया�

ऊपर��दए�गए�कथन��म��से�कौन�सा�सही�है�/ ह��?

a) केवल�1
b) केवल�1 और�3
c) केवल�2
d) 2 और�3
Correct Answer: B
Your Answer:
Explanation

Solution (b)

Basic Information:

• 1879 म��ज�मे�पे�रयार�को�त�मल��क��पहचान�और��वा�भमान�को�भुनाने�के��लए�आ�म-स�मान�आंदोलन�के��लए�याद��कया�जाता�है।
• पे�रयार�ने�अपने�राजनी�तक�जीवन�क��शु�आत�अपने�गृहनगर�इरोड�म��कां�ेस�काय�कता��के��प�म��क��थी।�उ�ह�ने��त�नेलवेली�के�पास
चेरनमहादे वी�म��रा�वाद��नेता�वी. वी. अ�यर�के��वा�म�व�वाले�कां�ेस-�ायो�जत��कूल�गु�कुलम�म���ा�ण�और�गैर-�ा�ण�छा���के
�लए�अलग-अलग�भोजन�करने�के����पर�गांधी�के�साथ��ववाद��कया।�
• कां�ेस�को�अपने����कोण�म��मोड़ने�म���वफल�होने�के�बाद�, पे�रयार�ने�1925 म��पाट��से�इ�तीफा�दे ��दया�, तथा��वयं�को�ज��टस
पाट��और�आ�म�स�मान�आंदोलन�से�जोड़ा�, �जसने�सामा�जक�जीवन�, खासकर�नौकरशाही�म���ा�ण��के��भु�व�का��वरोध��कया।�
• पे�रयार�क����स���1924 के�वैकोम�स�या�ह�के�दौरान�त�मल��े��से�परे�फैली��ई�थी।�बाद�म��उ�ह��वैकोम�वीरार�(वैकोम�के�हीरो) के
�प�म��जाना�जाएगा।�
• एक�समाज�सुधारक�के��प�म��, उ�ह�ने�सामा�जक�, सां�कृ�तक�और�ल��गक�असमानता��पर��यान�क���त��कया�, तथा�उनके�सुधार
के�एज�डे�ने��व�ास�, �ल�ग�और�परंपरा�के�मामल��पर����उठाया।�
• आ�म�स�मान�आंदोलन�म��उ�ह�ने��बना�र�म��के�शा�दय��को�बढ़ावा��दया�तथा�संप���और�साथ�ही�म�हला��के��लए�तलाक�के
अ�धकार�को�मंजूरी�द�।�
• ज��टस�पाट��, आ�धका�रक�तौर�पर�द��ण�भारतीय��लबरल�फेडरेशन�क���थापना�20 नवंबर�, 1916 को�म�ास�के��व�टो�रया
मेमो�रयल�हॉल�म��डॉ. सी. नतासा�मुद�लयार�और�ट�. एम. नायर�और�पी. �यागराय�चे����ारा�क��गई�थी।�

कथन��व�ेषण:

कथन�1 कथन�2 कथन�3

IASbaba
Web: http://ilp.iasbaba.com/ Score:
Email: ilp@iasbaba.com 3.00 / 200
Page 139
AIPTS/ILP VETERANS-
Exam Title :
2020 TE...
Email : yadavanurag075@gmail.com
Contact : 8882839768

स�य� अस�य� स�य�

वह�कां�ेस�के�सद�य�बन�गए�थे�, परन् उ�ह�ने�गांधी�का�डटकर��वरोध


1940 के�दशक�म��, पे�रयार�ने���वड़
तु�इसे�घृणावश�छोड़��दया�जब�उ�ह�ने �कया।�वह�उन�कुछ�नेता��म��से�थे
कज़गम�का�शुभारंभ��कया�, �जसम��शा�मल
पाया��क�रा�वा�दय���ारा�आयो�जत , �ज�ह�ने�साइमन�कमीशन�का
त�मल�, मलयालम�, तेलुगु�और�क�ड़�भाषी
एक�भोज�म��, बैठने�क���व�था�ने �वागत��कया�था।�उ�ह�ने�नमक
लोगो�के��लए�एक��वतं����वड़�नाडु �था।�
जा�त�भेद�का�पालन��कया� स�या�ह�का�कड़ा��वरोध��कया�

(Source: https://www.outlookindia.com/magazine/story/india-news-periyar-ev-ramasamy-the-
man-who-opposed-mahatma-gandhis-idea-of-india/302169 )

QUESTION 39.
�न�न�ल�खत�म��से�कौन-सी�कां�ेस�क���थापना�के�शु�आती�वष��म��कां�ेस�क��मांग��थ��?

1. �वधान�प�रषद��को�अ�धक���त�न�ध�बनाया�जाना�चा�हए�, तथा�अ�धक�श����दान��कया�जाना�चा�हए�, और�उन��ांत��म��आरंभ


�कया�जाना�चा�हए�, जहां�कोई�मौजूद�नह��थी।�
2. �सफ��लंदन�म��ही�नह��, ब��क�भारत�म��भी��स�वल�सेवा�क��परी�ाएं�आयो�जत�करना।�
3. �यायपा�लका�को�काय�पा�लका�से�पृथक�करना�

सही�कूट�का�चयन�कर�:

a) केवल�1 और�2
b) केवल�1 और�3
c) केवल�2 और�3
d) 1, 2 और�3
Correct Answer: D
Your Answer:
Explanation

Solution (d)

Basic Information:

• यह�अ�सर�कहा�गया�है��क�पहले�बीस�वष��म��कां�ेस�अपने�उ�े �य��और�तरीक��म��" उदारवाद��" थी।�


• इस�अव�ध�के�दौरान�, इसने�सरकार�और��शासन�म��भारतीय��के��लए�एक�बड़ी�आवाज�क��मांग�क�।�
• इसने�मांग�क���क�भारतीय��को�सरकार�म��उ�च�पद��पर�रखा�जाए।�इस�उ�े �य�के��लए�, इसने�इं�ल�ड�म��ही�नह��, ब��क�भारत�म��भी
�स�वल�सेवा�परी�ा��का�आयोजन��कया�जाए।�
• �शासन�के�भारतीयकरण�क��मांग�न�लवाद�के��खलाफ�एक�आंदोलन�का��ह�सा�थी�, �य��क�उस�समय�अ�धकांश�मह�वपूण��नौक�रय�
पर��ेत�एका�धकार�था।�भारतीयकरण�, यह�आशा����क��गई�थी�, यह�भी�इं�ल�ड�के��लए�धन��नकासी�को�कम�करेगा।�
• शु�आती�कां�ेस�ने�राज�व�म��कमी�, सै�य�खच��म��कटौती�और��स�चाई�के��लए�अ�धक�धन�क��मांग�क�।�इसने�नमक�कर�, �वदे श��म�
भारतीय�मज�र��के�इलाज�और�वनवा�सय��क��पीड़ा��पर�कई���ताव�पा�रत��कए�- जो��क�एक�दखल�दे ने�वाले�वन��शासन�के
कारण��ए।�
• इससे�पता�चलता�है��क��श��त�अ�भजात�वग��का�एक��नकाय�होने�के�बावजूद�, कां�ेस�ने�केवल�पेशेवर�समूह��, जम�दार��या
उ�ोगप�तय��क��ओर�से�बात�नह��क�।�

QUESTION 40.

IASbaba
Web: http://ilp.iasbaba.com/ Score:
Email: ilp@iasbaba.com 3.00 / 200
Page 140
AIPTS/ILP VETERANS-
Exam Title :
2020 TE...
Email : yadavanurag075@gmail.com
Contact : 8882839768

चा�स��वुड्स�एजुकेशन��ड�पैच�, 1854 के�बारे�म���न�न�ल�खत�कथन��पर��वचार�कर�:

1. इसने�भारत�के��लए�‘ डाउनवड���न�पंदन��स�ांत�’ (downward filtration theory) क��अं�ेजी��श�ा�नी�त�क���सफा�रश


क�।�
2. 1857 म��लंदन��व��व�ालय�के�मॉडल�पर�कलक�ा�, बॉ�बे�और�म�ास�म��तीन��व��व�ालय��के��नमा�ण�के�मा�यम�से�उ�च��श�ा�को
बढ़ावा��दया�गया�था।�
3. इसने�म�हला�और��ावसा�यक��श�ा�और��श�क��के���श�ण�पर�जोर��दया।�

ऊपर��दए�गए�कथन��म��से�कौन�सा�सही�है�/ ह��?

a) केवल�1 और�2
b) केवल�1 और�3
c) केवल�2 और�3
d) 1, 2 और�3
Correct Answer: C
Your Answer:
Explanation

Solution (c)

Basic Information:

• 1854 म��, चा�स��वुड�ने�भारत�के��लए�एक�शै��क��णाली�पर�एक��ेषण�तैयार��कया।�यह�" भारत�म��अं�ेजी��श�ा�का�मै�ना�काटा��"


माना�जाता�है�, यह�द�तावेज�भारत�म���श�ा�के��सार�क��पहली��ापक�योजना�थी।�
• इसने��श�ा�के�मा�यम�के��प�म���कूल��तर�पर�वना��यूलर�और�उ�च�अ�ययन�के��लए�अं�ेजी�क���सफा�रश�क�।�
• यह��नधा��रत��कया�गया�है��क�सरकारी�सं�थान��म���दान�क��जाने�वाली��श�ा�धम��नरपे��होनी�चा�हए।�
• इसने��नजी�उ�म�को��ो�सा�हत�करने�के��लए�अनुदान�क��एक��णाली�क���सफा�रश�क�।�
• �श�ा��वभाग�सभी��ांत��म���था�पत��कए�गए�थे।�
• वुड�के��ड�पैच�के�आदश��और�तरीके�पांच�दशक��तक��े��म��हावी�रहे�, �जसने�भारत�म���श�ा��णाली�का�तेजी�से�प��मीकरण�दे खा�,
जो�यूरोपीय�मु�या�यापक��और��ाचाय���ारा�संचा�लत��आ�

कथन��व�ेषण:

कथन�1 कथन�2 कथन�3

अस�य� स�य� स�य�

इसम��नीचे�के�गाँव��म��वना��यूलर��ाइमरी
इसने�भारत�सरकार�से�कहा��क�वह �कूल��से�पदानु�म�को��व��थत��कया�, इ
जनसाधारण�क���श�ा�के��लए��ज�मेदारी सके�बाद�एं�लो-वना��युलर�हाई��कूल�और
इसने�म�हला��श�ा�और�यहां�तक��क
�हण�करे�, इस��कार�‘ अधोगामी��न�पंदन �जला��तर�पर�एक�संब��कॉलेज�, और
�श�क���श�ण�को�बढ़ावा��दया।�
�स�ांत�’ (downward filtration कलक�ा�, बॉ�बे�और�म�ास�के��ेसीड�सी
theory) को��नर�त�करता�है।� शहर��म��संब���व��व�ालय��को�शा�मल
�कया�गया।�

IASbaba
Web: http://ilp.iasbaba.com/ Score:
Email: ilp@iasbaba.com 3.00 / 200
Page 141
AIPTS/ILP VETERANS-
Exam Title :
2020 TE...
Email : yadavanurag075@gmail.com
Contact : 8882839768

(Source: Spectrum)

QUESTION 41.
�न�न�ल�खत�म��से�कौन�सा�कथन�उ�ीसव��शता�द��के�सामा�जक�और�धा�म�क�सुधार�आंदोलन��क���वशेष��वशेषता��म��से�एक�नह��है�?

a) आंदोलन�एक�संक�ण��सामा�जक��थान�तक�ही�सी�मत�थे।�
b) अ�धकांश�सुधार�आंदोलन��क��अगुवाई�तीन�उ�च�जा�तय��, �ा�ण�, काय�थ�और�बै��से��ई।�
c) ऊपर�से�सुधार�लागू�करने�के��लए�उ�ह�ने�कानून�पर�अ�धक�भरोसा��कया।�
d) उ�ीसव��सद��के�शु�आती�दौर�के�सुधारक��ने�औप�नवे�शक�शासन�के�क�र��वरोध�का��दश�न��कया�
Correct Answer: D
Your Answer:
Explanation

Solution (d)

Basic Information:

• 19 व��सद��के�सामा�जक�धा�म�क�आंदोलन�जैसे����समाज�, यंग�बंगाल�मूवम�ट�, परमहंस�मंडली�, �ाथ�ना�समाज�, आय��समाज�,


आ�द।�
• उ�ीसव��शता�द��के�शु�आती�सुधारक��ने�औप�नवे�शक�शासन�के�उदार��वभाव�म��आंत�रक��व�ास�का��दश�न��कया�तथा�ऊपर�से
सुधार�लागू�करने�के��लए�कानून�पर�अ�धक�भरोसा��कया।�
• ये�आंदोलन�एक�संक�ण��सामा�जक��थान�तक�ही�सी�मत�थे�, �य��क�सुधारवाद��भावना�ने�केवल�एक�छोटे �से�कुलीन�समूह�से�अपील
क��, जो�मु�य��प�से�औप�नवे�शक�शासन�के�आ�थ�क�और�सां�कृ�तक�लाभाथ��थे।�
• सामा�जक��प�से�, वे��यादातर��ह���थे�, और�हालां�क�सद�यता�के��लए�जा�त�कोई�बड़ी�कसौट��नह��थी�, उनम��से��यादातर�तीन�उ�च
जा�तय��, �ा�ण�, काय�थ�और�बै��से�संबं�धत�थे।�इन�समूह���ारा����आंदोलन�को�लगभग��वशेष��प�से�संर�ण��दया�गया�था।�
• यह�साधारण�जनता�से�अलग-थलग�रहा।�
• जमीनी��तर�पर�एक�सुधारवाद��सामा�जक�चेतना�बनाने�के��लए�ब�त�कम�या�कोई��यास�नह���कया�गया�था�, जहां�धा�म�क
पुन��थानवाद�को�बाद�म��उपजाऊ�जमीन��मली।�
• ���टोफ़�जाफरलॉट�के�श�द��म��अपनी���थ�त�को�सं�ेप�म��बताने�के��लए�, उ�ह�ने�" �ह���परंपरा�के�मूल�को�संर��त�करते��ए�उ�ह�
प��मी�आधु�नकता�के�अनुकूल�बनाने�के��लए�अपने�समाज�और�इसक��धा�म�क��था��को�सुधारने�का�बीड़ा�उठाया।�"

(Source: “From Plassey to Partition: A History of Modern India”)

QUESTION 42.
�क�ूर��व�ोह�( 1824) �कससे�संबं�धत�है:

a) रानी�चे��मा�
b) रानी�अवंतीबाई�
c) रानी��गा�वती�
d) रानी����मा�
Correct Answer: A
Your Answer:
Explanation

Solution (a)

IASbaba
Web: http://ilp.iasbaba.com/ Score:
Email: ilp@iasbaba.com 3.00 / 200
Page 142
AIPTS/ILP VETERANS-
Exam Title :
2020 TE...
Email : yadavanurag075@gmail.com
Contact : 8882839768

Basic Information:

• �क�ूर�क��रानी�रानी�चे��मा�एक�ऐसी�यो�ा�थ��, �ज�ह�ने�19 व��शता�द��के�शु�आती��दन��म�����टश�सेना�के��खलाफ�यु��का�नेतृ�व


�कया�था�, जब�कई�शासक�अं�ेज��के�बुरे��प�से�प�र�चत�नह��थे।�
• चे��मा�का�ज�म�वत�मान�कना�टक�के�बेलगावी��जले�के�एक�छोटे �से�गाँव�ककाट��म���आ�था।�
• वह��क�ू��(अब�कना�टक�म�) क��रानी�बन�गई�, जब�उ�ह�ने�दे साई�प�रवार�के�राजा�मलसाराजा�से�शाद��क�।�
• �पगत��स�ांत�के�तहत�, ���टश�ने�गोद��लए�ब�चे�, �शव�ल�ग�पा�को��स�हासन�के�उ�रा�धकारी�के��प�म��खा�रज�कर��दया।�
• 1824 म�����टश�ई�ट�इं�डया�कंपनी��ारा��क�ूर�क���रयासत�पर�' �पगत��स�ांत�' थोपा�गया�था�, इससे�पहले�भी�इसे�लॉड�
डलहौजी��ारा�आ�धका�रक��प�से�����कया�गया�था।�
• अं�ेज��ने�रानी�चेन�मा�को�द�क�और�पूण��अ�धकार�क��नी�त�का�उपयोग�करते��ए�द�क�बालक��शव�ल�ग�पा�को��नवा��सत�करने�का
आदे श��दया।�ले�कन�चेन�मा�ने�आदे श�क��अवहेलना�क�।�
• रानी�चे��मा�ने�अपने�ले��टन�ट�, संगोली�राय�ा�और�गु��स��पा�क��मदद�से�जमकर�लड़ाई�लड़ी�, ले�कन�उसे�परा�जत�कर��दया�गया
और�अंततः�कैद�कर��लया�गया�और�बै�ह�गल��कले�म��कैद�कर��दया�गया�, जहाँ�21 फरवरी�1829 को�उसक��मृ�यु�हो�गई।�

(Source: https://pib.gov.in/newsite/printrelease.aspx?relid=148944 )

QUESTION 43.
1875 म��द�कन�दं गे�के�बारे�म���न�न�ल�खत�कथन��पर��वचार�कर�:

1. �कसान��ने��वशेष��प�से�सा�कार��के�क�जे�म��अपने�ऋण��से��नपटने�के��लए�बांड�, फरमान�और�अ�य�संबं�धत�द�तावेज��को�ल��त
�कया।�
2. �कसान��ने�उन��कसान��और�बालुटेदार��के��खलाफ�सामा�जक���तबंध�लगाए�जो�सा�कार��के�ब�ह�कार�म��शा�मल�नह��ह�गे।�
3. महा�मा��यो�तराव�फुले�के�स�यशोधक�समाज�ने��कसान��के�कारण�का�समथ�न��कया।�

ऊपर��दए�गए�कथन��म��से�कौन�सा�सही�है�/ ह��?

a) केवल�1 और�2
b) केवल�1 और�3
c) केवल�2 और�3
d) 1, 2 और�3
Correct Answer: A
Your Answer:
Explanation

Solution (a)

Basic Information:

• 1875 म��महारा��के�पूना�और�अहमदनगर��जल��म��एक�बड़ा�कृ�ष��कोप��आ।�
• अ�य�रैयतवाड़ी��े���के��कसान��क��तरह�, द�कन�के��कसान��को�भी�सा�कार�के�चंगुल�म��फंसने�और�तेजी�से�अपनी�जमीन�खोने�के
कारण�भू-राज�व�का�भुगतान�करना�मु��कल�हो�गया।�
• इसके�कारण��कसान��और�सा�कार��के�बीच�तनाव�बढ़�गया�था�, �जनम��से�अ�धकांश�बाहरी�लोग- मारवाड़ी�या�गुजराती�थे।�
• इस�समय�तीन�अ�य��वकास��ए।�1860 के�दशक�क��शु�आत�म��, अमे�रक��गृह�यु��कपास�के��नया�त�म��वृ���का�कारण�बना�था�,
�जसने�क�मत��को�बढ़ा��दया�था।�1864 म��गृहयु��क��समा��त�ने�कपास�के��नया�त�म��ती���गरावट�और�क�मत��म���गरावट�को�ज�म
�दया।�इसके�साथ�ही�, 1867 म��, सरकार�ने�भू-राज�व�म��लगभग�50 ��तशत�क��वृ���क�।�खराब�फसल�से���थ�त�और�खराब�हो
गई।�
• �दसंबर�1874 म���स�र�तालुक�के�कड़ाब�गांव�म��एक�सहज��वरोध�आंदोलन�शु���आ।�
• �कसान��ने�अपनी�मांग��को�शां�तपूण��तरीके�से��वीकार�करने�के��लए�मजबूर�करने�के��लए�‘ बाहरी�’ सा�कार��के�पूण��सामा�जक
ब�ह�कार�का�आयोजन��कया।�
• सामा�जक�ब�ह�कार�ज�द�ही�कृ�ष��धान�दं ग��म��त�द�ल�हो�गया�, जब�यह�ब�त��भावी�सा�बत�नह���आ।�12 मई�को�, �कसान
सुपारा�म��, भीमथरी�तालुक�म��, मजार�पर�इक�ा��ए�तथा�सा�कार��के�घर��और��कान��पर�एक��व��थत�हमला�शु���कया।�

IASbaba
Web: http://ilp.iasbaba.com/ Score:
Email: ilp@iasbaba.com 3.00 / 200
Page 143
AIPTS/ILP VETERANS-
Exam Title :
2020 TE...
Email : yadavanurag075@gmail.com
Contact : 8882839768

• औप�नवे�शक�सरकार�ने�द�कन�आयोग�क���थापना�क��, �जसक���सफा�रश��के�कारण�द�कन��कसान��के�राहत�अ�ध�नयम�1879
को�पा�रत��कया�गया�, ता�क�उ�ह��सा�कार��के��खलाफ�एक��न��त�सुर�ा��दान�क��जा�सके।�

कथन��व�ेषण:

कथन�1 कथन�2 कथन�3

स�य� स�य� अस�य�

महारा��के�आधु�नक�रा�वाद��बु��जी�वय�
खात��के�इस��नपटान�म��ब�त�कम��ह�सा
ने��कसान��के�कारण�का�समथ�न��कया।
�ई�थी।�एक�बार�जब�सा�कार��के �ारं�भक�फोकस��ह�सा�से�बचने�के��लए
ज��टस�रानाडे�क��अगुवाई�वाली�पूना
उ�पीड़न�के�ह�थयार�- ऋण�बांड�- को�न� सामा�जक�ब�ह�कार�जैसे�शां�तपूण��साधन�
साव�ज�नक�सभा�ने�1867 के�भू�म�राज�व
कर��दया�गया�था�, आगे��ह�सा�क��कोई का�उपयोग�करने�पर�था।�
समझौते�के��खलाफ��कसान��के�बीच�एक
आव�यकता�महसूस�नह���ई�
सफल�अ�भयान�चलाया�था।�

(Source: India’s Struggle for Independence)

QUESTION 44.
�न�न�ल�खत�म��से�कौन�सा�नेता�मु�य��प�से�अ�खल�भारतीय�रा�य��के�जन�स�मेलन�( AISPC) के�आयोजन�के��लए�उ�रदायी�थे�?

1. बलवंतराय�मेहता�
2. मा�ण�यलाल�कोठारी�
3. जी.आर. अ�यंकर�

सही�कूट�का�चयन�कर�:

a) केवल�1
b) केवल�2
c) केवल�1 और�2
d) 1, 2 और�3
Correct Answer: D
Your Answer:
Explanation

Solution (d)

Basic Information:

• 1920 म��शु���कए�गए�असहयोग�और��खलाफत�आंदोलन�के��भाव�म��रा�य��के�लोग��के�कई��थानीय�संगठन�अ��त�व�म��आए।�
• �जन�रा�य��म���जा�मंडल�या�रा�य��के�जन�स�मेलन�आयो�जत��कए�गए�उनम��से�कुछ�मैसूर�, हैदराबाद�, बड़ौदा�, का�ठयावाड़�रा�य�,
द�खन�रा�य�, जामनगर�, इंदौर�और�नवानगर�थे।�
• यह����या��दसंबर�1927 म��बॉ�बे�म��ऑल�इं�डया��टे ट्स�पीपु�स�कॉ���स�( AISPC) के�आयोजन�के�साथ�सामने�आई�, �जसम�
रा�य��के�700 राजनी�तक�काय�कता���ने�भाग��लया।�इस�पहल�के��लए�मु�य��प�से�उ�रदायी�लोग��म��बलवंतराय�मेहता�, मा�ण�क
यलाल�कोठारी�और�जी.आर. अ�यंकर�थे।�

IASbaba
Web: http://ilp.iasbaba.com/ Score:
Email: ilp@iasbaba.com 3.00 / 200
Page 144
AIPTS/ILP VETERANS-
Exam Title :
2020 TE...
Email : yadavanurag075@gmail.com
Contact : 8882839768

• एआईएसपीसी����टश�राज�क���रयासत��म��राजनी�तक�आंदोलन��का�एक�समूह�था�, �ज�ह���जा�मंडल�या�लोक�प�रषद�कहा�जाता�था।
• 1939 म��, जवाहरलाल�नेह��AISPC के�अ�य��बने�, 1946 तक�इस�पद�पर�रहे।�

(Source: India’s Struggle for Independence)

QUESTION 45.
1927 म��ग�ठत�फेडरेशन�ऑफ�इं�डयन�चै�बस��ऑफ�कॉमस��एंड�इंड����(�फ�क�) के�बारे�म���न�न�ल�खत�कथन��पर��वचार�कर�:

1. �फ�क��क��भू�मका�को��ापार�, वा�ण�य�और�उ�ोग�के�रा�ीय�संर�क�के��प�म��दे खा�गया�


2. यह�आम�तौर�पर�संघष��के�सभी��प�- संवैधा�नक�, जन�आंदोलन�, �ां�तकारी�रणनी�त�आ�द�को�पसंद�करता�था।�
3. इसने�अपने�सद�य��को�गोलमेज�स�मेलन�(आरट�सी) म��भाग�लेने�के��लए��ो�सा�हत��कया�

ऊपर��दए�गए�कथन��म��से�कौन�सा�गलत�है�/ ह��?

a) केवल�1 और�2
b) केवल�1 और�3
c) केवल�2 और�3
d) 1, 2 और�3
Correct Answer: C
Your Answer:
Explanation

Solution (c)

Basic Information:

• 1920 के�दशक�क��शु�आत�से�, जी.डी. �बड़ला�और�पु�षो�मदास�ठाकुरदास�जैसे��व�भ��पूंजीप�तय���ारा�भारतीय�वा�ण��यक�,


औ�ो�गक�और��व�ीय��हत��के�एक�रा�ीय��तर�के�संगठन�क���थापना�के��यास��कए�जा�रहे�थे।�
• FICCI को�ज�द�ही����टश�सरकार�के�साथ-साथ�भारतीय�जनता�ने�भी�सामा�य��प�से�मा�यता�दे �द��, �य��क��मुख�राय�के�साथ-
साथ�भारतीय�पूंजीवाद��वग��के�भीतर�सम��सहम�त�का���त�न�ध�व��कया।�
• भारतीय�पूंजीवा�दय��ने�अपनी�सभी�अ�भ���य��म��सा�ा�यवाद�क��एक��ापक�आ�थ�क�आलोचना��वका�सत�क��थी।�
• भारतीय�पूँजीप�त�वग��क��अपनी�धारणाएँ�थ���क��कस�तरह�सा�ा�यवाद-�वरोधी�संघष��को�छे ड़ा�जाना�चा�हए।�इस�बात�का�अंदेशा�था
�क�बड़े�पैमाने�पर�स�वनय�अव�ा�, खासकर�अगर�यह�लंबे�समय�तक�चलता�रहा�, तो�ऐसी�ताकत��पैदा�होत��, जो�आंदोलन�को
�ां�तकारी��प�से�सामा�जक�अथ��म��बदल�सकती�थ��(यानी�, पूंजीवाद�को�ही�खतरा�था)।�
• पूंजीप�त�सरकार�को�लंबे�समय�तक�बाहर�क��श�ुता�का�समथ�न�करने�के��लए�तैयार�नह��थे��य��क�यह��दन-��त�दन�के�कारोबार�को
जारी�रखने�से�रोकता�था�और�वग��के�अ��त�व�को�खतरे�म��डालता�था।�
• उ�ह�ने�आम�तौर�पर����टश�सरकार�के�साथ�बातचीत�करने�से�इनकार�कर��दया�, तथा��न��त��प�से�कां�ेस�के�पीछे �, संवैधा�नक�और
साथ�ही�, आ�थ�क�मु���पर�कोई�अं�तम���तब�ता��को�करने�के��लए�साथ��दया।�

कथन��व�ेषण:

कथन�1 कथन�2 कथन�3

स�य� अस�य� अस�य�

IASbaba
Web: http://ilp.iasbaba.com/ Score:
Email: ilp@iasbaba.com 3.00 / 200
Page 145
AIPTS/ILP VETERANS-
Exam Title :
2020 TE...
Email : yadavanurag075@gmail.com
Contact : 8882839768

पूँजीप�त�वग��के�नेता��ने
1930 म��, �फ�क��(उदारवा�दय��के
�प���प�से��फ�क��क�
यह�हमेशा�संवैधा�नक�पथ�और�बातचीत�क� �वपरीत) ने�अपने�सद�य��को�बताते��ए
भू�मका�को��ापार�, वा�ण�य
मेज�को�पूरी�तरह�से�नह��छोड़ने�के�प��म��था सलाह�द���क�वे�‘ गोलमेज�स�मेलन�’
और�उ�ोग�के�रा�ीय�संर�क
तथा�आम�तौर�पर�सामू�हक�नाग�रक�अव�ा (RTC) का�ब�ह�कार�कर�।�जब�तक�इस
के��प�म��दे खा�, यह�' औप�न
के��वपरीत�अपने�आधार�को�संवैधा�नक��प� तरह�के�स�मेलन�म��महा�मा�गांधी�ने�एक
वे�शक�भारत�म��आ�थ�क��े�
के�संघष��के�पीछे �रखना�पसंद�करता�था।� �वतं������के��प�म��भाग�नह���लया�है
म��एक�रा�ीय�सरकार�के
, या�कम�से�कम�उनक���वीकृ�त�है।�”
काय��का��दश�न�’ था।�

QUESTION 46.
कां�ेस-�खलाफत��वराज�पाट��(�वराजवा�दय�) से��ह����के��हत��क��र�ा�के��लए�‘ ��त��यावा�दय��' (responsivists) के��प�म��जाना
जाने�वाला�समूह�था।��न�न�ल�खत�म��से�कौन�सा�नेता�' ��त��यावाद��' (responsivist) नह��है�?

a) लाजपत�राय�
b) मदन�मोहन�मालवीय�
c) एन�सी�केलकर�
d) लोकमा�य��तलक�
Correct Answer: D
Your Answer:
Explanation

Solution (d)

Basic Information:

• �वराज�पाट��या�कां�ेस-�खलाफत��वरा�य�पाट��का�गठन�1 जनवरी�1923 को�सी. आर. दास�और�मोतीलाल�नेह��ने��कया�था।�


• 1 अग�त�, 1920 को�लोकमा�य��तलक�का��नधन�हो�गया�, इस�लए�वह��वरा�य�पाट��का��ह�सा�नह��थे।�
• 1924 तक�, �ापक�सां�दा�यक�दं ग��के�कारण��वराजवाद��क����थ�त�कमजोर�हो�गई�थी�, �वराजवा�दय��के�बीच��वयं�सां�दा�यक
और���त��यावाद��-गैर-��त��यावा�दय��क��रेखा��म���वभा�जत�हो�गए�और�1925 म��सी. आर. दास�क��मृ�यु�ने�इसे�और�कमजोर
कर��दया।�
• �वराजवा�दय�- लाला�लाजपत�राय�, मदन�मोहन�मालवीय�और�एन�सी�केलकर�ने�सरकार�के�साथ�सहयोग�क��वकालत�क��और�जहाँ
तक�संभव�हो�सामा�जक��ह����हत��क��र�ा�करने�के��लए�काया�लय�क���थापना�क�।�उ�ह�ने�मोतीलाल�नेह��जैसे�गैर-�ज�मेदार��पर
�ह����वरोधी�होने�और�गोमांस�खाने�का�आरोप�लगाया।�
• इस��कार�, �वरा�य�पाट��के�गैर-��त��यावा�दय��ने�बड़े�पैमाने�पर�नाग�रक�अव�ा�म���व�ास�दोहराया�और�माच��1926 म�
�वधानसभा��से�वापस�ले��लया�, जब�क��वराजवा�दय��(��यावाद�) का�एक�अ�य�वग��1926 के�चुनाव��म��एक�पाट��के��प�म��सभा
म��चला�गया।�

(Source: Spectrum)

QUESTION 47.
�न�न�ल�खत�म��से�कौन�सा�कथन����स��मशन�के��ावधान��पर�कां�ेस�क��आप���को�सही�ढं ग�से�दशा�ता�है�?

1. पूण���वतं�ता�के�बजाय�डो�म�नयन��टे टस�का��ावधान।�
2. सं�वधान�सभा�म���रयासत��का���त�न�ध�व�रा�य��के�लोग���ारा�नह��ब��क�शासक��के�नामांकन��ारा��कया�जाता�है।�
3. वह��ावधान�जो��कसी�भी��ांत�को�नए�सं�वधान�को��वीकार�करने�के��लए�तैयार�नह��था�, उसे���टे न�के�साथ�एक�अलग�समझौते�पर
ह�ता�र�करने�का�अ�धकार�होगा�

सही�कूट�का�चयन�कर�:

IASbaba
Web: http://ilp.iasbaba.com/ Score:
Email: ilp@iasbaba.com 3.00 / 200
Page 146
AIPTS/ILP VETERANS-
Exam Title :
2020 TE...
Email : yadavanurag075@gmail.com
Contact : 8882839768

a) केवल�1 और�2
b) केवल�1 और�3
c) केवल�3
d) 1, 2 और�3
Correct Answer: D
Your Answer:
Explanation

Solution (d)

Basic Information:

����स��मशन�माच��1942 के�अंत�म�����टश�सरकार��ारा���तीय��व��यु��म��अपने��यास��के��लए�पूण��भारतीय�सहयोग�और�समथ�न�हा�सल
करने�का�एक�असफल��यास�था।��मशन�का�नेतृ�व�एक�व�र��मं�ी�सर��टे फोड�����स�ने��कया�था।�

���स�ने�घोषणा�क���क�भारत�म�����टश�नी�त�का�उ�े �य�भारत�म���व-सरकार�का�ज�द�से�ज�द�संभव�बोध�था�

ले�कन�, ���स�और�कां�ेस�नेता��के�बीच�बातचीत�टू ट�गई।�जैसा��क�कां�ेस�ने��ावधान��पर�आप���जताई:

• घोषणा�ने�यु��के�बाद�भारत�डो�म�नयन��टे टस�और�एक�सं�वधान-�नमा�ण��नकाय�का�वादा��कया।�
• �रयासत��के�मामले�म��शासक���ारा�ना�मत��कए�जाने�वाले�सद�य।�
• और�भारत�के��वभाजन�के��ावधान�के�अनुसार।�
• ���टश�सरकार�ने�भी�भारतीय��को��भावी�श���के�त�काल�ह�तांतरण�और�भारत�क��र�ा�के��लए��ज�मेदारी�म��वा�त�वक��ह�सेदारी
क��मांग�को��वीकार�करने�से�इनकार�कर��दया।�

(Source: India’s Struggle for Independence)

QUESTION 48.
आईएनए�जाँच�( INA trials) 1945 के�बारे�म���न�न�ल�खत�कथन�पर��वचार�कर�:

1. आईएनए�कै�दय��क��र�ा�भुलाभाई�दे साई�, तेज�बहा�र�स�ू�, के.वी. काटजू�, नेह��और�आसफ��ारा�क��गयी�थी�


2. नेह��आईएनए�के�पकड़े�गए�सै�नक��को�पथ���दे शभ��मानते�थे।�
3. मु��लम�लीग�, �ह���महासभा�और�भारतीय�क�यु�न�ट�पाट��ने�आईएनए�कारण�का�समथ�न�नह���कया।�

ऊपर��दए�गए�कथन��म��से�कौन�सा�सही�है�/ ह��?

a) केवल�1
b) केवल�1 और�2
c) केवल�2 और�3
d) 1, 2 और�3
Correct Answer: B
Your Answer:
Explanation

Solution (b)

Basic Information:

• ��तीय��व��यु��के�अंत�के�बाद�, �जस�मु�े�ने�सबसे�अ�धक�लोक��य�क�पना�को�पकड़ा�, वह�सुभाष�चं��बोस�क��भारतीय�रा�ीय


सेना�(आईएनए) के�सद�य��का�भा�य�था�, �ज�ह��यु��के�पूव��भाग�म��अं�ेज��ने�पकड़��लया�था।�

IASbaba
Web: http://ilp.iasbaba.com/ Score:
Email: ilp@iasbaba.com 3.00 / 200
Page 147
AIPTS/ILP VETERANS-
Exam Title :
2020 TE...
Email : yadavanurag075@gmail.com
Contact : 8882839768

• कां�ेस�ने�एक�आईएनए��रलीफ�एंड�इं�वायरी�कमेट��का�आयोजन��कया�, �जसने�लोग��को�उनक���रहाई�के��लए�धन�और�भोजन�क�
छोट��रकम��दान�क��, तथा�इन�लोग��के��लए�रोजगार�को�सुर��त�करने�के��लए�सीमांत�सफलता�के�साथ��यास��कया।�
• INA �दवस�12 नवंबर�को�और�INA स�ताह�5 से�11 नवंबर�1945 तक�मनाया�गया।�
• 16 नवंबर�1945 को�धामनगांव�और�शोलापुर�म���कसान�स�मेलन�म��और�29 �दसंबर�1945 को�हैदराबाद�म��अ�खल�भारतीय
म�हला�स�मेलन�के�दसव��स��म���रहाई�क��माँग�क��गई।�
• आईएनए�लोग��के�साथ�सहानुभू�त�म��कुछ��े���म���दवाली�नह��मनाई�गई�थी।�
• सरकारी�कम�चा�रय��और�यहां�तक��क�सश���बल��के�लोग��के�मह�वपूण��वग��भी�INA- समथ�क�भावना�के��वार�म��डू बे��ए�थे।�
• �ेम�सहगल�, गु�ब�श��स�ह��ढ�ल��और�शाह�नवाज़�खान�- ये�����व�भारतीय�रा�ीय�सेना�के��ायल�या�लाल��कले��ायल�से�जुड़े�थे।�

कथन��व�ेषण:

कथन�1 कथन�2 कथन�3

स�य� स�य� अस�य�

INA कै�दय��का�बचाव�कां�ेस नेह��ने�उ�ह��दे शभ��, ले�कन�भटके��ए


मु��लम�लीग�, भारतीय�क�यु�न�ट�पाट��,
ने��कया�था�तथा�भुलाभाई�दे साई कहा�,
यू�नय�न�ट�पाट��, अका�लय��, ज��टस
, तेज�बहा�र�स�ू�, के.एन.
नेह��ने�अं�ेज��के�आ�ासन�के�म�े नजर पाट��, रावल�प�डी�म��अबरार��, रा�ीय
काटजू�ऐ�तहा�सक�लाल��कले
अ�धका�रय��से�उनके��यायपूण���वहार�का �वयंसेवक�संघ�, �ह���महासभा�और��सख
के��ायल�म��, नेह��और�आसफ
आ�ान��कया��क�' बड़े�बदलाव�' भारत�म� लीग�ने�अलग-अलग��तर��पर�INA कार
सभी�अदालत�म��उप��थत��ए
लागू�हो�रहे�ह�।� ण�का�समथ�न��कया।�
थे।�

(Source: Spectrum)

QUESTION 49.
‘ अ�खल�भारतीय�अ�पृ�यता��वरोधी�लीग�, 1932 (All India Anti Untouchability League) �कसके��ारा��था�पत��कया�गया
था:

a) डॉ. बी.आर. अ�बेडकर�


b) एम�के�गांधी�
c) एम.सी. राजा�
d) मदन�मोहन�मालवीय�
Correct Answer: B
Your Answer:
Explanation

Solution (b)

Basic Information:

• गांधी�ने�सां�दा�यक�पुर�कार�को�भारतीय�एकता�और�रा�वाद�पर�हमले�के��प�म��दे खा।�
• गांधी�ने�अपने�अ�य�सभी�काय��को�छोड़��दया�तथा�अ�पृ�यता�के��खलाफ�एक�बड़ा�अ�भयान�शु���कया�- पहले�जेल�से�और�उसके
बाद�अग�त�1933 म��बाहर�से�छोड़ने�के�बाद।�

IASbaba
Web: http://ilp.iasbaba.com/ Score:
Email: ilp@iasbaba.com 3.00 / 200
Page 148
AIPTS/ILP VETERANS-
Exam Title :
2020 TE...
Email : yadavanurag075@gmail.com
Contact : 8882839768

• जेल�म��रहते��ए�, उ�ह�ने��सतंबर�1932 म��अ�खल�भारतीय�अ�पृ�यता��वरोधी�लीग�क���थापना�क��थी�और�जनवरी�1933 म�


सा�ता�हक�ह�रजन�क��शु�आत�क��थी।�
• अपनी��रहाई�के�बाद�, वह�वधा��म��स�या�ह�आ�म�म���थानांत�रत�हो�गए��य��क�उ�ह�ने�1930 म��साबरमती�आ�म�न�लौटने�क�
कसम�खाई�थी�जब�तक��क��वराज�नह���मल�जाता।�
• वधा��से�शु��करते��ए�, उ�ह�ने�नवंबर�1933 से�जुलाई�1934 क��अव�ध�म��दे श�का�ह�रजन�दौरा��कया�, �जसम��20,000 �कलोमी
टर�क���री�तय�क��गई�, अपने�नए��था�पत�ह�रजन�सेवक�संघ�के��लए�धन�इक�ा��कया�, और�अपने�सभी��प��म��अ�पृ�यता�को��र
करने�का��चार��कया।�
• उ�ह�ने�अपने��यास��क��गंभीरता�और�मु�े�के�मह�व�के�बारे�म��अपने�अनुया�यय��को�समझाने�के��लए�8 मई�और�16 अग�त�, 1934
को�दो�उपवास��कए।�

(Source: Spectrum)

QUESTION 50.
���टश�को�कां�ेस- मु��लम�लीग�अवरोध�( Congress- Muslim League logjam) से�बचाने�के��लए�, वेवेल�ने�" �ेकडाउन��लान�"
��ता�वत��कया�, इसका�मु�य���ताव�था:

a) उ�र-प��म�और�उ�र-पूव��के�मु��लम��ांत��म�����टश�सेना�और�अ�धका�रय��क��वापसी�तथा�शेष�दे श�को�कां�ेस�को�स�पना।�
b) उ�र-प��म�और�उ�र-पूव��म��जनमत�सं�ह�क��शु�आत�करना�, जब�क�शेष�दे श�को�कां�ेस�को�स�पना।�
c) भारत�से����टश��क��तेजी�से�वापसी�, इस��कार�समझौता�करने�के��लए�कां�ेस�और�मु��लम�लीग�पर�दबाव�डालना�
d) इनम��से�कोई�भी�नह�।�
Correct Answer: A
Your Answer:
Explanation

Solution (a)

Basic Information:

• भारत�म��तेजी�से��वक�सत�होती�राजनी�तक���थ�त�से��नपटने�के��लए�लॉड��वेवेल�और�उनके��नकटतम�सलाहकार���ारा��ेकडाउन
�लान�तैयार��कया�गया�था�, �जसम��" दबाव�" और�" �नरोध�" के�बीच�एक�म�य�माग��क��क�पना�क��गई�थी।�
• 1945 म���शमला�स�मेलन�क���वफलता�के�बाद�, ठ�क�इसी�तरह�के�ल�य�का�पीछा�करने�के��लए�, वह�एक�गु�त�योजना�के�साथ�आये
, �जसे�इ�तहास�म��वेवेल�के�' �ेकडाउन��लान�' के��प�म��जाना�जाता�है।�
• इस�योजना�ने����टश�सेना�और�अ�धका�रय��क��उ�र-प��म�और�उ�र-पूव��के�मु��लम��ांत��से�वापसी�तथा�दे श�के�बाक���ह�स��को
कां�ेस�को�स�पने�क��प�रक�पना�क�।�
• वेवेल�क���ेकडाउन�योजना�को�दो�मु�य�ल�य��को��यान�म��रखकर�तैयार��कया�गया�था: पहला�, भारत�से�अं�ेज��क��सुर��त�वापसी�;
�सरी�बात�, इसे�एक�भौगो�लक�इकाई�के��प�म��बनाए�रखने�का��यास�करके�भारत�के��वभाजन�से�बचने�के��लए।�
• हालां�क�लंदन�म��एचएमजी��ारा�वेवेल�क��सम��योजना�को�खा�रज�कर��दया�गया�था�, हालां�क�, इसके�त�य�3 जून�क��योजना�म�
वेवल�के�उ�रा�धकारी�माउंटबेटन��ारा��नधा��रत�अं�तम��नकासी�योजना�म��शा�मल�थे।�

(Source: Spectrum)

QUESTION 51.
�ाचीन�भारतीय�महाका���के�संबंध�म��, ' जय�सं�हता�' और�' सतश�ी�सं�हता�' �कस���स��महाका��से�संब��ह��?

a) उप�नषद��
b) पुराण��
c) रामायण�
d) महाभारत�
Correct Answer: D
Your Answer:

IASbaba
Web: http://ilp.iasbaba.com/ Score:
Email: ilp@iasbaba.com 3.00 / 200
Page 149
AIPTS/ILP VETERANS-
Exam Title :
2020 TE...
Email : yadavanurag075@gmail.com
Contact : 8882839768

Explanation

Solution (d)

Basic Information:

महाका��( Epics):

• एक�महावा�य�, को�सं�कृत�क���वशेषता�के��प�म��जाना�जाता�है�तथा�ये�सा�ह�य�के�शु�आती��प�भी�ह�।�भारतीय�सा�ह�य�को�संसार
का�सबसे�पहला�सा�ह�य�माना�जाता�है।�
• एक�महाका��को�अ�याय�या�सग��म���वभा�जत��कया�जाना�चा�हए।�
• हर�अ�याय�सग��के��वषय�या��वषय�के�आधार�पर�एक����गत�और��व�श��तरीके�से�बना�होता�है।�
• महाका���को�कला�के��प�म��माना�जाता�है��य��क�मु�य�कथानक�के�अलावा�ब�त�कुछ�वण�न��दान��कया�गया�है।�
• दो�सबसे���स��महाका��महाभारत�और�रामायण�ह�।�

महाभारत:

• महाभारत�अ�धक��ाचीन�है�और�संभवतः�10 व��शता�द��ईसा�पूव��से�चौथी�शता�द��तक�क����थ�त�को�दशा�ता�है।�
• मूल��प�से�इसम��8800 छं द�शा�मल�थे�और�इसे�‘ जयसं�हता�’ या��वजय�से��नपटने�वाले�छं द��का�सं�ह�कहा�जाता�था।�
• इसके�अलावा�, छं द�को�24000 तक�बढ़ाया�गया�और�भारत�के�नाम�से�जाना�जाने�लगा�, �जसका�नाम�सबसे�पहले�वै�दक
जनजा�तय��म��से�एक�के�नाम�पर�रखा�गया�है।�
• छं द��के�अं�तम�संकलन�ने�सं�या�को�एक�लाख�तक�प�ंचा��दया�, �जसे�' महाभारत�' या�' सतसहा�ी�सं�हता�' के��प�म��जाना�जाता
है।�
• अं�तम�सं�करण�म��कथा�मक�, वण�ना�मक�और�उपदे शा�मक�साम�ी�शा�मल�है।�
• मु�य�कथा�कौरव-पांडव�संघष��क��है�और�उ�र�वै�दक�काल�क��है।�वण�ना�मक�भाग�का��योग�उ�र�वै�दक�काल�के��लए��कया�जाता�है
और�उपचारा�मक�भाग�मौय��र�और�गु�त�काल�के��लए�है।�

QUESTION 52.
उ�र-वै�दक�काल�के�सा�ह�य�के�संबंध�म��, �न�न�ल�खत�म��से�कौन�सा�यु�म�सही��प�से�सुमे�लत�है�?

1. �ौतसू�:: य��वे�दय��के��नमा�ण�के��लए�माप�के��व�भ��पहलु��से�संबं�धत�है।�
2. गृ�सू�:: ज�म�, नामकरण�सं�कार�, �ववाह�आ�द�से�जुड़े�घरेलू�अनु�ान��से�संबं�धत�ह��
3. धम��सू�:: आचार�सं�हता�से�संबं�धत�है।�
4. सु�वसू�:: तीन�उ�च�वण��से�संबं�धत�पदाथ��के�पु�ष�और�राजकुमार��के��लए�बड़े�साव�ज�नक�अनु�ान��से�संबं�धत�है।�

सही��वक�प�चुन�:

a) 1 और�4
b) 2 और�3
c) 2, 3 और�4
d) 1, 2, 3 और�4
Correct Answer: B
Your Answer:
Explanation

Solution (b)

Basic Information:

वै�दक�काल�के�बाद�के�सं�कार�सा�ह�य�का�एक�बड़ा�कोष�है।�

• �ौतसू��, गृ�सू��और�धम��सू���मलकर�क�पसू��बनाते�ह�।�

IASbaba
Web: http://ilp.iasbaba.com/ Score:
Email: ilp@iasbaba.com 3.00 / 200
Page 150
AIPTS/ILP VETERANS-
Exam Title :
2020 TE...
Email : yadavanurag075@gmail.com
Contact : 8882839768

• क�पसू���को��ह���धा�म�क�अ�यास�का�मैनुअल�माना�जाता�है।�उनम��से�कई�वेद�के��व�भ���कूल��के�भीतर�उभरे�, जो�भारत�का�सबसे
प�व��सा�ह�य�है।�
• ��येक�मैनुअल�अपने��कूल�क�����या��(क�प) क���ा�या�करता�है��य��क�यह�तीन�अलग-अलग��े�णय��पर�लागू�होता�है: य�
अनु�ान-�ौतसू��, घरेलू�अनु�ान�- गृ�सू��और�जीवन�का�आचरण�- धम�सू�।�वे�सू��क��लघु�कामो��पक�शैली�म���लखे�गए�ह�
(शा��दक��प�से�" धागा�") ता�क�वे��मृ�त�म��आसानी�से���तब��हो�सक�।�
• क�प�वेदंग�( " वेद�के��लए�सामान�") के��प�म��जाना�जाने�वाले��व�ान��के�छह��े���म��से�एक�है।�
• �ौतसू��और�गृ�सू��दोन��600-300 ईसा�पूव��के�ह�।�
• सु�वसू���ने�य��वे�दय��के��नमा�ण�के��लए�कई��कार�के�माप�बताए।�वे��या�म�त�और�ग�णत�क��शु�आत�को��च��त�करते�ह�।�

QUESTION 53.
�न�न�ल�खत�म��से��कसे�संगम�युग�का��ोत�माना�जा�सकता�है�?

1. ��ै बो�और���लनी�के�काय��
2. अशोक�के��शलालेख�
3. हाथीगु�फा��शलालेख�
4. सीलोन�क��पु�तक�' महावंश�' और�' द�पवंश�'

सही��वक�प�चुन��

a) 2 और�3
b) 1 और�4
c) 2, 3 और�4
d) 1, 2, 3 और�4
Correct Answer: D
Your Answer:
Explanation

Solution (d)

Basic Information:

• संगम�युग��ाचीन�त�मलनाडु �और�केरल�के�इ�तहास�और��ीलंका�के�कुछ��ह�स��(तब�त�मलनाडु �के��प�म��जाना�जाता�था) क��6 व�


शता�द��ईसा�पूव��से�तीसरी�शता�द��ई.पू. अव�ध�थी।�म�रई�शहर�म��क���त�क�वय��और��व�ान��के���स��संगम�अकाद�मय��के�नाम
पर�इसका�नाम�रखा�गया।�
• �व�भ��सा�ह��यक�और�पुराता��वक��ोत�संगम�युग��ोत��का��नमा�ण�करते�ह�।�
• संगम�सा�ह�य�म��मु�य��प�से�तोलक�पयम�, ए�टुथोगल�और�पाथुप��शा�मल�ह�।�ये�काय��संगम�युग�के�इ�तहास�को�जानने�के��लए
ब�मू�य�जानकारी��दान�करते�ह�।�इनम��तोलक�यम�सबसे�पहला�था।�संगम�काल�के�बाद�, प�थनन��कलकन�कू�या�अठारह�काय��क�
रचना�क��गई�थी।�जुड़वाँ�महाका�- �शल�प�दकारम�और�म�णमेखलई�- संगम�काल�के�बाद�का�था।�यह�सब�सा�ह�य�हम���ाचीन
त�मल��के�समाज�, अथ��व�था�और�सं�कृ�त�को�जानने�म��मदद�करता�है।�
• अशोक�के��शलालेख��म��चेर�, चोल�और�पां��सा�ा�य��का�उ�लेख�है।�क�ल�ग�नरेश�के�हाथीगुंफा��शलालेख�, खारवेल�म��भी�तीन
त�मल�रा�य��का�उ�लेख�है।�कलुगुमलाई��शलालेख�से�हम��त�मल����नामक��ाचीन�त�मल��ल�पय��के�बारे�म��पता�चलता�है।
�त�कोवकलुर��शलालेख��थानीय�सरदार��और�त�मल�क�व�, क�पलर�के��खद�अंत�का�उ�लेख�करता�है।�पुंगलुर�के�पास�अन�तार
पहा�ड़य��पर�पाए�गए��शलालेख��थम�शता�द��ई. के�थे�और�ये��शलालेख�चेर�राजा��के�संबंध�म��जानकारी���तुत�करते�ह�।�
• संगम�सा�ह�य�के�अलावा�, �वदे शी�सा�ह��यक�काय��संगम�युग�के�अ�ययन�के��लए�उपयोगी��ोत�ह�।��ीक�और�रोमन�लेखक��ने�अपने
लेख��म��संगम�त�मल��के�समाज�और�अथ��व�था�का�उ�लेख��कया�था।�मेग�थनीज�ने�अपनी�पु�तक�इं�डका�म��तीन�त�मल�रा�य��का
भी�उ�लेख��कया�है।�अ�य�लेखक�जैसे���ै बो�, ��लनी�और�टॉलेमी�संगम�युग�के�बारे�म��ब�मू�य�जानकारी��दान�करते�ह�।�सीलोन�क�
पु�तक��- महावंश�और�द�पवंश�- संगम�क���त�थ�तय�करने�म��हमारी�मदद�करती�ह�।�

QUESTION 54.
�न�न�ल�खत�म��से�कौन�सही��प�से�सुमे�लत�है।�

IASbaba
Web: http://ilp.iasbaba.com/ Score:
Email: ilp@iasbaba.com 3.00 / 200
Page 151
AIPTS/ILP VETERANS-
Exam Title :
2020 TE...
Email : yadavanurag075@gmail.com
Contact : 8882839768

( लेखक) ( पु�तक)

1. बाणभ��हष�च�रत�
2. सं�याकर�नंद��रामच�रत�
3. �ब�हण��व�मंकादे वचा�रत�
4. क�हण�राजतरं�गणी�

सही��वक�प�चुन�:

a) 1 और�2
b) 2 और�3
c) 1, 2 और�4
d) 1, 2, 3 और�4
Correct Answer: D
Your Answer:
Explanation

Solution (d)

Basic Information:

• बाणभ��ने�हष�च�रत�क��रचना�सातव��शता�द��ई�वी�म��क��थी।�यह�हष�वध�न�के�शु�आती�क�रयर�का�वण�न�करने�वाला�एक�अध�-जीवनी
संबंधी�काय��है।�यह�हष��के�दरबारी�जीवन�और�उनक��आयु�म��सामा�जक�और�धा�म�क�जीवन�का�एक�उ�कृ���वचार�दे ता�है।�
• सं�याकर�नंद��ने�रामच�रत��लखा।�इसम��कैवत���कसान��और�पाल�राजकुमार�रामपाल�के�बीच�संघष��क��कहानी�है।�
• �ब�हण�के��व�मणकदे वचा�रत�क�याणी�के�चालु�य�राजा�, अपने�संर�क��व�मा�द�य�VI (1076-1127 ई�वी) क��उपल��धय�
को�याद�करते�ह�।�
• " राजा��क��धारा�" राजतरं�गणी�बारहव��शता�द��म��क�हण��ारा��लखी�थी।�यह�क�मीर�के�राजा��क��आ�मकथा��क��एक�कड़ी
है।�

QUESTION 55.
�न�न�ल�खत�कथन��पर��वचार�कर�:

1. �े न��सांग�और�फा-�ान�दोन��चीनी�या�ी�बौ��थे।�
2. फ़ा-�ान�ने�गु�त�काल�म��भारत�क��सामा�जक-आ�थ�क���थ�तय��का�वण�न��कया�जब�क��े न��सांग�हष��के�काल�म��इसी�तरह�के�वृ�ांत
��तुत�करते�ह�।�

उपरो��कथन��म��से�कौन�सा�सही�है�/ ह��?

a) केवल�1
b) केवल�2
c) 1 और�2
d) कोई�नह��
Correct Answer: C
Your Answer:
Explanation

Solution (c)

Basic Information:

• फ़ा-�ान�और��े न��सांग�दोन��बौ��थे�तथा�वे�बौ��तीथ��थल��क��या�ा�करने�और�बौ��धम��का�अ�ययन�करने�के��लए�भारत�आए�थे।�

IASbaba
Web: http://ilp.iasbaba.com/ Score:
Email: ilp@iasbaba.com 3.00 / 200
Page 152
AIPTS/ILP VETERANS-
Exam Title :
2020 TE...
Email : yadavanurag075@gmail.com
Contact : 8882839768

फा-�ान:

• फा-�ान�एक�चीनी�बौ���भ�ु�और�अनुवादक�थे��ज�ह�ने��ाचीन�चीन�से��ाचीन�भारत�तक�पैदल�या�ा�क��, म�य�ए�शया�के�कई�प�व�
बौ���थल��पर�, 399-412 के�बीच�भारतीय�उपमहा��प�और�द��ण�पूव��ए�शया�का�दौरा�कर�बौ���ंथ��का�अ�ध�हण��कया।�
• फा-�ान�क��भारत�या�ा�चं�गु�त���तीय�के�शासनकाल�के�दौरान��ई�थी।�ले�कन�वह�शासक�या�अपने�काल�क��राजनी�तक���थ�त�के
बारे�म��कुछ�भी�नह��बताता�है।�
• वह�गौतम�बु��क��ज�म�थली�लु��बनी�क��तीथ�या�ा�के��लए�भी���स��है।�
• फा-�ान�ने�399 ई. म��भारत�क��या�ा�शु��क�।�उ�ह�ने�गोबी�के�रे�ग�तान�से�या�ा�क��और�खोतान�प�ँचे�जहाँ�उ�ह��कई�बौ��मठ
�मले।��फर�वह�शनशान�, ततार��दे श�और�का�गर�गया।�का�गर�का�त�कालीन�शासक�बौ��था।�इस�लए�, वह�बौ���भ�ु��से��मले
और�वहां�कई�मठ�भी�पाए।�उसके�बाद�, उ�ह�ने�पामीर�पठार�, �वात�को�पार��कया�और�गांधार��दे श�म���वेश��कया।�
• वह�भारत�म��लगभग�400 ई. तक�प�ँच�गया�और�यहाँ�411 ई. तक�रहा।�उसने�पेशावर�, पाट�लपु��, त��शला�, मथुरा�, क�ौज�,
�ाव�ती�, क�पलव�तु�, सारनाथ�और�कई�अ�य��थान�का�दौरा��कया।�
• उ�ह�ने�सामा�जक���थ�त�का�वण�न�इस��कार��कया।��जा�अपने�जीवन�से�समृ��और�संतु��थी।�साव�ज�नक�नै�तकता�अ�धक�थी।
�यादातर�लोग�शाकाहारी�थे�और�अपने�भोजन�म��मांस�और��याज�से�परहेज�करते�थे।�उ�ह�ने�शराब�और�अ�य�नशीले�पदाथ��का
उपयोग�नह���कया।�केवल�चांडाल�(अछू त) , जो�शहर��के�बाहर�रहते�थे�, �शकार�और�मछली�पकड़ने�म��लगे�थे�और�मांस�खाने�वाले
थे।�
• उ�ह�ने�ता��ल��त�(प��म�बंगाल) के�समु���बंदरगाह�से�सीलोन�के��लए�या�ा�क��शु�आत�क�।�वे�दो�साल�तक�सीलोन�(�ीलंका) म��रहे
और��फर�414 ई. म��जावा�के�ज�रए�चीन�वापस�प�ंचे।�

�च��1: फा-�ान�का�या�ा�माग��

�े न��सांग-:

• �े न-�सांग�एक�चीनी�बौ���भ�ु�थे�, �ज�ह�ने�627 ई�वी�म��बौ��धम��ंथ��को��ा�त�करने�के��लए�चीन�से�भारत�क��या�ा�क��थी।�


• वह�643 म��चीन�लौट�आया�, अपने�साथ�वह�ब�मू�य�पांडु�ल�पयाँ�लेकर�गया�, �जसका�उसने�तब�चीनी�अनुवाद��कया�था।�

IASbaba
Web: http://ilp.iasbaba.com/ Score:
Email: ilp@iasbaba.com 3.00 / 200
Page 153
AIPTS/ILP VETERANS-
Exam Title :
2020 TE...
Email : yadavanurag075@gmail.com
Contact : 8882839768

• अपनी�या�ा�के�दौरान��े न-�सांग�, ने�उन��थान��का�दौरा��कया�, �ज�ह��आज�हम�पा�क�तान�, नेपाल�, बां�लादे श�और�भारत�के��प�म�


जानते�ह�।�
• उ�ह�ने�स�ाट�हष��क��अव�ध�के�दौरान�भारत�का�दौरा��कया।�जब�वे�चीन�वापस�गए�, तो�उ�ह�ने�अपनी�पु�तक�' सी-यू-क��' या�' प��
मी�दे श��के��रकॉड��' म��हष��के�शासनकाल�के�दौरान�भारत�का��व�तृत��ववरण��लखा।�
• उनका��ववरण�उस�समय�भारत�क���शास�नक�, सामा�जक�और�सां�कृ�तक���थ�त�को�जानने�के�सव��म�उपल�ध��ोत�के��प�म�
�वीकार��कया�गया�है।�
• �े न-�सांग�ने�अपनी�या�ा�के��व�तृत��ववरण��को�छोड़��दया�, और�शु�आती�तांग�राजवंश�के�दौरान�चीनी�बौ��धम��और�भारतीय�बौ�
धम��के�बीच�बातचीत�के�बारे�म��भी��लखा।�उनक���कताब�" प��मी��े���पर�महान�तांग��रकॉड��" चीनी�सा�ह�य�म��एक��ला�सक�है�, त
था�7 व��शता�द��ई�वी�म��रेशम�माग��के�इस�खंड�के�साथ��मली�भू�म�और�लोग��के�बारे�म���ाथ�मक�जानकारी�शा�मल�है।�लगभग�900
साल�बाद�, पु�तक��म�ग�राजवंश�के�लेखक�वू�च�ग�जेन��ारा��ल�खत���स��उप�यास�" जन��टू �द�वे�ट�" के��लए��ेरणा�के��प�म��काय�
�कया।�

कथन��व�ेषण:

कथन�1 कथन�2

स�य� स�य�

फा-�ान�ने�चं�गु�त- ��तीय�के�शासनकाल�म��दौरा��कया�जब�क
दोन��या�ी�बौ��थे�
�े न-�सांग�ने�हष��के�शासनकाल�के�दौरान�दौरा��कया।�

QUESTION 56.
�न�न�ल�खत�म��से�कौन�भारत�म��नवपाषाण�युग�के��थल�ह��?

1. म�क��
2. उ�नूर�( Utnur)
3. ��हगीरी�
4. �मजा�पुर�म��बेलन�घाट��
5. हड़�पा�और�मोहनजोदड़ो�

सही��वक�प�चुन�:

a) 1, 2, 4 और�5
b) 1, 2, 3 और�4
c) 1, 2 और�3
d) 1, 2, 3, 4 और�5
Correct Answer: B
Your Answer:
Explanation

Solution (b)

Basic Information:

• नवपाषाण�श�द�दो�श�द��से�आया�है: नव�, या�नया�और��ल�थक�या�प�थर।�जैसे�, इस�समय�अव�ध�को�कभी-कभी�नव�पाषाण�युग


कहा�जाता�है।�

IASbaba
Web: http://ilp.iasbaba.com/ Score:
Email: ilp@iasbaba.com 3.00 / 200
Page 154
AIPTS/ILP VETERANS-
Exam Title :
2020 TE...
Email : yadavanurag075@gmail.com
Contact : 8882839768

• हालाँ�क�नव�पाषाण�युग�क��शु�आत�7000 ईसा�पूव��म���ई�थी�ले�कन�भारत�म��पाई�जाने�वाली�ब��तयाँ�6000 ईसा�पूव��से�अ�धक


पुरानी�नह��ह�।�द��ण�भारत�और�पूव��भारत�म��पाई�जाने�वाली�कुछ�ब��तयाँ�1000 ईसा�पूव���जतनी�पुरानी�ह�।�
• नवपाषाण�ब��तय���ारा�उपयोग��कए�जाने�वाले�कु�हा�ड़य��के��कार�के�आधार�पर�, नवपाषाण�ब��तय��के�तीन�मह�वपूण���े���को
जाना�जाता�है।�

◦ क�मीर�क��घाट��म��उ�र�म��पाया�जाने�वाला��े��बुज�होम�नामक��थान�पर�है।�
◦ गोदावरी�के�द��ण�क��ओर�द��ण�भारत�म��पाया�जाने�वाला��े�।�
◦ असम�क��पहा�ड़य��म����थत��े�।�(गारो��ह�स)

इसके�अ�त�र��नवपाषाण��थल��न�न��े���म��पाए�गए�ह�।�

1. उ�र��दे श�के��मजा�पुर�और�इलाहाबाद��जल��म���व��य�और�बेलन�घाट��के�उ�री�इलाक��म��
2. कना�टक�म��मा�क��, ���गरी�, ह�लूर�, कोडेकल�, सांगनाक�लू�, ट�. नरसीपुर�और�ट�कलको�ा�
3. त�मलनाडु �म��प�यमप�ली।�
4. आं���दे श�म���पकलीहल�और�उतनूर।�

नवपाषाण�युग�म��मनु�य��ने�अभी�भी�प�थर�के�औजार��और�ह�थयार��का�इ�तेमाल��कया�था�, ले�कन�वे�अपने�प�थर�के�औजार��को�उ�त�करने
लगे�थे।��ारं�भक�धातु��व�ान�के��माण�भी�ह��, तथा�अ�धक��म���के�बत�न��का��नमा�ण�भी�शा�मल�है।�

ले�कन��या�वा�तव�म��नव�पाषाण�युग�को��ाचीन�पाषाण�युग�, या�पुरापाषाण�युग�से�अलग�करता�है�, जो�इससे�पहले��आ�था�, यह�एक�ब�त


मह�वपूण���वशेषता�है�जो��क�इस�समय�क��मह�वपूण���वशेषता�है: कृ�ष�का�आ�व�कार।�

QUESTION 57.
भारत�म��ता�पाषाणकालीन�( Chalcolithic) चरण�के�संबंध�म���न�न�ल�खत�कथन��पर��वचार�कर�:

1. ता�पाषाणकालीन�लोग�पक���ई��ट��से�प�र�चत�थे�, जो�घर��के��नमा�ण�के��लए�उपयोग��कए�जाते�थे।�
2. ता�पाषाणकालीन�लोग��ने��व�भ���कार�के��म���के�बत�न��का�उपयोग��कया।�
3. ता�पाषाणकालीन�लोग��ने�कपास�के�साथ-साथ�गे�ं�और�चावल�का�उ�पादन��कया।�

नीचे��दए�गए�कूट�का�उपयोग�करके�सही�कथन�चुन�:

a) 2 और�3
b) केवल�1
c) 1, 2 और�3
d) कोई�नह��
Correct Answer: A
Your Answer:
Explanation

Solution (a)

Basic Information:

• नवपाषाण�काल�के�अंत�म��धातु��का�उपयोग�दे खा�गया।�तांबा�प�थर�के�साथ��योग�क��जाने�वाली�पहली�धातु�थी।�ऐसी�सं�कृ�त�को
ता�पाषाण�कहा�जाता�था।�
• इस�चरण�से�संबं�धत�शु�आती�ब��तयां�द��ण-पूव��राज�थान�, प��मी�महारा��और�पूव��भारत�म��पाई�जाती�ह�।�
• अहार�और��गलुंद�क��खुदाई�द��ण�पूव��राज�थान�म��क��गई�है।�
• मालवा�, कायथा�और�एरण�क��खुदाई�प��मी�म�य��दे श�म��क��गई�है।�
• प��मी�महारा��म��जोरवे�, नेवासा�, दायमाबाद�, चंदौली�, सोनगाँव�और�इनामगाँव�, ना�सक�क��खुदाई�क��गई�है।�
• इस�काल�के�लोग�प�थर�और�तांबे�के��लेड�से�बने�छोटे �औजार��और�ह�थयार��का�इ�तेमाल�करते�थे।�लगता�है��क��लेड�उ�ोग�इस
समय�के�दौरान�फला-फूला।�

IASbaba
Web: http://ilp.iasbaba.com/ Score:
Email: ilp@iasbaba.com 3.00 / 200
Page 155
AIPTS/ILP VETERANS-
Exam Title :
2020 TE...
Email : yadavanurag075@gmail.com
Contact : 8882839768

• उ�ह�ने��व�भ���कार�के��म���के�बत�न��का�उपयोग��कया�, �जनम��से�एक�को��लैक�और�रेड�कहा�जाता�था।�कभी-कभी�सफेद�रै�खक
�डजाइन��को��च��त��कया�जाता�था।�
• लोग��ने�पशु��को�पालतू�बनाया�तथा�कपास�स�हत�गे�ं�, चावल�, बाजरा�, मसूर�जैसे�अनाज�उगाए।�
• आम�तौर�पर�लोग�पक���ई��ट��से�प�र�चत�नह��थे�और�क�चड़�से�अपने�घर�बना�लेते�थे।�

कथन��व�ेषण:

कथन�1 कथन�2 कथन�3

अस�य� स�य� स�य�

इस�अव�ध�के�दौरान�लोग��ने��म���के�घर उ�ह�ने��व�भ���कार�के��म���के�बत�न
उ�ह�ने�कपास�, अलसी�और�बेर�के�साथ
बनाए।�उ�ह�ने�शायद�ही�कभी�पक���ई��ट� बनाए।�म�य��दे श�और�महारा��म��लोग��ने
�व�भ��खा�ा���का�उ�पादन��कया।�
का�इ�तेमाल��कया�हो।� कई��कार�के�बत�न��का�उ�पादन��कया।�

QUESTION 58.
�स�धु�घाट��स�यता�के�लोग��क��धा�म�क��थाएं�अ��तीय�थी।��न�न�ल�खत�म��से�कौन�संभवतः�इस�स�यता�के�लोग���ारा�पूजा�जाता�था�?

1. पशुप�त�
2. मातृदेवी�
3. कूबड़�वाला�बैल�
4. �व�णु�
5. पीपल�का�पेड़�

सही��वक�प�चुन�:

a) 1, 2 और�3
b) 1, 2, 3 और�4
c) 1, 2, 3 और�5
d) 1, 2, 3, 4 और�5
Correct Answer: C
Your Answer:
Explanation

Solution (c)

Basic Information:

• �स�धु�घाट��स�यता�द��ण�ए�शया�के�उ�र-प��मी��े���म��कां�य�युगीन�स�यता�थी�, जो�3300 ईसा�पूव��से�1300 ईसा�पूव��तक�थी�,


तथा�अपने�प�रप�व��प�म��2600 ईसा�पूव��से�1900 ईसा�पूव��तक�थी।�
• यह��स�धु�नद��के�घाट��म��फली-फूली।�
• स�यता�के�शहर��को�उनके�शहरी��नयोजन�, पक���ई��ट�के�घर��, �व�तृत�जल��नकासी��णा�लय��, जल�आपू�त���णा�लय��, बड़ी�गैर-
आवासीय�इमारत��के�समूह��और�ह�तकला�म��नई�तकनीक��(कारे�लयन�उ�पाद��, सील�न�काशी) और�धातु��व�ान�(तांबा�, कां�य�,
सीसा�और��टन) के��लए�जाना�जाता�था�
• मोहनजोदाड़��और�हड़�पा�के�बड़े�शहर��क��संभावना�30,000 और�60,000 ���य��के�बीच�थी।�

IASbaba
Web: http://ilp.iasbaba.com/ Score:
Email: ilp@iasbaba.com 3.00 / 200
Page 156
AIPTS/ILP VETERANS-
Exam Title :
2020 TE...
Email : yadavanurag075@gmail.com
Contact : 8882839768

• इस�स�यता�के�धा�म�क�अ�यास�अ��तीय�ह�।�उ�खनन��थल��पर�म�हला��क��कई�टे राकोटा�मू�त�याँ��मली�ह�।�एक�मू�त��म��एक�पौधे�को
एक�म�हला�के��ूण�से�बाहर��नकलते��ए��दखाया�गया�है।�यह�संभवतः�पृ�वी�क��दे वी�का���त�न�ध�व�करता�है।�मातृदेवी�से�संबं�धत
कई�कलाकृ�तयाँ�भी��थल��पर�पाई�जाती�ह�।�एक�मुहर�पर�पु�ष�दे वता�का���त�न�ध�व��कया�जाता�है।�इस�भगवान�के�तीन��सर�ह��और
उनके�स�ग�ह�।�वह�एक�हाथी�, बाघ�, राइनो�और�एक�भ�स�से��घरे�योगी�के�बैठने�क��मु�ा�म��दशा�या�गया�है।�इसे�पशुप�त�महादे व�कहा
जाता�है।�यहाँ��ल�ग�पूजा�और�म�हला�यौन�अंग��क��पूजा�के��चलन�म��भी��दखते�ह�।�वे�पीपल�के�पेड़�, कूबड़�वाले�बैल�आ�द�जानवर�
और�पेड़��क��भी�पूजा�करते�थे।�

Image 1: पशुप�त�दे व�

Image 2: मातृदेवी�

IASbaba
Web: http://ilp.iasbaba.com/ Score:
Email: ilp@iasbaba.com 3.00 / 200
Page 157
AIPTS/ILP VETERANS-
Exam Title :
2020 TE...
Email : yadavanurag075@gmail.com
Contact : 8882839768

QUESTION 59.
हड़�पा�स�यता�के�दौरान�शहर��का��वकास�एक�उ�लेखनीय��वशेषता�थी।�हड़�पा�स�यता�के�दौरान��न�न�ल�खत�म��से��कस��थान�को�शहर�माना
जा�सकता�है�?

1. कालीबंगा�
2. बनावली�
3. रंगपुर�
4. लोथल�
5. सुरकोटड़ा�

सही��वक�प�चुन�:

a) 3 और�5
b) 1, 2 और�4
c) 1, 2, 3 और�4
d) 1, 2, 3, 4 और�5
Correct Answer: B
Your Answer:
Explanation

Solution (b)

Basic Information:

• हालां�क�250 से�अ�धक���पाई��थल��को�केवल�छह�शहर��के��प�म��जाना�जाता�है�, �जनम��हड़�पा�, मोहनजोदड़ो�, च���ो�, लोथल


(गुजरात) , कालीबंगन�(राज�थान) और�बनवाली�(ह�रयाणा�म���हसार) शा�मल�ह�।�
• हड़�पा�और�मोहनजोदड़ो��स�ध��ांत�म��ह��और�दोन��पा�क�तान�के��ह�से�ह�।�
• च���ो�मोहनजोदड़ो�से�लगभग�130 �कमी�है।�
• लोथल�गुजरात�म��खंबात�क��खाड़ी�के�शीष��पर�है।�
• कालीबंगा�उ�री�राज�थान�म��है।�
• बनावली�ह�रयाणा�के��हसार��जले�म��है।�

QUESTION 60.
ऋ�वेद�म��' ग�व��' (Gavisthi) श�द�का��या�अथ��है�?

a) बढई�
b) आ�दवासी�मु�खया�
c) भू�म�क��इकाई�
d) यु��
Correct Answer: D
Your Answer:
Explanation

Solution (d)

Basic Information:

• ऋ�वेद�म��गाय��के�कई�संदभ��ह��जो�आय��को�चरवाहा�समुदाय�दशा�ती�है।�
• गाय�धन�का��तीक�थी�और�अ�धकांश�यु��गाय��को�अ�धकार�म��लेने�के��लए�होते�थे।�इस�लए�ऋ�वेद�म��गाय��के��लए�यु��या�खोज�के
�लए�श�द�को�' ग�व���' कहा�जाता�था।�

IASbaba
Web: http://ilp.iasbaba.com/ Score:
Email: ilp@iasbaba.com 3.00 / 200
Page 158
AIPTS/ILP VETERANS-
Exam Title :
2020 TE...
Email : yadavanurag075@gmail.com
Contact : 8882839768

• साथ�ही�, पुजा�रय��को��दए�गए��कसी�भी�उपहार�को�गाय��क��सं�या�के�संदभ��म��बताया�गया�था।�

QUESTION 61.
भारतीय�उप-महा��प�म��आय��के��वास�के�संबंध�म��, �न�न�ल�खत�कथन��पर��वचार�कर�:

1. आय��ने�भारतीय�उपमहा��प�म��कई�चरण��म���वास��कया।�
2. आय��जनजा�तय��ने�आपस�म��सौहाद� �बनाए�रखा�, �जससे�उ�ह��अ�य�गैर-आय��जनजा�तय��पर�शासन�करने�म��मदद��मली।�

नीचे��दए�गए��वक�प��का�उपयोग�करके�सही�कथन�चुन�:

a) केवल�1
b) केवल�2
c) 1 और�2 दोन��
d) न�तो�1 और�न�ही�2
Correct Answer: A
Your Answer:
Explanation

Solution (a)

Basic Information:

• अं�ेजी�श�द�' आय�न�' सं�कृत�के�श�द�आय��से�आया�है�, जो��क�वै�दक�भारतीय�लोग���ारा��यु��आ�म-पदनाम�है�, जो�लगभग�15


00 ईसा�पूव��भारतीय�उपमहा��प�म��चले�गए�थे।�
• म�य�ए�शया�से�आने�वाले�, खानाबदोश�लोग��के�इस�बड़े�समूह�ने��ह���कुश�पव�त�को�पार��कया�और��स�धु�घाट��स�यता�के�संपक��म�
आए�जो�उस�दौरान��वकास�के�अपने�चरम�पर�प�ंच�गई�थी।�
• आय��ने�इंडो-यूरोपीय�भाषाएं�बोल�।�
• भारत�के�रा�ते�म��आय�न�पहली�बार�ईरान�म���दखाई��दए�, जहाँ�भारतीय-ईरानी�लंबे�समय�से�रहते�थे।�

कथन��व�ेषण:

कथन�1 कथन�2

स�य� अस�य�

आय��दो��कार�के�संघष��म��शा�मल�थे।�पहला�, वे�पूव�-आय�
आय��कई�भाग��म��भारत�आए।�सबसे�शु�आती�दौर�का
से�लड़े�और��सरे�, वे�आपस�म��लड़े।�अंतर-आ�दवासी�संघष�
��त�न�ध�व�ऋग�वै�दक�लोग���ारा��कया�जाता�है�जो�1500
ने�लंबे�समय�तक�आय��समुदाय��को��हलाकर�रख��दया।
ईसा�पूव��म��उप-महा��प�म���दखाई��दए�थे।�वे��वदे शी
आय��को�पांच�जनजा�तय��म���वभा�जत��कया�गया��ज�ह�
�नवा�सय��के�साथ�संघष��म��आए��ज�ह��' दास�' ' द�यु�' कहा
पंचजन�कहा�जाता�है�और�ये�जनजा�तयाँ�आपस�म��लड़ती
जाता�है।�दास��को�लगता�है��क�वे��ारं�भक�आय��क��एक
रहती�थ�।�कभी-कभी�जनजा�तय��ने�अपने��वयं�के
शाखा�थे।�संभवतः�‘ द�यु�’ मूल��नवा�सय��का���त�न�ध�व
जनजा�तय��के�साथ�लड़ने�के��लए�गैर-आय��जनजा�तय��क�
करते�ह�।�
मदद�ली।�

QUESTION 62.

IASbaba
Web: http://ilp.iasbaba.com/ Score:
Email: ilp@iasbaba.com 3.00 / 200
Page 159
AIPTS/ILP VETERANS-
Exam Title :
2020 TE...
Email : yadavanurag075@gmail.com
Contact : 8882839768

अशोक�के��शलालेख��व�भ���ल�पय��म���लखे�गए�थे।�अशोक�के��शलालेख��म���यु���ल�पय��म��से�कौन-सी��ल�पयाँ�थ��?

1. �ा�ी�
2. खरो�ी�
3. यूनानी�
4. आरमाइक�

नीचे��दए�गए�कूट�का�उपयोग�करके�सही��वक�प�चुन�।�

a) 1 और�3
b) 1, 2 और�3
c) 1, 3 और�4
d) 1, 2, 3 और�4
Correct Answer: D
Your Answer:
Explanation

Solution (d)

Basic Information:

• अशोक�के�अ�भलेख��तंभ��पर�तीस�से�अ�धक��शलालेख��के�साथ-साथ�एका�म�और�गुफा�क��द�वार��पर�एक�सं�ह�है�, �जसका��ेय
मौय��सा�ा�य�के�स�ाट�अशोक�को��दया�जाता�है�, �ज�ह�ने�268 ईसा�पूव��से�232 ईसा�पूव��तक�शासन��कया�था।�
• �शलालेख��म��तीन�भाषा��का�उपयोग��कया�गया�था।��ाकृत�, �ीक�(पड़ोसी��ीको-बै���यन�सा�ा�य�क��भाषा�और�अशोक�के
अंतग�त�म��यूनानी�समुदाय) और�अरमाइक�(पूव��अखा�मद�सा�ा�य�क��आ�धका�रक�भाषा)।�
• चार��ल�पय��का�उपयोग��कया�गया�था।�आधु�नक�पा�क�तान�के��े��के��लए��ा�ी�और�खरो�ी��ल�पय��म���ाकृत��शलालेख��लखे�गए
थे।�यूनानी�और�आरमाइक��शलालेख��ने�अपनी�संबं�धत��ल�पय��का�उपयोग�अशोक�के��े��के�उ�र-प��मी��े���म��, आधु�नक
पा�क�तान�और�अफगा�न�तान�म���कया।�
• जब�क�अ�धकांश��शलालेख��ाकृत�म��थे�, कुछ�यूनानी�या�आरमाइक�म���लखे�गए�थे।�
• कंधार��शलालेख���भाषी��ीक-आरमाइक�है।�अशोक�का�कंधार��ीक�सं�करण�केवल��ीक�म��है�, और�मूल��प�से�संभवतः�सभी��मुख
�शलालेख�1-14 म��समा�हत�ह�।��शलालेख�म���यु���ीक�भाषा�ब�त�उ�च��तर�क��है�और�दाश��नक�शोधन�को��द�श�त�करती�है।�यह
तीसरी�शता�द��ईसा�पूव��म��हेले�नक��व��क��राजनी�तक�भाषा�क��गहन�समझ�को�भी��द�श�त�करता�है।�यह�कंधार�म��उस�समय�एक
अ�य�धक�सुसं�कृत��ीक�उप��थ�त�क��उप��थ�त�का�सुझाव�दे ता�है।�
• इसके��वपरीत�, कना�टक�और�आं���दे श�के�नए��व�जत��दे श��म��द��ण�भारत�म��उ�क�ण���शलालेख��म��, अशोक�ने��ा�ी��ल�प�के
साथ�ही�उ�र�क���ाकृत�को�संचार�क��भाषा�के��प�म��इ�तेमाल��कया।�

QUESTION 63.
भारत�म���ह���दश�न�क��छह��णा�लय��म��से�कौन�सी��न�न�ल�खत�ह��?

1. सां�य�
2. �यो�तष�
3. �याय�
4. �ाकरण�
5. योग�
6. मीमांसा�

नीचे��दए�गए�कूट�का�उपयोग�करके�सही��वक�प�चुन�:

a) 1, 2, 3 और�6
b) 1, 3, 5 और�6

IASbaba
Web: http://ilp.iasbaba.com/ Score:
Email: ilp@iasbaba.com 3.00 / 200
Page 160
AIPTS/ILP VETERANS-
Exam Title :
2020 TE...
Email : yadavanurag075@gmail.com
Contact : 8882839768

c) 1, 2, 4 और�5
d) 1, 2, 3, 4, 5 और�6
Correct Answer: B
Your Answer:
Explanation

Solution (b)

Basic Information:

�ह���दश�न�म��दाश��नक��, �व���वचार��और��श�ा��का��ज��है�जो��ाचीन�भारत�म��सामने�आए�थे।�इनम��छह��णा�लयाँ�शा�मल�ह��-

1. सां�य: एक�ना��तक�और��ढ़ता�से��ै त�सै�ां�तक�चेतना�और�पदाथ��का��व�तार।�


2. योग: �यान�, �च�तन�और�मु���पर�जोर�दे ने�वाला��वचार।�
3. �याय: �ान�के��ोत��क���ा�या��याय�सू��करता�है।�
4. वैशे�षक: परमाणुवाद�का�एक�अनुभववाद���कूल।�परमाणुवाद�एक��ाकृ�तक�दश�न�है�जो�यह���ता�वत�करता�है��क�भौ�तक��व�
मूलभूत�अ�वभा�य�घटक��से�बनी�है��ज�ह��परमाणु�के��प�म��जाना�जाता�है।�
5. मीमांसा: मीमांसा�एक�सं�कृत�श�द�है��जसका�अथ��है�" ��त�ब�ब�" या�" मह�वपूण��जांच�" और�इस�तरह��च�तन�क��परंपरा�को�संद�भ�त
करता�है�जो�कुछ�वै�दक��ंथ��के�अथ��म��प�रल��त�होता�है।�
6. वेदांत: शा��दक�अथ��" वेद��का�अंत�", वेदांत�उन��वचार��को�दशा�ता�है�जो�उप�नषद��म���न�हत�अनुमान��और�दश�न�से�उ�प���ए�ह��,
�वशेष��प�से�, �ान�और�मु��।�

इ�ह��आ��तक�(��ढ़वाद�) दाश��नक�परंपराएं�भी�कहा�जाता�है�तथा�वे�जो�वेद��को�एक�आ�धका�रक�, �ान�के�मह�वपूण���ोत�के��प�म���वीकार


करते�ह�।�

�ाचीन�और�म�ययुगीन�भारत�भी�दाश��नक�अवधारणा��को�साझा�करने�वाले�दश�न�का��ोत�था�, ले�कन�वेद��को�अ�वीकृत�कर��दया�था�, अ
तः�इ�ह��ना��तक�भारतीय�दश�न�कहा�गया।�ना��तक�भारतीय�दश�न�म��बौ��, जैन�, चावा�क�, अ�ज�वक�आ�द�शा�मल�ह�।�

वेदांग�वेद�के�अ�ययन�और�समझ�से�जुड़े�छह�सहायक��वषय�ह�।�उनमे�शा�मल�है�

1. �श�ा�(�वर-�व�ान)
2. क�प�(अनु�ान�संबंधी)
3. �ाकरण�(�ाकरण)
4. �न���(�प�ीकरण)
5. छं द�(वै�दक�लेख)
6. �यो�तष�(�यो�तष)

QUESTION 64.
5 व��और�6 व��शता�द��ईसा�पूव��के�दौरान�ईरा�नय��के�आ�मण�ने�उ�री�भारत�म��मह�वपूण���वकास�लाए।��न�न�ल�खत�म��से�कौन�सा
घटना�म�ईरानी�आ�मण�से�संबं�धत�है�?

1. खरो�ी��ल�प�का�आ�व�कार।�
2. इसने�मौय�कालीन�मू�त�कला�को��भा�वत��कया।�
3. इसने�भारत�पर��सकंदर�के�आ�मण�का�नेतृ�व��कया।�

सही�कथन�चुन��

a) 1 और�2
b) 1 और�3
c) 2 और�3
d) 1, 2 और�3

IASbaba
Web: http://ilp.iasbaba.com/ Score:
Email: ilp@iasbaba.com 3.00 / 200
Page 161
AIPTS/ILP VETERANS-
Exam Title :
2020 TE...
Email : yadavanurag075@gmail.com
Contact : 8882839768

Correct Answer: D
Your Answer:
Explanation

Solution (d)

Basic information:

• ईरान�के�अखा�मद�शासक��ने�उ�र-प��म�भारत�क���व�भ���रयासत��जैसे��क�कंबोज�, गंधार�आ�द�पर�राजनै�तक��व�े ष�का�लाभ


उठाते��ए�भारत�पर�आ�मण��कया।�
• ईरानी�शासक�डे�रयस�ने�516 ईसा�पूव��म��उ�र-प��म�भारत�म���वेश��कया�तथा�पंजाब�, �स�धु�से�प��म�म���वेश��कया।�इस��े��ने
ईरान�के�20 व���ांत�या���प�का�गठन��कया।�ईरानी�सा�ा�य�म����प��क��कुल�सं�या�28 थी।�

कथन��व�ेषण:

कथन�1 कथन�2 कथन�3

स�य� स�य� स�य�

ईरानी�लेख��ने�भारत�म��लेखन�का�एक��प मौय�कालीन�मू�त�कला�पर�ईरानी��भाव��प�
ईरा�नय��के�मा�यम�से�, यूना�नय��को
लाया��जसे�खरो�ी��ल�प�के��प�म��जाना �प�से�है।�अशोक�काल�के��मारक�, �वशेष
भारत�क��महान�संप���के�बारे�म��पता
जाता�है।�यह�दाएं�से�बाएं��लखी�जाती�थी। �प�से�घंटे�के�आकार�, राजधानी�म��ईरानी
चला�और�अंततः�भारत�के��सकंदर�ने
कुछ�अशोक�के��शलालेख�इस��ल�प�म� मू�त�कला�से�इसक��वा�तुकला�का�पता
आ�मण��कया।�
�लखे�गए�ह�।� लगाया�जा�सकता�है।�

QUESTION 65.
उ�री�काले�पो�लश�वाले�मृदभांड�( NBP) चरण�के�संबंध�म���न�न�ल�खत�कथन��पर��वचार�कर�।�

1. एनबीपी�चरण�ने�भारत�म���सरे�शहरीकरण�क��शु�आत�को��च��त��कया।�
2. इसने�पहली�बार�धा��वक�मु�ा�क��शु�आत�दे खी।�

सही�कथन�चुन��

a) केवल�1
b) केवल�2
c) 1 और�2 दोन��
d) न�तो�1 और�न�ही�2
Correct Answer: C
Your Answer:
Explanation

Solution (c)

IASbaba
Web: http://ilp.iasbaba.com/ Score:
Email: ilp@iasbaba.com 3.00 / 200
Page 162

You might also like